CR Practice Questions For GMAT

You might also like

Download as doc, pdf, or txt
Download as doc, pdf, or txt
You are on page 1of 75

CR TOPIC-WISE

EXERCISES
(700-800 LEVEL)
1
The 700-800 Club
Critical Reasoi!
Critical Reasoi! To"ic #$ CO%CL&SIO%
1. In the United States, about $5,200 per person per year is spent on health care, while in Britain the amount is about
half that. A recent study indicated that middlea!ed white Americans ha"e a si!nificantly hi!her rate of diabetes and
heart disease than do middlea!ed white Britons. #"en after eliminatin! from the study the lifestyle differences of
diet, e$ercise, smo%in!, and drin%in!, the data showed that the Americans ha"e poorer health than their British
counterparts. The statements above, if true, best support which of the following assertions?
&ealth care spendin! in the United States should be reduced by 50'.
(ore e$pensi"e health care causes a hi!her incidence of certain diseases.
)he money spent on health care in the United States is not bein! used effecti"ely.
)he a"era!e health care spendin! for middlea!ed white Americans is probably less than the a"era!e health
care spendin! for Americans in !eneral.
Somethin! other than diet, e$ercise, smo%in!, and drin%in! must account for the difference in health for the
two !roups in the study.
2. Spo%esperson* In the 200+ election of the city mayor, 55' of the "oters were female. All the "oters were
between a!es ,- and .0 and 2/0 of them supported the incumbent mayor. )he incumbent mayor won the
election with a substantially !reater number of "otes than any other candidate. If the statements made by the
Spokesperson are true, then which of the following must be true?
At least ,/2 of the female "oters supported the incumbent mayor.
)he incumbent mayor recei"ed stron!er support from the female "oters than from the male "oters.
)here were no other candidates in the election who recei"ed more than 00' of all the "otes.
15' of the "oters in the election were male and none of them were .5 years old.
If the proportion of male and female "oters in the city remains the same, the incumbent mayor is also li%ely to
win the ne$t election.
0. In 2000, the (a%in! &its 2ecord 3ompany spent 10' of its total bud!et on the production of ten albums, 00' of its
bud!et on the mar%etin! of these albums, and the remainder of its bud!et on o"erhead costs. In the same year, the
Son! 4actory 2ecord 3ompany spent 20' of its total bud!et on the production of ,0 albums and +0' of its bud!et
on the mar%etin! of these albums. (a%in! &its sold a total of -00,000 copies of the ten records it produced in 2000,
while the Son! 4actory sold a total of ,,+00,000 copies of the ten records it produced in 2000. Assuming each
company met its budget, which of the following conclusions is best supported by the information given
above?
)he amount of money spent on mar%etin! is directly related to the number of copies sold.
(a%in! &its spent more money on the production of its albums in 2000 than did the Son! 4actory.
Son! 4actory5s total re"enue from the sale of albums produced in 2000 was hi!her than that of (a%in! &its.
In 2000, (a%in! &its spent a lar!er percenta!e of its bud!et on o"erhead costs than did the Son! 4actory.
)he Son! 4actory sold more copies of its 2000 albums than (a%in! &its did because the Son! 4actory spent
a hi!her percenta!e of its bud!et on the mar%etin! of its albums.
4. 6ue to hi!h 7et fuel costs, airline carriers are loo%in! for new ways to increase re"enues and thereby counteract
declinin! profits. Airline A has proposed increasin! the number of passen!ers that can fit on its airplanes by
creatin! se"eral standin! room only 8seats9 in which passen!ers would be propped a!ainst a padded bac%board
and held in place with a harness. )his proposal, since it relates to passen!er safety, cannot be implemented
without prior appro"al by the 4ederal A"iation Administration. The above statements, if true, indicate that Airline
A has made which of the following conclusions?
)he addition of standin! room only 8seats9 will !enerate more re"enue than the cost of ensurin! that these
seats meet safety standards.
)he 4ederal A"iation Administration will appro"e Airline A5s specific proposal.
)he re"enue !enerated by the addition of standin! room only 8seats9 is !reater than the current cost of 7et fuel.
)here are no safer ways in which Airline A can increase re"enues.
:assen!er safety is less important than increasin! re"enue.
2
5. A recent research study of under!raduate students analy;ed the effects of music on human emotions. #ach
of the 200 participants attended at least , twohour concert of classical music per wee% o"er the course of ,2
wee%s of their sprin! semester. At the end of the e$periment, all of the students filled out a <uestionnaire
assessin! their emotional state. Based on the results of the <uestionnaires, all of the ,0 students who
attended the !reatest number of concerts reported lower stress le"els and hi!her satisfaction with their li"es.
Also, most of the 20 students who attended the fewest number of concerts reported belowa"era!e le"els of
emotional comfort. Which of the following must be true based on the evidence presented above?
(ost of the 200 participants impro"ed their emotional state and lowered their stress le"els.
6urin! each wee% of the e$periment, the participants spent at least 2 hours less on their academic wor% as a
result of concert attendance.
=istenin! to classical music for at least 2 hours per wee% impro"es the emotional wellbein! of the ma7ority of
youn! adults.
(ore than + participants attended at least ,1 concerts durin! the course of the e$periment.
At least some of the students participated in the study in order to !ain free access to classical concerts.
6. 3olumnist* )he winner of this year5s national spellin! bee won by correctly spellin! the spo%en word Ursprache, which
means 8fame9 in >erman. >i"en the richness of our lan!ua!e, why must we resort to words ta%en from modern forei!n
lan!ua!es to challen!e our best spellers? Ursprache is listed in our dictionary, as are words from many other forei!n
lan!ua!es, but future spellin! bees should limit themsel"es to words in our dictionary that ha"e been an!lici;ed in all
aspects because spellin! #n!lish words, not %nowled!e of lin!uistics and international phonetics, is the point of these
contests. Which of the following can most reasonably be inferred from the argument above?
)he spellin! contest winner %new how to spell most of the an!lici;ed words in the dictionary.
4orei!n words are more difficult than an!lici;ed words for all contestants to spell.
Spellin! contestant winners should be determined by their facility with all aspects of lan!ua!e.
)o spell forei!n words, contestants must reco!ni;e the lan!ua!e and %now its pronunciation.
)he #n!lish lan!ua!e contains more borrowed words than most other lan!ua!es.
.. In #astland, from 2000 to 2005, the total consumption of fish increased by 1.5 percent, and the total
consumption of poultry products increased by @.0 percent. 6urin! this time, the population of #astland
increased by + percent, in part due to new arri"als from surroundin! areas. Which of the following can one
infer based on the statements above?
4or new arri"als to #astland between 2000 and 2005, fish was less li%ely to be a ma7or part of families5 diet
than was poultry.
In 2005, the residents of #astland consumed twice as much poultry as fish.
)he per capita consumption of poultry in #astland was hi!her in 2005 than it was in 2000.
Between 2000 and 2005, both fish and poultry products were a re!ular part of the diet of a si!nificant
proportion of #astland residents.
Between 2000 and 2005, the profits of wholesale distributors of poultry products increased at a !reater rate
than did the profits of wholesale distributors of fish.
8. #"eryone who has !raduated from )opAotch &i!h School has an intelli!ence <uotient BICD of o"er ,20. (ost
students with an IC of o"er ,20 and all students with an IC of o"er ,50 who apply to one or more I"y =ea!ue
uni"ersities are accepted to at least one of them. The statements above, if true, best support which of the
following conclusions?
#"ery !raduate of )opAotch &i!h School with an IC of ,50 has been accepted to at least one I"y=ea!ue school.
If a person is a hi!hschool !raduate and has an IC of less than ,00, he or she could not ha"e been a student
at )opAotch &i!h School.
If a person has an IC of ,00 and is attendin! an I"y=ea!ue school, it is possible for him or her to ha"e
!raduated from )opAotch &i!h School.
At least one !raduate from )opAotch hi!h school who has applied to at least one I"y=ea!ue uni"ersity has
been accepted to one of them.
If a hi!hschool !raduate has an IC of ,50 and is not attendin! an I"y=ea!ue school, then he or she did not
apply to one of them.
3
9. Accordin! to a recent study on financial roles, onethird of hi!h school seniors say that they ha"e 8si!nificant
financial responsibilities.9 )hese responsibilities include, but are not limited to, contributin! to food, shelter, or
clothin! for themsel"es or their families. At the same time, a second study demonstrates that a crisis in
money mana!ement e$ists for hi!h school students. Accordin! to this study, -0' of hi!h school seniors ha"e
ne"er ta%en a personal finance class e"en thou!h the same percenta!e of seniors has opened ban% accounts
and onethird of these account holders ha"e bounced a chec%. Which of the following conclusions can be
properly drawn from the statements above?
&i!h schools would be wise to incorporate personal finance classes into their core curricula.
At least onethird of hi!h school seniors wor% parttime 7obs after school.
)he number of hi!h school seniors with si!nificant financial responsibilities is !reater than the number of
seniors who ha"e bounced a chec%.
Any hi!h school seniors who contribute to food, shelter, or clothin! for themsel"es or their families ha"e
si!nificant financial responsibilities.
)he ma7ority of hi!h school students ha"e no financial responsibilities to their families.
10. Analyst* 3reati"e professionals, such as clothin! desi!ners, !raphic desi!ners, and decorators, often ha"e
"ery poor mana!erial s%ills and do not succeed when they try to run their own businesses. In fact, most of
these creati"e types are less s%illed in business than is the a"era!e whitecollar professional who does not
wor% in a creati"e field. >enerally, creati"e talent and business acumen rarely !o hand in hand. If the
analysts argument is taken as true, which of the following statements can properly be concluded?
Ao successful businesspeople are creati"e.
Some creati"e types are not less s%illed at business than is the a"era!e whitecollar wor%er who is not creati"e.
3reati"ity precludes success in business.
Any whitecollar wor%er who is not creati"e is more successful in business than any creati"e professional.
Business is not a creati"e endea"or.
11. Ad"ocates insist that health sa"in!s accounts are an efficient method to reduce medical e$penses. &owe"er,
widespread adoption of these accounts will soon undermine the public5s health. Ene reason for this is that most
people will be reluctant to deplete their accounts to pay for re!ular pre"enti"e e$aminations, so that in many cases
a serious illness will !o undetected until it is far ad"anced. Another reason is that poor people, who will not be able
to afford health sa"in!s accounts, will no lon!er recei"e "accinations a!ainst infectious diseases. The statements
above, if true, most support which of the following?
Fealthy indi"iduals will not be affected ne!ati"ely by health sa"in!s accounts.
:ri"ate health insurance will no lon!er be a"ailable.
(ost diseases are detected durin! re!ular pre"enti"e e$aminations.
Some people without health sa"in!s accounts are li%ely to contract infectious diseases.
)he causal relationship between an indi"idual5s health and that person5s medical care has been ade<uately
documented.
12. Albinism is a rare !enetic condition that inhibits the production of melanin, or pi!mentation, in the s%in and hair.
:eople born with albinism are unusually susceptible to sunburn, melanoma, and a ran!e of other health issues that
are !enerally connected to e$cessi"e e$posure to the sun. The statements above, if true, provide the most support
for which of the following conclusions?
:eople born with albinism de"elop other biolo!ical protections a!ainst melanoma and other sunrelated
health issues.
&umans with a hi!h production of melanin can easily i!nore health issues related to e$posure to the sun.
Fhen a nonalbino person !ets sunburn, the amount of melanin produced by that person decreases.
In humans, melanin plays a role in protectin! the s%in from de"elopin! sunburn and other sunrelated ailments.
It is not possible for a person born with albinism to adopt other artificial protecti"e measures a!ainst
e$cessi"e e$posure to the sun.
4
13. 3eliac disease results from an inability of the di!esti"e tract, specifically the small intestine, to absorb !luten,
a protein found in wheat, barley, and certain other !rains. )he body5s immune system attac%s the !luten as if
the protein were a harmful patho!en, often resultin! in serious dama!e to the intestinal linin!. :eople who
suffer from celiac disease must eliminate !luten from their diets. Symptoms of the disease include abdominal
cramps, bloatin!, and anemia. If the statements above are true, which of the following assertions can be
made on the basis of them?
Anyone who suffers from celiac disease will e$perience anemia.
#liminatin! !luten from one5s diet will cure celiac disease.
:eople e$periencin! abdominal cramps, bloatin!, and anemia ha"e celiac disease.
>luten is found only in !rains.
)he human body cannot always reco!ni;e harmless substances.
14. (ay"ille Airport and Aewcomb Airport ha"e the same number of fli!ht departures each day. (ay"ille Airport
e$periences 2+ departure delays per ,00 fli!hts, while Aewcomb Airport e$periences 20 departure delays per
,00 fli!hts. Fhen delays caused by bad weather are disre!arded, (ay"ille Airport has 5 fewer departure
delays per ,00 fli!hts than Aewcomb Airport does. Which of the following conclusions is best supported by
the information given above?
Bad weather causes a !reater number of departure delays at (ay"ille Airport than at Aewcomb Airport.
En a"era!e, the weather at (ay"ille Airport is worse than it is at Aewcomb Airport.
(echanical problems cause a !reater number of delays at Aewcomb Airport than at (ay"ille Airport.
)he fleet of airplanes lea"in! from Aewcomb Airport is better e<uipped to handle inclement weather than the
fleet of airplanes lea"in! from (ay"ille Airport.
(ay"ille Airport e$periences a !reater number of arri"al delays per ,00 fli!hts than Aewcomb Airport does.
,5. )he head ba%er at Barry5s Ba!els can either purchase flour inperson from the local flour mill, =arry5s =ocal (ill, or
order a shipment of flour from an outofstate mill, Isadore5s Interstate (ill. )he cost of the flour from Isadore5s
Interstate (ill is ,0 percent less than the cost of the flour from =arry5s =ocal (ill. #"en after shippin! and
handlin! fees are added, it is still cheaper to order flour that has to be shipped from Isadore5s than to buy flour
locally from =arry5s. The statements above, if true, best support which of the following assertions?
:roduction costs at Isadore5s Interstate (ill are ,0 percent below those at =arry5s =ocal (ill.
Buyin! flour from Isadore5s Interstate (ill will eliminate ,0 percent of the local flour mill 7obs.
)he shippin! and handlin! fees for a batch of flour purchased from Isadore5s Interstate (ill are less than ,0
percent of the cost of an identical batch of flour purchased from =arry5s =ocal (ill.
)he shippin! and handlin! fees for a batch of flour purchased from Isadore5s Interstate (ill are more than ,0
percent of the cost of Isadore5s flour.
Isadore5s Interstate (ill produces flour ,0' more efficiently than =arry5s =ocal (ill does.
16. Bo$ office receipts for independent mo"ies for the first half of this year ha"e increased by 20 percent o"er the total
receipts for independent mo"ies for all of last year. =ast year, 50 independent mo"ies were released, while so far
this year only 20 independent mo"ies ha"e been released. )he number of independent mo"ies slated for release in
the second half of this year is rou!hly e<ual to the number released so far. If the statements above are true,
which of the following must be true?
)he total bo$ office receipts for independent mo"ies this year will be si!nificantly more than 20 percent
!reater than the receipts for independent mo"ies last year.
)he number of independent mo"ies released in the first half of this year is e<ual to the number released in the
first half of last year.
)he price of a mo"ie tic%et has not increased since last year.
)he a"era!e re"enues of the independent films released durin! the first half of this year is !reater than that of
all independent films released last year.
)he number of people seein! independent mo"ies durin! the first half of this year is !reater than the number
who saw independent mo"ies last year.
5
,.. Accordin! to a recent ma!a;ine article, of those office employees who typically wor% - hours at the office
each day but sometimes say that they will wor% at home on a particular day, 25 percent actually wor% less
than one hour. At the same time, o"er @0 percent of those same office employees belie"e they are more
producti"e wor%in! at home than wor%in! in their office. The statements above, if true, best support which of
the following conclusions about the office employees discussed in the article?
En a"era!e, the office employees wor%in! at home for a day wor% fewer hours than office employees wor%in!
at the office.
,0 percent of the office employees are less producti"e wor%in! from home than wor%in! in their office.
At least ,5 percent of the office employees do not define producti"ity e$clusi"ely in terms of the number of
hours wor%ed.
At least 25 percent of the office employees can complete the same amount of wor% in one hour at home as in
- hours at the office.
Some of the office employees ma%e statements re!ardin! their producti"ity that are not in fact true.
,-. En (onday, 6aisy5s =emonade Stand sold lemonade at 20 cents per cup. )he =emon Shac% sold lemonade
at 00 cents per cup. At the end of the day, 6aisy5s =emonade Stand and the =emon Shac% reported identical
re"enues and identical profits. The statements above best support which of the following assertions?
En (onday, 6aisy5s =emonade Stand sold fewer cups of lemonade than did the =emon Shac%.
)he =emon Shac% sells hi!her <uality lemonade than does 6aisy5s =emonade Stand.
En (onday, 6aisy5s =emonade Stand and the =emon Shac% incurred identical costs to run their businesses.
In !eneral, lemonade consumers prefer the lemonade at 6aisy5s =emonade Stand to the =emonade at the
=emon Shac%.
)he =emon Shac% would not increase its re"enues by lowerin! its prices.
19. >o"ernment restrictions ha"e se"erely limited the amount of stem cell research American companies can conduct.
Because of these restrictions, many American scientists who speciali;e in the field of stem cell research ha"e
si!ned lon!term contracts to wor% for forei!n companies. 2ecently, 3on!ress has proposed liftin! all restrictions
on stem cell research. Which of the following conclusions can most properly be inferred from the information
above?
At least some forei!n companies that conduct stem cell research wor% under fewer restrictions than some
American companies do.
Because American scientists are under lon!term contracts to forei!n companies, there will be a si!nificant
influ$ of forei!n professionals into the United States.
In all parts of the world, stem cell research is dependent on the financial bac%in! of local !o"ernment.
In the near future, American companies will no lon!er be at the forefront of stem cell research.
If restrictions on stem cell research are lifted, many of the American scientists will brea% their contracts to
return to American companies.
20. As many as @-,000 people die each year due to medical error. In a campai!n to reduce lethal errors, thousands of
hospitals introduced si$ %ey chan!es, includin! rapidresponse teams, rechec%s of patient medication, and new
!uidelines for pre"entin! infection. )he campai!n estimated that, o"er an ,-month period, more than ,00,000 li"es
were sa"ed as a direct result of the pro!ram. Which of the following can be most properly inferred from the
above statements?
6octors and nurses should be more careful when doin! their 7obs.
)he campai!n sa"ed all of the people who otherwise would ha"e died due to medical error in that time period.
In the future, no one will die because of medical error.
If the campai!n had not been implemented, more than ,00,000 people mi!ht ha"e died durin! the ,-month
period due to medical error.
)he %ey chan!es initiated by the campai!n will continue to be implemented in the future.
6
21. 4ederal law prohibits businesses from reimbursin! any employees for the cost of ownin! and operatin! a
pri"ate aircraft that is used for business purposes. )hus, many American companies themsel"es purchase
pri"ate aircraft. )he "ast ma7ority of the business a"iation fleet is owned by small and midsi;e businesses,
and fli!hts are strictly for business purposes, with mostly midle"el employees on board. )hese companies
and their boards of directors are in full compliance with the law and with what is best for their businesses.
Which of the following can be most properly inferred from the statements above?
)he 4ederal law in <uestion costs businesses money.
(ost e$ecuti"es would rather fly on company owned planes than on commercial airlines.
=ar!e businesses usually ha"e their e$ecuti"es fly first or business class on commercial fli!hts.
Upper le"el e$ecuti"es are less often in compliance with the law.
By not recei"in! any reimbursement for these fli!hts, the midle"el e$ecuti"es on board are complyin! with
the law.
22. Antoine* )he alarmin! fact is that amon! children a!ed ,@ years and youn!er, the number ta%in!
antipsychotic medicines soared .0 percent in the last four years. )hat is !reater than the increase in the
number of adults ta%in! antipsychotic medicines durin! the same period.
=ucy* But the use of antipsychotic dru!s by adults is considered normal at the current rate of ,, adults per
,,000 ta%in! the dru!s. In contrast, the number of children on antipsychotic medication last year was +.+ per
,,000 children. !ucys argument is structured to lead to which of the following as a conclusion?
)he current le"el of antipsychotic dru! use in children is abnormally hi!h.
)he fact that the number of children ta%in! antipsychotic medicines increased .0 percent o"er the last four
years is not an indication that the current le"el of use is abnormally hi!h.
If only +.+ out of e"ery ,,000 children are ta%in! an antipsychotic medication, the increase in the use of such
medicines cannot be the percenta!e Antoine cites.
It is unli%ely that the increase in the use of antipsychotic medicines by children will continue at the same rate.
If the number of children ta%in! antipsychotic dru!s is !i"en as a certain number, the actual rate of such dru!
use is e"en hi!her.
20. )he ability to analy;e !enomes G se<uences of 6AA G has !rown more and more sophisticated. Scientists are
able to e$amine the biolo!ical past in finer detail and with !reater accuracy. A new analysis of the !enetic lin%s
between early humans and chimpan;ees has led to a hypothesis that the two species di"er!ed more recently than
pre"ious estimates indicated. )his !i"es credence to a "ery startlin! new theory* the ancestors of humans and
chimpan;ees mi!ht ha"e hybridi;ed to produce the linea!e from which modern humans e"entually de"eloped.
What can be inferred from the statements above?
&ybridi;ed species combine the best of both linea!es.
:olitical and reli!ious pressures will affect the course of future research.
4uture research will enable scientists to determine the e$act characteristics of this hybrid ancestor of modern
man.
#arlier !enome analysis established !enetic lin%s between early humans and chimpan;ees.
&uman 6AA is more comple$ than that of chimpan;ees.
24. )he restaurant business wastes more ener!y than any other industry in the United States. Aearly -0 percent of the
$,0 billion spent on ener!y by the restaurant industry each year is s<uandered by the use of inefficient e<uipment.
At the same time, appro$imately .0 percent of restaurants in the United States are small businesses that are
usually too cash poor to in"est in ener!yefficient technolo!y. Which of the following statements draws the most
reliable conclusion from the information above?
)he a"ailability of ener!yefficient e<uipment will reduce the ener!y costs of the restaurant industry by
appro$imately 00 percent.
Ao industry in the United States spends !reater than $,0 billion each year on ener!y.
By usin! ener!yefficient technolo!y, a small restaurant will reduce its e$penses by a !reater percenta!e than
will a lar!e restaurant.
Appro$imately $2 billion of the amount spent on ener!y each year by the restaurant industry is not s<uandered.
)he replacement of inefficient e<uipment represents the lar!est potential source of ener!y sa"in!s for the
restaurant industry.
7
25. Under a new clean air proposal, the !o"ernment has decided to ti!hten controls on the release of certain to$ic
chemicals, includin! ben;ene, formaldehyde, and other carcino!ens, by chemical plants. )he stated purpose
of this proposal is to reduce cancers caused by air pollution. Het, the chemical industry, rather than the
!o"ernment, is responsible for monitorin! the implementation of the proposal. If the past actions of certain
polluters in the chemical industry are any indication of future beha"ior, the net result of the new proposal will
be an increase, rather than a decrease, in carcino!ens released into the air. The author is arguing that """"#
no chemical companies can be trusted to follow the clean air proposal
the chemical industry is responsible for releasin! the ma7ority of carcino!ens into the air
allowin! selfmonitorin! for the new clean air proposal will result in the opposite of its intended conse<uence
to ensure effecti"e implementation, the !o"ernment should always monitor the e$ecution of its proposals
ben;ene and formaldehyde are two of the most ha;ardous cancercausin! chemicals
2+. Since ,@@5, 3on!ress has e$empted oil companies that ha"e leases issued by the federal !o"ernment allowin!
them to drill for deepwater oil off the >ulf of (e$ico from royalty payments as an incenti"e to spur de"elopment in
times of low oil and !as prices. )hese leases were supposed to ha"e included a pro"ision that reinstates the
royalties should the mar%et prices of oil and !as e$ceed a certain le"el. Because of an error by the federal
!o"ernment, howe"er, the lan!ua!e that reinstates the royalties is missin! from the more than ,,,00 leases issued
by the U.S. !o"ernment in ,@@- and ,@@@. Since the mar%et price of oil and !as has recently risen far abo"e the
threshold le"els, this error could allow the oil companies to reap a windfall of more than $,0 billion throu!h the life
of the leases. In response, the !o"ernment is pressurin! the oil companies to rene!otiate the leases. )he
e$ecuti"es of the oil companies stron!ly oppose rene!otiationI all ha"e issued statements statin! that they e$pect
the !o"ernment to honor the terms of the contracts and that rene!otiatin! a duly si!ned a!reement would set a
bad precedent. Which of the following statements best reflects the position of the oil company e$ecutives?
Epportunity seldom %noc%s twice.
6o unto others as you would ha"e done unto you.
Ene man5s loss is another man5s !ain.
Hou don5t chan!e the rules in the middle of the !ame.
2e"en!e is so sweet.
2.. Enly those students who maintain 2.5 !rade point a"era!es are allowed to participate in school sports. Amy is
captain of the school5s tennis team, so she must ha"e at least a 2.5 >:A. Which of the following statements best
summari%es the main point of the above argument?
Students who don5t maintain a 2.5 >:A can5t participate in sports.
Amy is a !ood tennis player.
)he school should only re<uire a minimum >:A of 2.0 to participate in sports.
Amy has earned at least the minimum >:A re<uired to participate on a school sports team.
Amy wouldn5t be captain of the tennis team if her >:A were lower.
2-. 3ompanies are often torn between the benefits of focusin! on one ma7or product or ser"ice and the drawbac%s of
relyin! too hea"ily on one primary source of income. Fhile narrow focus can pro"ide a company with an ad"anta!e
o"er competitors that offer a wider ran!e of products or ser"ices, an undi"ersified income stream can lea"e a
company susceptible to ma7or fluctuations in cash flow. Fe can see this tension reali;ed when, for e$ample,
JJJJJJJJJJJJJJJJ. Which of the following best completes the passage below?
,. a local messen!er ser"ice %nown for its speedy deli"eries is forced to lay off twenty percent of its wor% force
after a rise in local ta$es encoura!es many local businesses to mo"e out of state
2. an ad"ertisin! a!ency loses one of its clients
0. a holdin! company that owns a car rental a!ency and a national dou!hnut chain is now interested in
purchasin! a professional bas%etball team
1. a construction company opts to use nonunion labor to increase its profits
5. a specialty sandwich store decides to open franchises throu!hout the country that will focus on usin! local in!redients
8
29. 3alorie restriction, a diet hi!h in nutrients but low in calories, is %nown to prolon! the life of rats and mice by
pre"entin! heart disease, cancer, diabetes, and other diseases. A si$month study of 1- moderately
o"erwei!ht people, who each reduced their calorie inta%e by at least 25 percent, demonstrated decreases in
insulin le"els and body temperature, with the !reatest decrease obser"ed in indi"iduals with the !reatest
percenta!e chan!e in their calorie inta%e. =ow insulin le"el and body temperature are both considered si!ns
of lon!e"ity, partly because an earlier study by other researchers found both traits in lon!li"ed people. If the
above statements are true, they support which of the following inferences?
3alorie restriction produces similar results in humans as it does in rats and mice.
&umans who reduce their calorie inta%e by at least 25 percent on a lon!term basis will li"e lon!er than they
would ha"e had they not done so.
3alorie inta%e is directly correlated to insulin le"el in moderately o"erwei!ht indi"iduals.
Indi"iduals with low insulin le"els are healthier than indi"iduals with hi!h insulin le"els.
Some indi"iduals in the study reduced their calorie inta%e by more than 25 percent.
30. =ast Kanuary, in an attempt to lower the number of traffic fatalities, the state le!islature passed its 83lic% It or
)ic%et9 law. Under the new law, motorists can be pulled o"er and tic%eted for not wearin! their seat belts,
e"en if an additional dri"in! infraction has not been committed. =awyers and citi;ens5 !roups are already
protestin! the law, sayin! it unfairly infrin!es on the ri!hts of the state5s dri"ers. =aw enforcement !roups
counter these claims by statin! that the new re!ulations will sa"e countless additional li"es. Which of the
following inferences is best supported by the passage above?
:rior to the 83lic% It or )ic%et9 law, motorists could not be stopped simply for not wearin! a seat belt.
)he 83lic% It or )ic%et9 law "iolates current search and sei;ure laws.
=aws similar to 83lic% It or )ic%et9 ha"e effecti"ely reduced traffic fatalities in a number of states.
)he pre"ious seatbelt laws were ineffecti"e in sa"in! li"es.
=aw enforcement !roups, rather than citi;ens !roups, should determine how to best ensure the safety of motorists.
31. In a certain state, huntin! permits re<uire the hunter to be at least ,- years old, possess a "alid dri"er5s
license or state identification, and ha"e completed a safety pro!ram within the past 5 years. )he hunter must
also si!n a le!al document pled!in! not to consume alcohol while huntin!. Which of the following can be
correctly inferred from the above statements?
All states ha"e the same re<uirements for huntin! permits.
&unters under the a!e of 2, don5t need to si!n the alcohol pled!e because they are not le!ally allowed to
drin% alcohol under any circumstances.
If a person last completed the safety pro!ram si$ years a!o, he will ha"e to complete the pro!ram a!ain
before he can be eli!ible for a permit.
Someone who isn5t ,- years old cannot obtain a huntin! permit in this state.
&untin! is such a dan!erous acti"ity that state controls and re<uirements are necessary to ensure that
nobody !ets hurt.
32. )he public often protests when an unre!ulated ser"ice industry is found to be corrupt. &owe"er, re!ulation often
leads to increased costs for the consumer. 4ewer companies sur"i"e in a re!ulated mar%et, leadin! to decreased
competition and hi!her prices. )he public then responds ne!ati"ely to the increased costs of these ser"ices. The
statements above best support which of the following?
Ser"ice industries should not be re!ulated.
)he public should not protest unre!ulated ser"ices.
Enly unre!ulated ser"ices are sub7ect to public protest.
)he public is sometimes the cause of its own complaints.
6ecreased competition always leads to increased prices.
00. A certain medication used to treat mi!raine headaches acts by bloc%in! pain receptors in the brain. Fhen a person
ta%es the medication within one hour after in!estin! !rapefruit or !rapefruit 7uice, howe"er, the effecti"eness of
the medication is si!nificantly diminished. 2esearchers ha"e determined that the !rapefruit contains a compound
that alters the shape of the pain receptors, with the result that the medication can no lon!er bind with them
completely. Which of the following conclusions could be most properly drawn from the information given above?
If one ta%es the medication more than an hour after in!estin! !rapefruit, its effecti"eness is not diminished.
In!estin! !rapefruit after ta%in! the medication does not diminish the effecti"eness of the medication.
)here is only one type of pain receptor in the brain.
)he medication is fully effecti"e only when it properly binds with its tar!et pain receptors.
It is not possible to desi!n a medication for mi!raine headaches that can bond with the altered receptors.
9
01. #ducator* =i%e any other difficult pursuit, music re<uires intense study and practice in order for one to become
proficient. But many school music pro!rams encoura!e only children who demonstrate early aptitude to
continue studyin! music, while children who are not especially musical are directed towards other acti"ities.
&a"in! learned to thin% of themsel"es as musically inept, these children do not de"ote any time to music and
thus depri"e themsel"es of the opportunity to de"elop a latent talent. The educators statements, if true,
would best support which of the following conclusions?
(usic education should not de"ote special attention to talented students.
#"eryone has the potential to learn music.
)alent is not always apparent at an early a!e.
3hildren are particularly sensiti"e to criticism from adults.
All children should study music.
05. Impro"ed technolo!y and e<uipment often result in fewer in7uries durin! hi!hris% acti"ities such as roc%
climbin! and scuba di"in!. But participant education also plays a lar!e role in reducin! the number of in7uries
sustained durin! these acti"ities. :eople who are poorly trained in these acti"ities run a much hi!her ris% of
in7ury e"en if pro"ided with the latest and best e<uipment. Which of the following can be properly inferred
from the information above?
)rainin! is a more important safety factor than e<uipment in hi!hris% acti"ities.
:eople who are properly trained in their acti"ities do not sustain in7uries.
)he safety benefits of the latest e<uipment can be offset by inade<uate preparation.
2oc% climbin! and scuba di"in! are more ris%y than any other acti"ities.
:eople with the latest e<uipment often ne!lect proper trainin!.
36. )he new heart scans offer patients si!nificant benefits. )hey can be completed in a fraction of the time re<uired for an
an!io!ram, with no reco"ery time necessary. 4urthermore, the scans are more sensiti"e and can identify problem areas
that an an!io!ram mi!ht not percei"e. &owe"er, heart scans use more radiation than most dia!nostic procedures, and
can cause undue concern o"er and treatment for the harmless abnormalities often pic%ed up by such sensiti"e
technolo!y. Which of the following conclusions is best supported by the statements above?
A heart scan is safer than an an!io!ram procedure.
:atients should not be concerned about heart abnormalities that appear in a heart scan.
A heart scan could result in indirect harm by causin! a patient to under!o ris%y, unnecessary procedures.
An an!io!ram is the more appropriate of the two procedures for most patients.
)he heart scan is a more e$pensi"e procedure than the an!io!ram.
Critical Reasoi! To"ic '$ (ssu)"tios
,. )o decrease the number of crimes in city H, the city5s :olice 3ommissioner proposed ta%in! some police
officers from lowcrime districts of the city and mo"in! them to hi!hcrime districts of the city. &is proposal is
based on city H crime data that show that the number of crimes in any district of the city decreases when
additional police officers are mo"ed into that district. The &olice 'ommissioners proposal depends on which
of the following assumptions?
3ity L e$perienced a drastic reduction in crime after implementin! a proposal similar to that proposed by the
:olice 3ommissioner of city H.
)he se"erity of crimes committed in any district of the city decreases when additional police officers are
mo"ed into that district.
)he number of crimes committed in all hi!hcrime districts of city H is more than triple the number of crimes
committed in all lowcrime districts of city H.
)here are more lowcrime districts than hi!hcrime districts in city H.
6istricts of the city from which police officers are remo"ed do not e$perience si!nificant crime increases
shortly after the remo"al of those officers.
10
2. Althou!h there has been !reat scientific debate for decades o"er !lobal warmin!, most scientists now a!ree
that human acti"ity is causin! the #arth5s temperature to rise. )hou!h predictions "ary, many !lobal warmin!
e$perts belie"e that a"era!e !lobal temperatures will rise between three and ei!ht de!rees 4ahrenheit durin!
the ne$t century. Such an increase would cause an alarmin! rise in sea le"els, displacin! millions of people
by destroyin! ma7or population centers alon! the world5s coastlines. Which of the following is an assumption
in support of the arguments conclusion?
Aew technolo!ical de"elopments in the ne$t century will not di"ert risin! seas from the world5s coastal cities.
Indi"iduals will not become more aware of the steps they can ta%e to reduce the emission of !reenhouse !ases.
2isin! sea le"els similarly affect all coastal population centers.
Some !lobal warmin! e$perts predict a !reater than ei!ht de!ree 4ahrenheit increase in !lobal temperatures
durin! the ne$t century.
&uman acti"ity is the sole cause of increasin! !lobal temperatures.
3. 8)he new &itIt4ar dri"er is the only !olf club that uses spacea!e )itaniumMryptonium alloy to stren!then its
head. )his dri"er is SE !ood, the last ,2 winners of the ma7or tour championships ha"e all recently switched
to itN Isn5t it time for you to add power to your swin! and distance to your dri"es? )rade in your old dri"er today
G and &itIt4ar tomorrowN9 All of the following claims are either implied or made e$plicitly in the above
advertisement ()'(&T*
Switchin! to the &itIt4ar dri"er will impro"e your play.
)he &itIt4ar dri"er helped the last ,2 ma7or championship winners achie"e their "ictory.
(a7or championship winners are e$perts and %now what constitutes a !reat !olf club.
Hour e$istin! dri"er is inferior to the &itIt4ar dri"er.
Enly &itIt4ar !olf clubs ha"e )itaniumMryptonium alloy in the head of their dri"ers.
1. 2esearchers studyin! the spread of the Blac% :la!ue in si$teenthcentury #n!land claim that certain people sur"i"ed the
epidemic because they carried a !enetic mutation, %nown as 6elta02, that is %nown to pre"ent the bacteria
that causes the :la!ue from o"erta%in! the immune system. )o support this hypothesis, the researchers tested the direct
descendants of the residents of an #n!lish town where an unusually lar!e proportion of people sur"i"ed the :la!ue. (ore
than half of these descendants tested positi"e for the mutation 6elta02, a fi!ure nearly three times hi!her than that found
in other locations. The researchers hypothesis is based on which of the following assumptions?
6elta02 does not pre"ent a carrier from contractin! any disease other than the :la!ue.
)he :la!ue is not similar to other diseases caused by bacteria.
6elta02 did not e$ist in its current form until the si$teenth century.
Ao one who tested positi"e for 6elta02 has e"er contracted a disease caused by bacteria.
)he :la!ue does not cause !enetic mutations such as 6elta02.
5. )he popular notion that a tree5s a!e can be determined by countin! the number of internal rin!s in its trun% is
!enerally true. &owe"er, to help re!ulate the internal temperature of the tree, the outermost layers of wood of the
Bra;ilian ash often peel away when the temperature e$ceeds @5 de!rees 4ahrenheit, lea"in! the tree with fewer
rin!s than it would otherwise ha"e. So only if the temperature in the Bra;ilian ash5s en"ironment ne"er e$ceeds @5
de!rees 4ahrenheit will its rin!s be a reliable measure of the tree5s a!e. Which of the following is an assumption
on which the argument above depends?
)he !rowth of new rin!s in a tree is not a function of le"els of precipitation.
Enly the Bra;ilian ash loses rin!s because of e$cessi"e heat.
Enly one day of temperatures abo"e @5 de!rees 4ahrenheit is needed to cause the Bra;ilian ash to lose a rin!.
)he internal rin!s of all trees are of uniform thic%ness.
)he number of rin!s that will be lost when the temperature e$ceeds @5 de!rees 4ahrenheit is not predictable.
6. )he 6epartment of &omeland Security has proposed new federal re<uirements for dri"er5s licenses that would
allow them to be used as part of a national identification system. Usin! licenses for purposes not directly related to
operatin! a motor "ehicle is unAmerican because it would re<uire U.S. citi;ens to carry the e<ui"alent of 8papers.9
Such a re<uirement would allow the !o"ernment to restrict their mo"ements and acti"ities in the manner of
totalitarian re!imes. In time, this could ma%e other limits on freedom acceptable. The author assumes which of
the following?
)he ne$t presidential election will be dishonest, as has happened in eastern #uropean countries.
)he !o"ernment will soon start curtailin! the acti"ities of those it considers 8dissidents.9
Blan%et restrictions on lawabidin! indi"iduals are contrary to the traditions of American culture and law.
)he ma7ority of Americans are not willin! to !i"e up their ri!ht to tra"el and mo"e about without identification.
Americans should resist all !o"ernment re!ulation of their li"es.
11
.. Since the new publisher too% control, a news ma!a;ine5s co"ers ha"e featured only models and mo"ie stars.
:re"iously, the co"ers had displayed only politicians, soldiers, and business leaders. A leadin! !ossip columnist
claimed that the chan!es made the ma!a;ine rele"ant a!ain. &owe"er, many newspaper editorials disa!reed and
su!!ested that the new publisher is more interested in boostin! sales than in reportin! important news e"ents.
Which of the following is an assumption necessary for the argument made by the gossip columnists opponents?
)he charitable acti"ities of models and mo"ie stars often focus public attention on pressin! problems.
4inal authority for choosin! the co"er sub7ect of the ma!a;ine lies with the publisher.
A ma!a;ine can boost sales while hi!hli!htin! the co"era!e of important world leaders.
Some of the mo"ie stars featured are now runnin! for political office.
(a!a;ine issues with models or mo"ie stars on the co"ers are purchased at a rate more than three times
!reater than is the case with issues featurin! politicians on the co"ers.
-. In response to the increasin! cost of producin! ener!y throu!h traditional means, such as combustion, many utility
companies ha"e be!un in"estin! in renewable ener!y sources, chiefly wind and solar power, hopin! someday to rely on
them completely and thus lower ener!y costs. )he utility companies claim that althou!h these sources re<uire si!nificant
initial capital in"estment, they will pro"ide stable ener!y supplies at low cost. As a result, these sources will be less ris%y
for the utilities than nonrenewable sources, such as !as, oil, and coal, whose prices can fluctuate dramatically accordin!
to a"ailability. The claim of the utility companies presupposes which of the following?
)he public will embrace the de"elopment of wind and solar power.
Ao new deposits of !as, oil, and coal will be disco"ered in the near future.
Feather patterns are consistent and predictable.
)he necessary technolo!y for con"ersion to wind and solar power is not more e$pensi"e than the technolo!y
needed to create ener!y throu!h combustion.
Ebtainin! ener!y from nonrenewable sources, such as !as, oil and coal, cannot be made less ris%y.
9. Sur"eys consistently show that the bestsellin! ice cream fla"or is "anilla, althou!h those who prefer chocolate
rarely order "anilla. Oanillafla"ored candy, then, probably sells better than chocolatefla"ored candy. Which of
the following is an assumption upon which the author of the argument relies?
Because someone prefers "anilla ice cream does not mean he prefers "anillafla"ored candy.
3hildren who prefer "anilla ice cream also tend to li%e chocolate candy.
)hose who prefer neither "anilla nor chocolate ice cream also prefer other fla"ors of candy.
Someone who prefers "anilla ice cream may still order chocolate on occasion.
:references for certain ice cream fla"ors are similar to preferences for candy fla"ors.
10. )he media claim that the economy is enterin! a phase of !rowth and prosperity. )hey point to lower unemployment
rates and increased producti"ity. )his analysis is false, thou!h. )he number of people filin! for ban%ruptcy has
increased e"ery month for the last si$ months, and ban%ruptcy lawyers report that they are busier than they ha"e
been in years. Which of the following is an assumption on which the argument depends?
Unemployment rates are not useful indicators of !rowth and prosperity.
#conomic !rowth cannot be measured in terms of producti"ity.
=e!islation has not been recently passed to ma%e le!al ban%ruptcy easier to obtain.
)here has not been an increase in the number of ban%ruptcy lawyers.
)he media often misrepresent the current state of economic affairs.
11. A newly disco"ered paintin! on wooden panel by (ichelan!elo must ha"e been completed after ,50. but before
,50@. It cannot ha"e been painted earlier than ,50. because one of its central fi!ures carries a coin that was not
minted until that year. It cannot ha"e been painted after ,50@ because it contains a pi!ment that (ichelan!elo is
%nown to ha"e abandoned when a cheaper alternati"e became a"ailable in that year. Which of the following is an
assumption on which the argument depends?
Ao stoc%s of the abandoned pi!ment e$isted after ,50@.
(ichelan!elo did not wor% on the paintin! o"er the course of se"eral years.
)he coin depicted in the paintin! was %nown to !eneral public in ,50..
)he wooden panel on which the paintin! was e$ecuted cannot be tested accurately for a!e.
(ichelan!elo5s paintin! style did not chan!e between ,50. and ,50@.
12
12. In order to sa"e money, some of 3ompany L5s manufacturin! plants con"erted from oil fuel to natural !as last year, when
the cost of oil was more than the cost of natural !as. Because of a sudden, une$pected shorta!e, howe"er, natural !as
now costs more than oil, the price of which has fallen steeply o"er the past year. )he cost of con"ersion bac% to oil would
more than ne!ate any cost sa"in!s in fuel. So 3ompany L5s fuel costs this year will be si!nificantly hi!her than they were
last year. Which of the following is an assumption on which the argument above depends?
3ompany L does not ha"e money set aside for the increased costs of fuel.
)he increase in the cost of fuel cannot be offset by reductions in other operatin! e$penses.
)he price of natural !as will ne"er a!ain fall below that of oil.
)he cost of fuel needed by those of 3ompany L5s plants that con"erted to natural !as is not less than the cost
of fuel needed by those plants still usin! oil.
)he price of oil will not e$perience a sudden and steep increase.
13. Ad"ertisement* A"ian Eculars are the newest in binocular technolo!y for the professional bird watcher. 4or a
price comparable to that of traditional binoculars, A"ian Eculars are specially desi!ned with the features
demanded by birdin! enthusiasts* they are li!htwei!ht, compact and e$tremely durable. So, use A"ian
Eculars on your ne$t bird watchin! e$cursion, and see some of the world5s rarest species in all of their
beautiful pluma!e. Which of the following is an assumption on which the author of the advertisement relies?
A"ian Eculars wei!h less than traditional binoculars.
A"ian Eculars should be used only by bird watchers.
)he reader will tra"el throu!h the habitats of rare species durin! his or her ne$t bird watchin! trip.
A"ian Eculars are similar in cost to traditional binoculars.
Birdin! enthusiasts often determine the specifications of new products.
,1. #"ery year many people become ill because of airborne mold spores in their homes. After someone becomes ill,
specialists are often hired to eradicate the mold. )hese specialists loo% in damp areas of the house, since mold is
almost always found in places where there is substantial moisture. If one wishes to a"oid mold poisonin!, then,
one should ma%e sure to %eep all internal plumbin! in !ood condition to pre"ent lea%a!e that could ser"e as a
breedin! !round for mold. Which of the following is an assumption on which the argument depends?
(old itself does not create moisture.
(ost homeowners %now enou!h about plumbin! to determine whether theirs is in !ood condition.
(old cannot !row in dry areas.
Ao "arieties of mold are harmless.
(old spores cannot be filtered from the air.
15. )he le!islature of the :hilippines "oted recently to abolish the death penalty. In contrast, the death penalty remains
le!al in the United States. )he difference in the le!ality of capital punishment demonstrates that the ma7ority of
American citi;ens belie"e in the death penalty, while the ma7ority of 4ilipino citi;ens do not. Which of the following
is an assumption upon which the above argument depends?
)here are more murders per year in the United States than in the :hilippines.
)he le!al status of capital punishment in the United States and the :hilippines ali!ns with how the ma7ority of
citi;ens in those respecti"e countries "iew the death penalty.
)here are not stron! "oices opposin! the death penalty in the United States.
(ost American citi;ens who belie"e in the death penalty thin% that it acts as a deterrent for potential criminals,
while most 4ilipino citi;ens do not.
)he le!al standard used to determine whether a criminal should be sentenced to the death penalty in the United
States is similar to the le!al standard used in the :hilippines before capital punishment was abolished there.
16. Smo%in! is a %nown cause of certain serious health problems, includin! emphysema and lun! cancer. Aow, an
additional concern can be added to the list of maladies caused by smo%in!. A recent study sur"eyed both smo%ers
and nonsmo%ers, and found that smo%ers are si!nificantly more an$ious and ner"ous than nonsmo%ers. Which of
the following is an assumption on which the argument rests?
An$iety and ner"ousness can lead to serious health problems.
An$iety and ner"ousness do not ma%e indi"iduals more li%ely to start smo%in!.
#<ui"alent numbers of smo%ers and nonsmo%ers were sur"eyed for the study.
Smo%ers are aware of the "arious health problems attributed to smo%in!, includin! lun! cancer and emphysema.
Smo%ers who had smo%ed a ci!arette immediately before respondin! to the sur"ey were more an$ious and
ner"ous than smo%ers who had not smo%ed for se"eral hours.
13
17. E"er the past 5 years, 3ompany L has posted doubledi!it !rowth in annual re"enues, combined with a substantial
impro"ement in operatin! mar!ins. Since this !rowth is li%ely to persist in the future, the stoc% of 3ompany L will
soon e$perience dramatic appreciation. The argument above is based on which of the following assumptions?
3ompany L has a lar!e mar%et share in its industry.
:rior to the last 5 years, 3ompany L had e$perienced similarly dramatic !rowth in sales associated with
stable or impro"in! operatin! mar!ins.
)he !rowth of 3ompany L is li%ely to persist in the future.
)he current price of the stoc% of 3ompany L does not fully reflect the promisin! !rowth prospects of the firm.
)he stoc% of 3ompany L will outperform other stoc%s in the same industry.
,-. Antoine* )he alarmin! fact is that amon! children a!ed ,@ years and youn!er, the number ta%in!
antipsychotic medicines soared .0 percent in the last four years. )hat is !reater than the increase in the
number of adults ta%in! antipsychotic medicines durin! the same period.
=ucy* But the use of antipsychotic dru!s by adults is considered normal at the current rate of ,, adults per
,,000 ta%in! the dru!s. In contrast, the number of children on antipsychotic medication last year was +.+ per
,,000 children. !ucys argument relies on the assumption that """"""#
normal le"els of antipsychotic dru! use are rarely e$ceeded.
the percenta!e of adults ta%in! antipsychotic medication is always hi!her than the percenta!e of children on
such medication.
the use of antipsychotic medication in children is no different from the use of such medications in adults.
Antoine is not consciously distortin! the statistics he presents.
a rapid increase in the number of children ta%in! antipsychotic dru!s !enerates more fear of random "iolence
by adolescents than does %nowled!e of the absolute number of children on such medications.
,@. A recent article stated that only 5.5' of American colle!es !rant the ma7ority of their de!rees in the liberal arts.
3itin! this, a reader wrote to lament that this was further e"idence of the decline of academic ri!or in American
post hi!h school education. Which of the following is an assumption on the part of the reader?
)he percenta!e of American colle!es !rantin! liberal arts de!rees would continue to drop.
All colle!es should !rant the ma7ority of their de!rees in the liberal arts.
(ost postsecondary scientific, en!ineerin!, and "ocational trainin! does not in"ol"e as much academic ri!or
as liberal arts trainin!.
Academic ri!or is the most important aspect of post hi!h school education.
Ef the colle!es that do not !rant the ma7ority of their de!rees in the liberal arts, many !ranted fewer than a
<uarter of their de!rees in the liberal arts.
20. 6octor* 2esearch shows that adolescents who play "ideo !ames on a re!ular basis are three times as li%ely
to de"elop carpal tunnel syndrome as are adolescents who do not play "ideo !ames. 4ederal le!islation that
prohibits the sale of "ideo !ames to minors would help curb this painful wrist condition amon! adolescents.
The doctors conclusion depends on which of the following assumptions?
)he ma7ority of federal le!islators would "ote for a bill that prohibits the sale of "ideo !ames to minors.
Aot all adolescents who play "ideo !ames on a re!ular basis suffer from carpal tunnel syndrome.
:layin! "ideo !ames is the only way an adolescent can de"elop carpal tunnel syndrome.
(ost parents would refuse to purchase "ideo !ames for their adolescent children.
)he re!ular playin! of "ideo !ames by adolescents does not produce such beneficial effects as better hand
eye coordination and impro"ed reaction time.
21. 3ompany L recei"es most of its re"enues from the sale of !asoline throu!h a networ% of !as stations that it owns
across the country. )he company purchases readyforsale !asoline from se"eral oil refineries at wholesale prices
and sells it to the final consumer at its !as stations. E"er the ne$t <uarter, the mana!ement of 3ompany L e$pects
that the mar%et price of !asoline will rise by appro$imately ,0 percent. )herefore, the mana!ement pro7ects that the
ne$t <uarter5s re"enues from the sale of !asoline will also increase by appro$imately ,0 percent. The
managements pro+ection is based on which of the following assumptions?
3onsumption of !asoline at the company5s !as stations will not drop in response to hi!her prices.
3ompany profits will not decline below their current le"el.
&i!her !asoline prices will not reduce the company5s re"enues from other business lines.
)he costs of !asoline purchased by the company for subse<uent sale at its !as stations will remain relati"ely
constant.
)he supply of !asoline is li%ely to decline o"er the ne$t <uarter.
14
22. An oil field prospector and de"eloper reported a lar!e oil deposit in southwestern )e$as. As a result, a lar!e oil and
!as company purchased the field with the intention of drillin! oil wells in the area soon afterwards. &owe"er, the
company found that what had been reported to be a lar!e oil deposit was actually much smaller than had been
indicated. )hus, the methods that the prospector had used to determine the si;e of the oil deposit must ha"e been
inaccurate. Which of the following is an assumption on which the argument depends?
)he company5s methods of measurin! the si;e of the oil deposit were determined by a third party to be more
accurate than those used by the prospector.
)he prospector did not purposefully fabricate or misrepresent the si;e of the oil deposit.
)hou!h smaller than ori!inally thou!ht, the oil deposit contained enou!h oil to ma%e drillin! commercially feasible.
)he prospector did not e$plore other oil fields and use the same methods to determine the ma!nitude of the
oil present, if any.
)he company had successfully drilled for oil in other lar!e oil fields in )e$as throu!hout the early twentieth
century.
20. :arent* )he city education department is unable to distin!uish between annoyances and important problems. 4or
instance, prohibitin! students from ha"in! cell phones is an o"erreaction. If a student uses one and thus interferes
with instruction, confiscate it. All in all, we need educational leadership that can sol"e problems, not create them.
Which of the following is an assumption made by the parent?
Students ha"in! cell phones does not constitute an important problem for the city schools.
Students ha"e no need for cell phones in school.
4aculty and staff should be allowed to possess cell phones.
Students need to ha"e cell phones because some of them ha"e no stayathome parent.
An interest in sol"in! problems is the most important attribute of an educational leader.
21. )he #r!onomic Society conducted a study that indicated that many people de"elop se"ere bac% problems durin!
adulthood, and that "irtually all such people who recei"ed chiropractic treatment showed !reat impro"ement.
)herefore, in order to minimi;e the proportion of the population that suffers from bac% pain, the #r!onomic
Society recommended that chiropractic treatment be directed toward those adults who suffer from se"ere
bac% problems. Which of the following is an assumption on which the argument depends?
Any person who recei"es chiropractic treatment for bac% pain may also benefit from other forms of treatment.
=ar!e insurance carriers co"er chiropractic care for bac% problems to a lesser de!ree than they do other
medical treatments.
Indi"iduals who recei"e chiropractic or other treatment prior to de"elopin! se"ere bac% problems are not less
li%ely to de"elop bac% pain than those who do not.
3hiropractic treatment is more effecti"e in treatin! se"ere bac% problems when utili;ed o"er a lon! period of
time, as opposed to sporadically.
Se"ere bac% pain and other problems often cause indi"iduals to miss wor%days.
25. Since the new publisher too% control, a news ma!a;ine5s co"ers ha"e featured only models and mo"ie stars.
:re"iously, the co"ers had displayed only politicians, soldiers, and business leaders. A leadin! !ossip columnist
claimed that the chan!es made the ma!a;ine rele"ant a!ain. &owe"er, many newspaper editorials disa!reed and
su!!ested that the new publisher is more interested in boostin! sales than in reportin! important news e"ents.
Which of the following is an assumption necessary for the argument made by the gossip columnists opponents?
)he charitable acti"ities of models and mo"ie stars often focus public attention on pressin! problems.
4inal authority for choosin! the co"er sub7ect of the ma!a;ine lies with the publisher.
A ma!a;ine can boost sales while hi!hli!htin! the co"era!e of important world leaders.
Some of the mo"ie stars featured are now runnin! for political office.
(a!a;ine issues with models or mo"ie stars on the co"ers are purchased at a rate more than three times
!reater than is the case with issues featurin! politicians on the co"ers.
15
2+. 4orprofit colle!es ser"e far fewer students than either public or pri"ate nonprofit colle!es. At the same time,
relati"e to nonprofit colle!es, forprofit colle!es draw a disproportionate share of federal and state financial
aid, such as tuition !rants and !uaranteed loans, for their students. It must be, then, that forprofit colle!es
enroll a !reater proportion of financially disad"anta!ed students than do nonprofit colle!es. The conclusion
above depends on which of the following assumptions?
:ublic nonprofit colle!es and pri"ate nonprofit colle!es enroll a similar proportion of financially
disad"anta!ed students.
4orprofit colle!es do not en!a!e in fraudulent practices in helpin! their students obtain unneeded federal
and state financial aid.
)he number of students recei"in! federal and state financial aid at forprofit colle!es is !reater than the
number of students recei"in! federal and state financial aid at nonprofit colle!es.
4orprofit colle!es are of similar educational <uality as nonprofit colle!es.
)he ma7ority of students at forprofit colle!es do not default on repayment of their loans after they complete
colle!e.
27. 2ecent research has indicated that married people are not only happier than unmarried people, but also
healthier. )his study has been widely reported by the media, with most commentators concludin! that bein!
married is !ood for one5s health and attitude. The conclusion of the media commentators depends on which
of the following assumptions?
)he lon!er people are married, the happier and healthier they become.
(arried couples who had a lar!e, e$tra"a!ant weddin! are happier than those who had a small, simple ceremony.
(arried people cannot !et depressed.
Sin!le people with depression or health problems are 7ust as li%ely to !et married as are other sin!le people.
Some marria!es are more harmonious than others.
28. 3ountry L imposes hea"y tariffs on imported manufactured !oods. 3ompany H has determined that it could
increase its profits in the lon! term by openin! a factory in 3ountry L to manufacture the !oods that it currently
produces in its home country for sale in 3ountry L. ,or 'ompany -s determination to be true, which of the
following assumptions must also be true?
3ompany H will be able to obtain all the necessary permits to open a factory in 3ountry L.
3ompany H currently produces no !oods outside its home country.
A sustainable mar%et for 3ompany H5s !oods currently e$ists in 3ountry L.
3ompany H5s home country does not impose tariffs on imported !oods.
=abor costs in 3ountry L are lower than those in 3ompany H5s home country.
29. 3ountry L imposes hea"y tariffs on imported manufactured !oods. 3ompany H has determined that it could
increase its profits in the lon! term by openin! a factory in 3ountry L to manufacture the !oods that it
currently produces in its home country for sale in 3ountry L. ,or 'ompany -s determination to be true, which
of the following assumptions must also be true?
3ompany H will be able to obtain all the necessary permits to open a factory in 3ountry L.
3ompany H currently produces no !oods outside its home country.
A sustainable mar%et for 3ompany H5s !oods currently e$ists in 3ountry L.
3ompany H5s home country does not impose tariffs on imported !oods.
=abor costs in 3ountry L are lower than those in 3ompany H5s home country.
00. Fhen a company refuses to allow other companies to produce patented technolo!y, the consumer in"ariably loses.
)he company that holds the patent can char!e e$orbitant prices because there is no direct competition. Fhen the
patent e$pires, other companies are free to manufacture the technolo!y and prices fall. 3ompanies
should therefore allow other manufacturers to license patented technolo!y. The argument above
presupposes which of the following?
3ompanies cannot find le!al ways to produce technolo!y similar to patented technolo!y.
3ompanies ha"e an obli!ation to act in the best interest of the consumer.
)oo many patents are !ranted to companies that are unwillin! to share them.
)he consumer can tell the difference between patented technolo!y and inferior imitations.
3onsumers care more about price than about <uality.
16
31. Inor!anic pesticides remain acti"e on the surfaces of fruits and "e!etables for se"eral days after sprayin!,
while or!anic pesticides dissipate within a few hours after application, lea"in! the surface of the sprayed
produce free of pesticide residue. )herefore, when purchasin! from a farm that uses inor!anic pesticides, one
must be careful to wash the produce thorou!hly before eatin! it to pre"ent the in!estion of to$ins. But one
need not worry about in!estin! pesticides when purchasin! from farms that use only or!anic pesticides. The
argument above assumes that
3onsumers are aware of the ori!ins of the produce they purchase.
:roduce from farms that use or!anic pesticides reaches the consumer within hours after it is pic%ed or har"ested.
Ao farm uses both or!anic and inor!anic pesticides.
Ao pesticide is capable of penetratin! the s%in of a fruit or "e!etable.
)he use of either type of pesticide does not increase the cost of produce.
32. )he downturn in the economy last year has prompted many companies to ma%e widely publici;ed layoffs, resultin!
in thousands of lost 7obs. #conomists predicted that these layoffs would cause people !enerally to cut bac% on their
discretionary spendin!, e"en if their 7obs were secure, in anticipation of comin! hard times. &owe"er, this prediction
has not come to pass, since there has been no increase in the amount of money set aside by the !eneral public in
sa"in!s accounts. Which one of the following is an assumption on which the argument depends?
)he economy has not impro"ed in recent months.
)here has been no increase in the amount of money in"ested in stoc%s, certificates of deposit, or other
sa"in!s "ehicles.
Salaries ha"e decreased as a result of the economic downturn.
Ao business sectors ha"e seen !rowth in recent months.
)hose who were laid off ha"e been able to find other employment.
00. Because most hospitals suffer a chronic undersupply of physicians, patients must sometimes wait hours in the
emer!ency room to see a doctor. Aurses should therefore perform initial e$aminations in hospital emer!ency
rooms to determine which patients merit immediate treatment and which can wait until the emer!ency physicians
ha"e more time to see them. Which of the following is an assumption on which the argument above is based?
&ospitals should e$pand their medical staffs.
:hysicians cannot be trained to perform initial e$aminations themsel"es.
#mer!ency rooms will run more smoothly if initial e$aminations are performed.
&ospitals are always fully staffed with nurses.
Aurses are competent to 7ud!e the se"erity of patients5 conditions.
01. Scientists ha"e disco"ered a new species of butterfly that li"es only in a small re!ion of 3entral America and is
acti"e only at ni!ht. 6urin! the day, it rests in treetops, where its !reen color matches the folia!e perfectly.
)herefore, the scientists must ha"e disco"ered the butterfly at ni!ht. The ar!u)et *e"e*s o +hich o,
the ,ollo+i! assu)"tios-
)he newly disco"ered butterfly is not related to any other species of butterfly.
)here is no way for the scientists to detect the butterfly durin! the day.
Ao other butterfly species li"es in this re!ion of 3entral America.
)he folia!e in the butterfly5s habitat is completely !reen.
)he butterfly cannot sur"i"e in areas outside of 3entral America.
Critical Reasoi! To"ic .$ WE(/E%
,. A certain baseball team has 7ust completed its season. In stadiums that seat 20,000 or fewer people, the team
a"era!ed , home run per !ameI in stadiums that seat between 20,000 and 10,000 people, the team a"era!ed 2
home runs per !ameI and, in stadiums that seat 10,000 or more people, the team a"era!ed 0 home runs per !ame.
Eb"iously, the e$citement of playin! in front of lar!e crowds moti"ated the team to hit more home runs.
Assuming that all stadiums during the season were filled to capacity, which of the following, if true, most undermines
the argument above?
)he team5s leadin! home run hitter hit more home runs in midsi;ed stadiums than in lar!e stadiums.
)he fans in the lar!er stadiums often cheered a!ainst the team.
)he team a"era!ed only 2 home runs per !ame when playin! in the lea!ue5s lar!est stadium.
In order to create seatin! for the additional fans, the outfield walls in the lar!er stadiums were constructed
closer to home base.
)he team5s announcer cited crowd noise as a ma7or moti"ator for the team.
17
2. )he recent decline in the employment rate was spurred by predictions of slow economic !rowth in the comin!
year. &owe"er, those predictions would not ha"e affected the employment rate if it had not been for the lac%
of capital reser"es of ma7or industries. So if ma7or industries increase their capital reser"es, the employment
rate will not decline in the future. Which of the following, if true, casts the most doubt on the validity of the
argument above?
(a7or industry foresaw the drop in employment.
Some ma7or industries had appreciable capital reser"es.
An increase in labor costs could ad"ersely affect the employment rate.
)he !o"ernment could pass le!islation mandatin! that ma7or industries set aside a fi$ed amount as capital
reser"es e"ery year.
)he drop in the employment rate was more se"ere this year than last.
0. 4armers in de"elopin! countries claim that the United States !o"ernment, throu!h farm subsidies, is responsible
for the artificially low !lobal price of wheat. Because the U.S. !o"ernment buys whate"er wheat American
farmers are unable to sell on the open mar%et, American farmers ha"e no incenti"e to modulate the si;e of
their crops accordin! to the needs of the !lobal mar%et. As a result, American farmers routinely produce more
wheat than the !lobal mar%et can absorb and the !lobal price of wheat is %ept low. Fithout these subsidies,
the farmers in de"elopin! economies claim, American farmers would produce only the amount of wheat that
they could sell on the open mar%et and the !lobal price of wheat would rise. Which of the following, if true,
most weakens the claims of the farmers in developing countries regarding the price of wheat?
Fheat that is not processed for consumption is often used for certain industrial applications.
Aon!o"ernmental buyers of wheat and wheat products are able to predict how much wheat they will need
se"eral years in ad"ance.
)he United States !o"ernment offers similar subsidies to soybean farmers, thou!h the !lobal price of
soybeans is si!nificantly hi!her than that of wheat.
Ether countries, such as 3anada and 2ussia, are li%ely to produce more wheat if the United States were to
reduce its output.
)he price of sor!hum, a crop for which the United States !o"ernment offers no subsidies, is lower than that of
wheat.
1. )he amount of money estimated to be lost by &ollywood studios due to 6O6 piracy o"erseas increased to more
than $500 million last year. As a conse<uence, &ollywood studios ha"e little prospect of ma%in! money this
year by e$portin! their films worldwide. All of the following, if true, weaken the argument e$cept*
$500 million represents a "ery small fraction of o"erseas profits.
Aew laws and hei!htened international enforcement aimed at this piracy were implemented in Kanuary of this
year and ha"e been effecti"e.
#"en in countries where 6O6 piracy is widespread, theatrical firstrun distribution is both "ery popular and profitable.
=ast year, all films that were nominated for an Academy Award were %nown to ha"e been pirated on 6O6.
6omestic 6O6 sales were sharply hi!her last yearI in the past, stron! domestic sales ha"e been an indicator
of hi!her international 6O6 sales in the followin! year.
5. :olitical Analyst* Because our city is a border city, ille!al immi!ration is an important issue in the current race for
mayor. Ef the two candidates for mayor, one supports a plan that would attempt to deport the city5s @,000 ille!al
immi!rants and the other does not. Sur"eys consistently show that about +0' of the city5s residents are opposed to
the plan, while about 05' are in support of the plan. )herefore, the candidate who does not support the plan will
win the election for mayor. All of the following statements weaken the analysts argument, ()'(&T*
In the city at issue, most "oters ma%e their "otin! decisions based on the candidates5 positions on abortion.
Ef the 05' of residents who support the plan, some are willin! to consider alternate plans for addressin!
ille!al immi!ration.
(any of the residents who oppose the plan are not re!istered "oters.
)he candidate who supports the plan is the incumbent mayor, and has been elected to four consecuti"e terms
despite ta%in! contro"ersial positions on many important issues.
Kust under 00' of the city5s residents are ille!al immi!rants who cannot "ote.
18
6. Adam will spend $1,000 to rent a booth at the town5s annual county fair to promote his new Sandwich
Shoppe. In pre"ious years, the a"era!e food booth at the fair ser"ed 100 customers. )hus, in order to co"er
his costs, Adam has calculated that he must sell 100 sandwiches for $,0 each. Which of the following
statements casts the most doubt on Adams chances of breaking even at the county fair?
Adam should not limit himself to 100 sandwichesI he should be prepared to sell more.
Adam has not factored in the costs of raw materials or labor in calculatin! the amount of re"enue he needs to
!enerate to brea% e"en.
If booths in pre"ious years a"era!ed 100 customers, that means some booths ser"ed more than 100 and
some ser"ed fewer than 100.
(any people prefer other types of food to sandwiches.
If Adam shares his booth, and the cost of the rental, with a complementary business, he will not ha"e to sell
as many sandwiches to brea% e"en.
.. )he people of :rohibitionland are considerin! bannin! the ser"ice of alcoholic be"era!es in restaurants to
curb unruly beha"ior on the part of its residents. :roprietors of restaurants in :rohibitionland are protestin!
the ban on the !rounds that it will reduce their re"enues and profits. &owe"er, se"eral pro"inces in
:rohibitionland enacted restrictions on alcoholic be"era!es last year, and the sales ta$es paid by the
restaurants in those pro"inces rose by an a"era!e of 50 percent. In contrast, the sales ta$es paid by
restaurants located in areas of :rohibitionland that did not ha"e any restrictions rose by an a"era!e of 00
percent. Which of the following, if true, supports the restaurant proprietors economic stance against the ban?
In the pro"inces that restricted alcoholic be"era!es, there was a shortterm ne!ati"e impact on restaurant
"isitation in the be!innin! of last year.
)he sales ta$ in :rohibitionland is lower on food and be"era!es than it is on other consumer !oods, such as
clothin!.
)he consumption of alcoholic be"era!es in :rohibitionland has been on a !radual decline the last 20 years.
)he restrictions on alcoholic be"era!es enacted last year allowed for the ser"ice of drin%s be!innin! around
dinnertime each e"enin!.
E"erall sales ta$ re"enue did not increase at a substantially hi!her rate in the pro"inces that enacted the
restrictions on alcoholic be"era!es than in the rest of :rohibitionland last year.
-. 2ecently, some critics of the U.S. !o"ernment ha"e pointed out that this country is the only ad"anced industriali;ed nation
without a national "accine laboratory and su!!ested that this lac% ma%es the American public more "ulnerable than other
de"eloped nations to infectious diseases, such as a"ian flu. A !o"ernment official said these critics were disloyal and thus
wron! about the public5s "ulnerability. )o support his claim, the official cited the !enerally lon! life span and low infant
mortality of United States citi;ens, relati"e to all United Aation member nations. (entionin!
the hi!h <uality of American hospitals, he added that all of the #uropeans that he %new preferred to under!o ma7or
medical treatments in the United States rather than in the sociali;ed medical systems in place in their home
countries. All of the following are weaknesses or potential weaknesses in the officials argument ()'(&T*
)he hi!h <uality of hospitals in the United States is not a factor affectin! the public5s "ulnerability to infectious
disease.
Fhether or not the critics are disloyal has no bearin! on whether or not they are wron!.
)he #uropeans that the official cited are a demo!raphically narrow sample, o"erwhelmin!ly composed of
wealthy males o"er the a!e of fifty.
)he a"era!e life span of United States citi;ens is determined not only by deaths due to infectious diseases
but also by deaths due to all other causes.
3omparin! the United States to all United Aations member nations does not address the concern that the
U.S. is behind other ad"anced industriali;ed nations in a particular way.
19
9. #$perts estimate that insurance companies5 tardiness in payin! doctors for le!itimate medical claims adds
appro$imately ,0 percent in o"erhead costs for physicians. Insurance companies counter that the tardiness
sometimes results from billin! errors made by the doctors themsel"es. Since dealin! with these billin! errors
costs the insurance companies time and money, it is clear that insurance companies do not ha"e a si!nificant
economic incenti"e to delay claim payments to doctors. Which of the following pieces of information, if true,
weakens the conclusion above?
Some doctors who submit accurate bills to insurance companies still recei"e tardy payments.
)he cost to the insurance companies to process incorrect bills from doctors5 offices is rou!hly e<ui"alent to
the increased costs that physicians accrue as a result of tardy payments from insurance companies.
A risin! proportion of medical claims submitted by doctors to insurance companies are deemed ille!itimate by
those insurance companies.
)he billin! errors made by doctors5 offices are typically "ery minor, such as the submission of a claim with an
outdated patient home address.
)he o"erhead costs incurred by doctors as a result of delayed insurance payments result in an increase in the
premiums paid by consumers to health insurance companies that far e$ceeds any increase in the fees paid to
doctors by insurance companies.
,0. Ef the people who mo"ed from one state to another when they retired, the proportion who retired to SunState
has decreased by ,0 percent o"er the past fi"e years. Since many local businesses in SunState cater to
retirees, this decline is li%ely to ha"e a noticeably ne!ati"e economic effect on these businesses. Which of the
following, if true, most seriously weakens the argument?
SunState attracts more people who mo"e from one state to another when they retire than does any other state.
)here are far more local businesses in SunState that cater to tourists than there are local businesses that
cater to retirees.
)he number of retirees who ha"e mo"ed out of SunState to accept reemployment in other states has
increased o"er the past fi"e years.
SunState has lower property ta$es than any other state, ma%in! the state a ma!net for retirees.
)he total number of people who retired and mo"ed to another state for their retirement has increased
si!nificantly o"er the past fi"e years.
11. (ar%et Analyst* 2ecent research confirms that the main cause of bad breath is bacteria buildup on the
ton!ue. )he research also concludes that ton!ue scrapers, when used properly, can eliminate up to 10' of
the bacteria from the ton!ue. As the effecti"eness of ton!ue scrapers becomes more widely %nown, the
mar%et for less effecti"e breath freshenin! products, such as mints, !ums, and sprays, will decline
si!nificantly. Which of the following provides the best evidence that the analysts argument is flawed?
Some breath freshenin! products are ad"ertised to eliminate up to 00' of the bacteria from the ton!ue.
)on!ue scrapers ha"e already been on the mar%et for a number of years.
(any dentists recommend re!ular flossin!, and not the use of the ton!ue scraper, to combat bad breath.
A recent sur"ey shows that @1' of those who re!ularly purchase breath freshenin! products are aware of the
effecti"eness of the ton!ue scraper.
Some people buy breath freshenin! products for reasons other than to fi!ht bad breath.
12. 3ompanies that offer 8employer sponsored insurance9 B#SID pay a portion of employees5 health care costs. In the
manufacturin! sector last year, companies that offered #SI had wor%er absentee rates 22' lower, on a"era!e, than
those at companies that did not offer #SI. If, on the basis of the evidence above, it is argued that (SI
decreases worker absenteeism, which of the following, if true, would most seriously weaken that argument?
2esults similar to those cited for the manufacturin! sector ha"e been found in other sectors of the economy
where #SI is offered.
At companies that offer #SI, employees ha"e access to pre"entati"e health care such as re!ular chec%ups,
routine laboratory tests, and nutrition counselin!.
Because initiatin! an #SI plan re<uires a lot of paperwor% for the company, employees, and the insurance
pro"ider, doin! so is comple$ and timeconsumin!.
(any firms in the manufacturin! sector ha"e impro"ed wor%place safety and decreased the occurrence of on
the7ob in7uries in the last fi"e years, and most of these companies introduced #SI at the same time.
In manufacturin! firms where #SI is offered, the a"era!e producti"ity is 2' hi!her than it is in those firms
where wor%ers are not co"ered by an #SI plan.
20
13. Studies ha"e shown that an automobile that runs on a blend of -5' ethanol/,5' !asoline !ets better
milea!e than an otherwise similar car e<uipped with a !asoline en!ine. (any American le!islators ha"e
concluded that an increase in ta$ incenti"es for ethanol production would lessen our dependence on forei!n
oil. Which of the following, if true, casts the most doubt upon the validity of the legislators conclusion?
It ta%es ,.5 !allons of oil to produce , !allon of ethanol.
#lectric cars are cheaper to operate than cars runnin! on the ethanol fuel mi$.
It costs thousands of dollars to retrofit an automobile to run on the ethanol fuel mi$.
)he ethanol/!asoline blend emits more pollution that re!ular !asoline.
)he ethanol/!asoline blend has not been widely adopted in #urope.
,1. )he American 2e"olution arose partly in response to British !eneral search warrants, which !a"e arbitrary and
intrusi"e powers to !o"ernment officers. )hus, the founders created the 4ourth Amendment to protect a!ainst
unreasonable and warrantless intrusions of pri"acy by a powerful and partisan federal !o"ernment. 6urin! the
20th century, countless dictators ha"e used arbitrary and intrusi"e sur"eillance to monitor and suppress
dissidents. Any democracy that does not enforce e<ually e$tensi"e protections will systematically suppress
dissent. Which of the following, if true, best weakens the argument?
)he United Min!dom5s parliamentary system does not ha"e such e$tensi"e protections but has ne"er
systematically suppressed dissent.
(any dictators ha"e been "ery beneficial and constructi"e leaders for their countries.
(any democracies ha"e hapha;ardly used sur"eillance to monitor dissidents and suppress dissent durin!
times of war.
Some dictators ha"e been supported by the United States.
At least some countries in the former So"iet Union now ha"e democratic elections but still suppress dissent.
15. A researcher studyin! corporate e$ecuti"es found that they tend to ha"e 8ta%e char!e9 personalities, with the
predominant traits of asserti"eness, decisi"eness, and selfconfidence. )he researcher concluded that people who
are more 8ta%e char!e9 than the a"era!e person are more li%ely to become corporate e$ecuti"es. Which of the
following, if true, most seriously weakens the researchers conclusion?
&oldin! the 7ob of a corporate e$ecuti"e causes people to de"elop 8ta%e char!e9 personality traits.
Fhen wor%in! on charitable or community pro7ects, corporate e$ecuti"es often use their ability to ma%e
decisions and lead people to ma%e those pro7ects successful.
Some people who are not e$ecuti"es ha"e stron!er 8ta%e char!e9 personalities than some people who
currently ser"e as corporate e$ecuti"es.
(any people who aspire to become e$ecuti"es e$hibit different mana!ement styles in their current 7obs.
)he e$ecuti"es that the researcher studied were often unsuccessful when they tried to mana!e their family
acti"ities as they do their business acti"ities.
,+. Because of a rare type of fun!us that %illed off many cacao trees in Bra;il, there was an unusually mea!er
har"est of cacao beans this year. )he wholesale price of cocoa solids and cocoa butter has increased
si!nificantly and is unli%ely to fall in the foreseeable future. As a result, the retail price of chocolate is certain
to increase within si$ months. Which of the following, if true, most seriously weakens the argument above?
3onsumers will purchase other sweets if the price of chocolate increases.
2esearchers ha"e disco"ered an effecti"e method to %ill the fun!us.
6ar% and bittersweet "arieties of chocolate will be affected more seriously than mil% "arieties.
)he price of chocolate has decreased steadily for three years.
(ost chocolate in stores is manufactured from cocoa that was purchased two years earlier.
17. A recent sur"ey conducted in the Aew &omes ma!a;ine indicated that more than .0' of its readers are plannin!
to purchase or build a new home o"er the ne$t two years. Based on the results of the sur"ey, the editor of the
ma!a;ine concluded that o"er the ne$t two years, the real estate industry is li%ely to e$perience rapid !rowth dri"en
by the hi!h mar%et demand for new homes. Which of the following statements, if true, would be the most
damaging to the conclusion of the maga%ines editor?
)he sur"ey included o"er 50,000 respondents representin! all ma7or a!e cate!ories.
)he sur"ey was mailed to all the subscribers of the ma!a;ine and included prepaid posta!e to facilitate
returnin! the <uestionnaire.
2eaders of the Aew &omes ma!a;ine are much more li%ely to be interested in buyin! or purchasin! a home
than the rest of the public.
)he number of the ma!a;ine5s readers has more than doubled o"er the past 2 years.
All sur"ey respondents recei"ed one free issue of the ma!a;ine as a to%en of appreciation for fillin! out the sur"ey.
21
18. (ost water companies in the United States add fluoride to tap water to help pre"ent ca"ities. Some dentists
ar!ue, howe"er, that this practice actually causes more harm than !ood because people o"erestimate the
protection afforded by the fluoride and do not ta%e the proper steps to care for their teeth, such as brushin!
and flossin! after e"ery meal. If water companies did not add fluoride, the dentists claim, people would be
forced to be more acti"e in their dental hy!iene and tooth decay would decline as a result. Which of the
following, if true, most weakens the dentists claims?
4luoride is widely used in commercially a"ailable dental care products.
(ost Americans are not aware that fluoride is added to tap water.
Annual "isits to the dentist are the most effecti"e means of controllin! tooth decay.
)he United States has the lowest rate of tooth decay in the world.
(ost Americans already brush their teeth daily.
,@. 2ecently in 3ity L, de"elopers ha"e stopped buyin! land, contractors ha"e found themsel"es !oin! without wor%
for lon!er periods, and ban%s ha"e issued fewer mort!a!es. )here must be fewer new residents mo"in! to 3ity L
than there were pre"iously. Which of the following indicates a flaw in the reasoning above?
)his year se"eral housin! bloc%s ha"e !one on the mar%et after bein! held up for months by le!al red tape.
)he a"era!e si;e of a new home has increased si!nificantly o"er the past se"eral years.
2esales of condominiums ha"e increased o"er the past si$ months.
)he cost of materials such as lumber and cement has decreased o"er the past year.
Sales of other bi!tic%et items, such as automobiles and boats, has remained steady o"er the past year.
20. En a"era!e, the number of speedin! tic%ets issued in 3ounty L e"ery year is three times !reater than the number
of speedin! tic%ets issued in 3ounty H durin! the same period. )herefore, the number of people who e$ceed the
speed limit must be hi!her in 3ounty L than in 3ounty H. Which of the following describes a flaw in the reasoning
above?
)he ar!ument fails to ta%e into account that the speed limit may be different in the two counties.
)he ar!ument fails to ta%e into account that the number of tic%ets issued in 3ounty L may reflect a lower
proportion of dri"ers o"erall in that county.
)he ar!ument fails to ta%e into account that a sin!le dri"er can recei"e more than one tic%et in a !i"en year.
)he ar!ument fails to ta%e into account that residents of 3ounty H may be more lawabidin! o"erall.
)he ar!ument fails to ta%e into account that residents of 3ounty L may not be aware of the speed limit in that
county.
2,. )he principal of School L has proposed a plan that would add an hourlon! study period to the end of the school
day. She claims that the e$tension would impro"e the school5s a"era!e score on the math section of the state
assessment by allowin! students more time to complete math homewor% that they wouldn5t otherwise ha"e time to
complete. Which of the following statements, if true, would most weaken the argument presented above?
(ath teachers of students in School L ha"e noted an o"erall decline in the number of homewor% assi!nments
completed by students since the school eliminated midday study periods two years a!o.
Administrators from nei!hborin! School H recently implemented a school day e$tension and ha"e seen no
si!nificant impro"ement in student test scores on the state assessment.
(usic department faculty members at School L stron!ly oppose the plan because they feel any time added to
the school day should be used to bolster the music curriculum.
:arents of students from School L ha"e e$pressed concern that the proposed schedule chan!e would
interfere with students5 e$tracurricular acti"ities.
)he core components of School L5s math curriculum are not ali!ned with the topics tested on the state math
assessment.
22. )he anticipated retirement of tens of thousands of baby boomers will create an unprecedented opportunity to mo"e
si!nificant numbers of people into careertrac% 7obs at familysupportin! incomes. (a7or industries, from health care
and construction to automoti"e repair, will soon face deep shorta!es of wor%ers as a result of pro7ected !rowth and
boomer retirements. 4ortunately, many of these 7obs ha"e relati"ely low barriers to entry and could be filled by out
ofwor% youn! people. )o achie"e this result, the city !o"ernment should con"ene employers and educators to
determine how best to create paths of upward mobility in these fields. Which of the following, if true,
most weakens the argument?
Immi!ration reform will limit the pool of a"ailable wor%ers.
>o"ernment efforts ha"e been shown to affect employment trends only rarely.
)he best a"ailable positions re<uire s%ills not possessed by the "ast ma7ority of the unemployed.
A small proportion of baby boomers will not retire as soon as is anticipated.
(any outofwor% youn! people are unaware of these loomin! employment opportunities.
22
20. Accordin! to a recent research study, more than @0' percent of !raduates of pri"ate hi!h schools in a certain
county continue their education in colle!e. By contrast, only +5' of !raduates of public hi!h schools
subse<uently pursue colle!e education. )herefore, if parents in the county wish to increase the li%elihood that
their children will attend colle!e, they should send them to pri"ate rather than public schools. Which of the
following statements would most seriously weaken the argument above?
>raduates of pri"ate schools typically score hi!her on standardi;ed tests and other tests of academic achie"ement.
Fhile pri"ate schools are typically "ery e$pensi"e, attendance of public school is free for the residents of the
county.
In comparison with !raduates of pri"ate schools, a substantially !reater proportion of public school !raduates
recei"e needbased financial aid for their colle!e education.
In comparison with pri"ate schools, public schools pro"ide more opportunities for student in"ol"ement in sports and
other athletic acti"ities, which almost always increase the li%elihood of students5 acceptance to colle!es.
Since most public schools are located in rural areas of the county populated primarily by farmers, nearly 00'
of students from public hi!h schools choose to pursue farmin! occupations rather than apply to colle!es.
21. )wo years a!o, the cost of the raw material used in a particular product doubled after an earth<ua%e
disrupted production in the re!ion where the material is mined. Since that time, the company that ma%es the
product has seen its profit mar!ins decline steadily. Aimin! to impro"e profit mar!ins, the company5s head of
en!ineerin! has decided that he must find a new source for the raw material. Which of the following, if true,
would cast the most doubt on the validity of the head of engineerings decision?
Aew competitors ha"e entered the mar%et e"ery si$ months for the past two years, resultin! in price wars that
ha"e pro!ressi"ely dri"en down re"enues across the mar%et.
Althou!h the earth<ua%e occurred two years a!o, the re!ion5s mines ha"e still not reco"ered to pre
earth<ua%e production capacity.
)here are se"eral other re!ions in the world where the raw material is mined, but those re!ions do not
produce as much of the raw material as the current source re!ion.
)he company could use a completely different raw material to ma%e its product.
2ecent ad"ances in minin! technolo!y will ma%e minin! the raw material much more efficient and cost
effecti"e in the future.
25. )he current administration and 3on!ress ha"e once a!ain practiced bad public policy in failin! to increase :ell
!rants or at least limit their reduction for ne$t year5s bud!et. :ell !rants impro"e access to hi!her education for
those who ha"e historically been disad"anta!ed in our society by financial or other life circumstances, thereby
helpin! recipients ele"ate themsel"es to the middle class. Fithout that access, the !ap between the rich and poor
in this country will continue to widen, increasin!ly strainin! the stability of our democracy. Which of the following,
if true, most seriously weakens the conclusion of this argument?
)otal spendin! on pro!rams tar!eted at impro"in! access to hi!her education for disad"anta!ed students will
increase in ne$t year5s federal bud!et.
)he neediest candidates for :ell !rants often lac% information about their eli!ibility for such !rants.
3on!ress recently authori;ed a bill that will increase afterschool pro!rams in urban communities.
En a"era!e, an indi"idual :ell !rant funds less than ,5' of the full cost of attendin! a fouryear colle!e or
uni"ersity.
4ederal spendin! on education for ne$t year will increase as a percenta!e of the total bud!et.
2+. In"estment Ad"isor* It is well%nown that in"estin! in mutual funds reduces portfolio ris% throu!h di"ersification. It is
also true that past in"estment performance is often related to future in"estment prospects. )herefore, to help my
clients earn hi!h returns with low ris%, I select a !roup of mutual funds that meet the client5s ob7ecti"es and then
in"est the client5s assets in the fund that has deli"ered the hi!hest returns in this !roup o"er the past 2 years.
Which of the following statements, if true, would demonstrate a serious flaw in the approach of the
Investment Advisor?
(ana!ers of many mutual funds that ha"e deli"ered the hi!hest returns o"er the past se"eral years ha"e
already used up their best in"estment ideas and are unli%ely to sustain this le"el of performance in the future.
(utual funds span a wide spectrum of in"estment styles and performance ob7ecti"es and no sin!le fund is
suitable for e"ery in"estor.
(any indi"idual in"estors choose to mana!e their own portfolios rather than consult an in"estment ad"isor.
)he funds that ha"e had the stron!est past performance tend to continue to outperform other funds with
similar ob7ecti"es for many years in the future.
)he number of clients ser"ed by the in"estment ad"isor has declined by nearly 50' o"er the past 5 years.
23
2.. Koe* )he si!n on this !arden says 86o Aot :ic% )he 4lowers.9
Sally* Hou are ri!ht. But there are a lot of flowers here. :ic%in! 7ust one will not hurt.
Koe* )hat is not true. If e"eryone thou!ht that way and pic%ed a flower, the !arden would be
destroyed. Koe5s response to Sally is <uestionable because JJJJJJ.
it i!nores the possibility that there may be circumstance where destroyin! the !arden is 7ustified
it is circular in reasonin!
it contradicts itself by pointin! out that collecti"e action has a different result than does an indi"idual action
it cites the conse<uence of e"eryone performin! an action rather than that of Sally5s action itself
it attac%s Sally5s character in order to undermine her credibility
2-. 3onsumer ad"ocates ar!ue that the coatin! found on nonstic% coo%ware contains harmful chemicals that are
released into the air when the coo%ware is heated abo"e a certain temperature. )he manufacturer of the coo%ware
ac%nowled!es this ha;ard but assures consumers that the temperature threshold is much hi!her than would e"er
be needed for food preparation and therefore no special precautions need be ta%en in usin! the coo%ware.
Which of the following, if true, would cast the most serious doubt on the claims of the manufacturer?
)he chemicals released by the coatin! can lin!er in the air for days
#mpty coo%ware left on the flame often reaches e$ceptionally hi!h temperatures.
Se"eral consumers ha"e already claimed illness as a result of usin! the coo%ware.
)he manufacturer did not test the coo%ware for this phenomenon until consumer ad"ocates brou!ht the issue
to its attention.
)here are effecti"e nonstic% coatin!s that do not release to$ins when heated.
2@. Unli%e 7u"enile diabetes, which is a !enetic condition present from birth, type2 diabetes is ac<uired in adulthood,
!enerally as a result of obesity and inacti"ity. )he number of cases of type2 diabetes has been steadily increasin!
in the United States since ,@.0, indicatin! to many researchers that the American population is becomin!
increasin!ly hea"y and sedentary. If the !o"ernment wishes to stem the spread of the disease, it should educate
the public about the dan!ers of an inacti"e, calorieladen lifestyle and promote healthful diets and e$ercise. Which
of the following, if true, provides the strongest reason to believe that the proposed education program will ./T be
effective?
School health pro!rams already educate middleschool students about the issue.
)he public already has access to this information throu!h the Internet.
4ood companies encoura!e the public to indul!e in unhealthful snac%s.
)he !o"ernment has not set aside money for such a pro!ram.
&ealthful foods and e$ercise pro!rams are beyond the financial means of many people.
30. Absenteeism amon! employees has decreased steadily o"er the past se"eral years. )wo possible e$planations
ha"e been offered. 4irst, impro"ed health insurance has allowed employees to maintain better health. Second,
impro"ed wor%in! conditions ha"e impro"ed morale, leadin! to less desire to a"oid wor%. &owe"er, since
absenteeism has also decreased at companies with poor or no health insurance and poor wor%in! conditions, one
must conclude that the cause of the decrease is the increased tendency of employers to deduct missed days from
employees5 paychec%s. Which of the following indicates a flaw in the reasoning above?
#mployees who lac% health insurance often ha"e low morale.
3ompanies with poor wor%in! conditions are most li%ely to deduct pay for missed wor% days
(ost employees are not honest about the reasons that they miss wor%
)he definition of absenteeism is not clear
Impro"ed health care, impro"ed wor%in! conditions, and fear of losin! money are not the only possible
e$planations for decreased absenteeism
24
0,. 2ecently, many critics of the U.S. !o"ernment ha"e pointed out that this country is the only industriali;ed
nation without a national "accine laboratory and su!!ested that this lac% ma%es the American public more
"ulnerable than other ad"anced nations to diseases such as a"ian flu or other flu epidemics. Fhen as%ed at a
press conference, a !o"ernment official said these critics were disloyal and thus wron! about the public5s
"ulnerability. )o support his claim, he cited the international preeminence of American doctors and hospitals
as well as the middle ran%in! of the United States amon! United Aation member nations in the health
cate!ories of infant mortality, life span, and nutrition. &e also added that all of the #uropeans that he %new
preferred to under!o ma7or medical treatments in the United States rather than in the sociali;ed medical
systems in place in their home countries. All of the following are flaws in the officials logic ()'(&T*
)he official accepts that the <uality of physicians and hospitals is a ma7or factor, albeit not the only one,
affectin! the public5s "ulnerability to disease.
)he critics could be disloyal but not wron!.
)he #uropeans that the official cited are o"erwhelmin!ly wealthy males o"er the a!e of fifty.
)he official relies on health statistics that are based on a complete accountin! of deaths, in7uries, and illnesses
suffered by the American public from all causes, includin! the ten percent attributable to infectious diseases.
)he UA health ran%in! that the official cited is based on an almost complete sur"ey of the nations of the world.
02. 6urin! the recent spate of brushfires in the Southwest, homeowners who li"ed near affected areas were ad"ised to
douse their roofs with water to pre"ent their houses from catchin! fire, before e"acuatin! the area. After the fires
were brou!ht under control and the homeowners were allowed to return to the area, many who doused their roofs
disco"ered si!nificant fire dama!e to their houses. 3learly, then, dousin! their roofs was a wasted effort.
Which of the following, if true, would most weaken the conclusion above?
)he houses of owners who did not douse the roofs with water suffered appreciably more fire dama!e than did
those of owners who did douse the roofs with water.
Aot all homeowners who doused their roofs did so to the same e$tent.
)he fire insurance rates for those who doused their roofs did not increase after the fire.
)he houses that suffered the least dama!e were those in which the owners remained and continuously
doused the roofs.
(ost of the homeowners who doused their roofs had been throu!h a brushfire e"acuation before.
33. 4or years, the debate o"er public education reform has centered on financin!. (any claim that pourin! more
money into the public schools will impro"e student performance. &owe"er, the only way to fi$ our school
systems is to in7ect new ideas and new approaches. )oday the schools are or!ani;ed to benefit their adult
employees rather than the students. Which of the following, if true, best weakens the argument?
Schools that ha"e instituted 8new approaches9 attract the best performin! students.
Schools without outside play!rounds ha"e lower le"els of student performance than schools that do.
Studies show that student performance corresponded most directly with the education of the students5 families.
School employees, by an o"erwhelmin! mar!in, said that the system performed well, citin! superior benefits
than those a"ailable in comparable pri"ate institutions.
2esearchers in education ha"e shown that students from school districts with hi!h percapita spendin! tend
to recei"e hi!her scores on standardi;ed tests.
34. 3ompany L conducted a taste test to determine whether its new soft drin% had a !ood chance of commercial
success. A sample of consumers was as%ed to compare the fla"or of the new soft drin% to that of an established
brand without %nowin! the true identity of either be"era!e. E"erwhelmin!ly, the consumers preferred the taste of
the proposed soft drin% to that of the established brand. 3learly, 3ompany L has a !ood chance of commercial
success with its new soft drin%. Which of the following, if true, would most seriously weaken the argument
above?
Some of the consumers in the taste test preferred the fla"or of the established brand.
)he other soft drin% used in the taste test is also manufactured by 3ompany L.
)he new soft drin% will cost more than three times as much as any other soft drin% on the mar%et.
3ompany L has not yet desi!ned a label for the new soft drin%.
)he name of the new soft drin% is "ery close to that of the established brand.
25
35. (ath education in this country does a disser"ice to our children. In the lower !rades, it should focus on the
basic s%ills that students will need in hi!her !rades to de"elop the ability to sol"e comple$ problems. =earnin!
basic math s%ills is li%e learnin! the scales and chords that one will later use to master complicated concertos
and symphonies. &owe"er, math educators in this country seem to ha"e it bac%ward, emphasi;in! in hi!her
!rades the same narrow, s%ills based approach that students learned in lower !rades rather than the
analytical tools they will need to sol"e comple$ math problems. Which o, the ,ollo+i!0 i, true0 +oul* )ost
seriousl1 +ea2e the coclusio *ra+ abo3e-
Fhile music is common in elementary school curriculums, it is rarely tau!ht in hi!h school.
En international tests of math s%ills, hi!hschool students in this country performed no worse than did their
counterparts from countries where problemsol"in! is emphasi;ed in hi!her !rades.
Fhen presented with a math problem to sol"e, students in hi!her !rades are more li%ely to arri"e at different
answers than students in lowers !rades are.
Elder students tend to recei"e hi!her !rades in math than do youn!er students.
Uni"ersities in this country report a steady increase in the percenta!e of nati"e firstyear students who <ualify
to ta%e ad"anced mathematics courses such as calculus.
0+. )he recent boom in new home construction has finally be!un to taper off. 6e"elopers are not buyin! land,
contractors are findin! themsel"es !oin! without wor% for lon!er periods, and ban%s are issuin! fewer
mort!a!es. :eople must not be as interested in buyin! new homes as they were e"en si$ months a!o.
Which of the following indicates a flaw in the reasoning above?
Interest rates for home mort!a!es ha"e increased si!nificantly o"er the past si$ months.
)he a"era!e si;e of a new home has increased si!nificantly o"er the past se"eral years.
Sales of condominiums ha"e increased o"er the past si$ months.
)he cost of materials such as lumber and cement has decreased o"er the past year.
Sales of other bi!tic%et items, such as automobiles and boats, has remained steady o"er the past year.
0.. 2esearchers ha"e noticed that people whose blood shows abnormally low le"els of calcium usually ha"e
laryn!eal polyps, which can permanently dama!e "ocal cords and result in partial or e"en total loss of "oice.
In order to pre"ent the polyps, the researchers recommend a diet hi!h in calciumrich foods such as dairy and
!reen, leafy "e!etables. Which of the following, if true, most strongly suggests that it would not be
advisable to follow the researchers recommendation?
6airy contains compounds that are difficult for many people to di!est.
=aryn!eal polyps sometimes disappear without treatment.
=aryn!eal polyps cause a chan!e in body chemistry that bloc%s the absorption of calcium.
4resh "e!etables are not always a"ailable in all seasons.
=ow le"els of calcium can sometime be remedied with "itamin pills.
Critical Reasoi! To"ic 4$ STRE%5T6E%
1. )he United States !o"ernment uses only a household5s cash income before ta$es to determine whether that
household falls below the po"erty line in a !i"en yearI capital !ains, noncash !o"ernment benefits, and ta$ credits
are not included. &owe"er, yearly cash income is not a foolproof measure of a !i"en household5s disposable
income. 4or e$ample, retirees who li"e off of capital !ains from an e$tensi"e portfolio could earn hundreds of
thousands of dollars, yet be classified by the !o"ernment as li"in! in 8po"erty9 because this income is not included
in the calculation. Which of the following, if true, validates the contention that the governments calculation methods
must be altered in order to provide statistics that measure true poverty?
4or more than @@' of those classified as li"in! in po"erty, yearly cash income comprises the "ast ma7ority of
each household5s disposable income.
Fhile the !o"ernment5s calculation method indicated a ,2.5' po"erty rate in 2000, the same calculation
method indicated anywhere from a @' to a ,+' po"erty rate durin! the precedin! decade.
(ost established research studies conducted by the pri"ate sector indicate that the number of people truly
li"in! in po"erty in the U.S. is less than that indicated by the !o"ernment5s calculation method.
Se"eral prominent economists endorse an alternate calculation method which incorporates all income, not
7ust cash income, and ad7usts for ta$es paid and other core e$penses.
)he !o"ernment5s calculation method also erroneously counts those who do not earn income in a !i"en year
but who ha"e substantial assets on which to li"e durin! that year.
26
2. &istorically, the dru! industry promoted its products to physicians by educatin! them in their offices or at
industry conferences. In the last ,0 years, it has become much more commonplace for dru! companies to
ad"ertise prescription dru!s directly to consumers, "ia tele"ision ad"ertisin! and other media. Some public
health ad"ocates ha"e become concerned that patients, encoura!ed by ad"ertisin!, may pursue the use of
prescription dru!s that may be inappropriate for the indi"idual patient or situation. &owe"er, since physicians
must prescribe these medications, there is no reason for such concern. Which of the following pieces of
information would be most helpful in addressing the concern articulated by the public health advocates?
3ertain o"erthecounter medications are as effecti"e for many common medical conditions as more powerful
nonprescription medications.
:rescription medication tele"ision ad"ertisements directed at the !eneral public only appear on certain
pro!rams and are not seen by many potential consumers.
:hysicians are also sub7ect to prescription dru! ad"ertisements that are directed toward consumers.
:hysicians are not susceptible to pressure from patients in determinin! appropriate courses of treatment,
includin! dru! prescriptions.
:rescription medicines ha"e been pro"en to be safe and effecti"e treatments for many patient conditions.
3. Studies in restaurants show that the tips left by customers who pay their bill tend to be lar!er when the bill is
presented with the ser"er5s name handwritten on the bill. :sycholo!ists hypothesi;e that simply seein! a hand
written name ma%es many consumers feel more of a personal identification with the ser"er, encoura!in! lar!er tips.
Which of the following, if true, most strongly supports the psychologists interpretation of the studies?
)he effect noted in the studies applies to patrons payin! with either credit cards or cash.
Aameta!s for ser"ers ha"e not been shown to ha"e any effect on the si;e of the bill.
>reetin! card companies ha"e found that charities which send holiday cards with handwritten si!natures are
more li%ely to recei"e donations than those which send cards with printed si!natures.
)he studies indicated much lar!er a"era!e tips if the customer ordered alcoholic be"era!es with his or her meal.
(any of the restaurants in which the studies were conducted are located in tourist areas, where people are
tra"elin! for leisure acti"ities.
1. Analyst* )he pace of technolo!ical de"elopment brin!s a constant stream of new de"ices to the mar%et, and many
of them en7oy commercial success. But announcin! new technolo!y too soon after the introduction of a successful
de"ice can bac%fire. Ence consumers hear about the new de"ice, they may stop buyin! the one currently on sale.
So, if a company wishes to announce the upcomin! sale of a new de"ice, it should wait until purchases of the old
de"ice ha"e be!un to decline. Which of the following, if true, would best support the analysts main assertion?
Aew technolo!y often becomes less e$pensi"e after an initial sur!e in sales.
(edia outlets, such as tele"ision pro!rams and ma!a;ines, often report on the planned introduction of new
de"ices while the sales of old de"ices are still stron!.
(any consumers are unable to determine whether new technolo!y is superior to current technolo!y.
Sur"eys ha"e shown that some consumers ma%e only one or two technolo!y purchases per year, whereas
others ma%e more fre<uent purchases.
3onsumers tend to be loyal to technolo!y companies whose products they en7oy usin!.
5. (ore and more companies ha"e be!un to consume less ener!y by ma%in! themsel"es more efficient. E"er time,
these efforts could place the United States at the forefront of an emer!in! !lobal mar%et for cleaner technolo!ies.
Such efforts are also essential to tac%lin! the two bi! ener!yrelated issues of the a!e* !lobal warmin! and the
dependence on precarious supplies of oil. )he federal !o"ernment should encoura!e these efforts by pro"idin! the
necessary incenti"es, whether as loans, direct !rants or tar!eted ta$ brea%s. Which of the following, if true,
provides the most effective support for the argument?
En the a"era!e, 3anadian companies are more ener!y efficient than those in the United States.
#$perts belie"e that ener!y efficiency could lower the ener!y use of the United States to the le"el of ,@@5.
In the past, !o"ernment incenti"es ha"e made ad"ances in ener!y conser"ation feasible, especially in the
auto industry.
)he dependence on forei!n oil is a !reater problem in the present than !lobal warmin!.
)he mar%et for cleaner technolo!ies is currently relati"ely small because of the infrastructure re<uirements.
27
+. State L recently decided to cut state fundin! for the public library system in 3ounty L. )o help counteract this
cut in fundin!, the county library system has increased library late fees from $.,0 per day to $.,2 per day.
Since the fee increase, library administrators ha"e seen no decline in the number of o"erdue boo%s. )he
director of the county library system concludes that the fee hi%e has helped to counteract the cut in state
fundin!. Which of the following statements, if true, most strengthens the directors claim?
Since the fee increase, library administrators ha"e noted a si!nificant decrease in the number of boo%s
borrowed each day.
)he library system incurred minor costs to ma%e its "isitors aware of the late fee increase.
Since the fee increase, there has been no si!nificant chan!e in the a"era!e number of days that boo%s are
o"erdue before they are returned.
)he library system in 3ounty L trac%s its boo%s throu!h a "ery ad"anced database system, allowin! library
administrators to ha"e access to "ery accurate statistics on the number of o"erdue boo%s at any !i"en time.
Since the reduction in state fundin!, the library system in 3ounty L has eliminated ,0' of its staff, creatin! a
2' reduction in costs.
.. In the ,-th and ,@th centuries, it was belie"ed in many coastal American cities that the waterfront was an undesirable
location for residential buildin!s. As a result, much of the waterfront in these cities was ne"er de"eloped aesthetically and
instead was left to industry and commerce. )oday, howe"er, waterfront properties are !enerally seen as presti!ious, as
e"idenced by the lar!e sums paid for homes alon! the beach front. A de"eloper who wishes to ma%e a lar!e profit would
be wise to buy urban waterfront lots and erect residential buildin!s on them.
Which of the following, if true, most strongly supports the claim made about urban waterfront properties?
:eople today ha"e more money, relati"ely spea%in!, to spend on real estate than they did in pre"ious centuries.
&omeowners will be willin! to spend lar!e sums on residential properties in traditionally industrial or
commercial districts.
(any urban waterfront lots are a"ailable for purchase.
(any coastal American cities are encoura!in! de"elopers to rehabilitate the waterfront throu!h ta$ incenti"es.
:roperties in interior residential districts in coastal American cities are si!nificantly more e$pensi"e than those
alon! the waterfront.
-. 3harter schools are independent public schools that are !i"en !reater autonomy in e$chan!e for increased
accountability. 3harter school operators are freed from many of the re!ulations of the traditional public school
bureaucracy, thereby allowin! them to pursue more inno"ati"e educational ideas than noncharter public schools
can pursue. At the same time, charter schools are held accountable for achie"in! specific educational outcomes
and are closed down if those outcomes are not met. Which of the following, if true, best supports the assertion that
students attending charter schools will, on average, perform better on assessments of writing ability than
students attending traditional public schools?
Students who attend schools that emphasi;e order and discipline perform worse on assessments of writin!
ability than students who attend schools that do not emphasi;e order and discipline.
)he ma7ority of students who score in the @@th percentile on assessments of writin! ability attend charter schools.
:ublic schools that operate outside of the traditional public school bureaucracy spend more time teachin!
students writin! than do traditional public schools.
Students who attend schools that are allowed to e$periment with their writin! curricula perform better on
assessments of writin! ability than students who attend schools that ha"e less fle$ible curricula.
)here are far more students attendin! noncharter public schools than students attendin! charter schools.
@. Efficials of the Houth &oc%ey =ea!ue and parents of players in the lea!ue ha"e become concerned with the
number of fla!rant fouls occurrin! durin! lea!ue !ames. )his past season, the number of fla!rant fouls was double
the number from the season before. =ea!ue officials plan to reduce the number of such fouls durin! the comin!
season by implementin! mandatory suspensions for players who commit fla!rant fouls. Which of the
following statements, if true, provides the best evidence that the officials plan will be effective?
(ost serious in7uries occurrin! durin! lea!ue !ames are a direct result of fla!rant fouls.
=ea!ue referees ha"e been trained to reco!ni;e fla!rant fouls and to report incidents in"ol"in! such fouls.
:arents of players in the lea!ue are in support of mandatory suspensions for fla!rant fouls.
A similar lea!ue suspends players for committin! fla!rant foulsI this lea!ue has a relati"ely low incidence of
fla!rant fouls when compared with the Houth &oc%ey =ea!ue.
(ost players in the lea!ue stri"e to be selected for the AllStar team, and lea!ue rules state that no player
with a record of suspension shall be selected for the AllStar team.
28
10. Almost e"ery modern %itchen today is e<uipped with a microwa"e o"en, mainly because microwa"e o"ens
offer a fast and con"enient way of coo%in! and reheatin! food. Indeed, it has become a standard appliance in
most households. Studies ha"e shown, howe"er, that microwa"e o"ens are not completely safe and their use
has occasionally resulted in serious in7ury. Because of this, some consumer ad"ocates ar!ue that microwa"e
o"ens should not be so readily accepted as a standard appliance until they can be certified to be completely
safe. Which of the following, if true, would most strengthen the argument of the consumer advocates?
(icrowa"e o"ens ha"e ta%en much of the 7oy out of coo%in!.
)here ha"e been many reported incidences of people who ha"e been scalded by li<uids superheated in
microwa"e o"ens.
Absolute safety is the only criterion by which an appliance should be 7ud!ed to be acceptable as 8standard.9
)here is no such thin! as a completely safe appliance.
Sto"es and o"ens that use natural !as consume ener!y much more efficiently than microwa"e o"ens.
00. Ene feature of the !lobal food economy is the simultaneous import and e$port of the same items, a
phenomenon %nown as 8redundant trade.9 In 3alifornia, for e$ample, domestic cherries are e$ported to
3anada and Kapan, while a nearly e<ui"alent number of cherries are imported from 3hile, Italy, and >ermany.
3alifornia also e$ports and imports nearly identical amounts of lettuce and almonds. Althou!h shippin! fresh
fruits and "e!etables is an e$pensi"e underta%in!, there is a 7ustifiable economic rationale for redundant
trade. Which of the following, if true, most strongly supports the conclusion above?
#stablishin! international ties throu!h trade facilitates access to other desired !oods that are more efficiently
produced abroad.
Underta%in! free trade with one5s political allies helps to maintain international !oodwill.
In recent years, consumers in 3alifornia boycotted domestic cherries, demandin! better wor%in! conditions
for a!ricultural laborers in the state.
=ocal !rowers could sell their products at a premium in domestic mar%ets.
)he economic !lobali;ation of redundant trade allows for the sharin! of cultural norms and "alues.
12. Fide dissemination of wireless networ%s in cities is a practical way to meet the needs of city households, schools
and businesses. 2ural communities ha"e found that wireless networ%s are both more reliable and cheaper than
landbased networ%s. Which of the following would most likely be cited by a supporter of the argument?
Urban areas do not pose additional problems for the effecti"e operation of wireless networ%s.
Fireless networ%s wor% far better where population density is low.
Iceland, a "ery rural country, successfully uses wireless networ%s.
)he e$penses of wireless transmission in areas with lar!e buildin!s is much hi!her.
:oor nei!hborhoods ha"e less access to cable internet than do educators or businesses.
,0. Sur"eys re"eal that the "ast ma7ority of hotel !uests in the United States resent the hi!h prices of the items in the
minibars in their hotel rooms.)hese !uests would prefer to ha"e an empty refri!erator in their rooms in order to
ha"e space to put their own food and be"era!es, althou!h a lar!e percenta!e of these !uests would still ma%e at
least one purchase from their inroom minibar. After analy;in! the results of the study, the mana!ement of &otel )
decided that it would be more profitable to eliminate the minibar and install empty refri!erators in each room.
Which of the following, if true, would support 0otel Ts plan to increase profitability by eliminating in room mini1
bars in favor of empty refrigerators?
)here is currently some space a"ailable in &otel )5s inroom minibars for !uests to put their own items.
&otel ) is located in the United States.
Some !uests of &otel ) do not ma%e any purchases from their inroom minibars.
)he money that &otel ) ma%es from the minibar purchases of its !uests is less than the money that &otel )
loses from discardin! minibar items that ha"e not sold by their e$piration dates.
It will cost &otel ) less money to maintain empty refri!erators in its !uest rooms than to maintain stoc%ed
minibars in those rooms.
29
,1. )here is only one ma7or road, 4reeway P, that lin%s 3ounty L and 3ounty H. )he border of the two counties is
primarily defined by a mountain ran!e, o"er which the construction of new roads is se"erely restricted by
en"ironmental laws. A costeffecti"e solution to the problem of traffic con!estion on 4reeway P is to build a
commuter train tunnel throu!h the mountain ran!e. )he successful implementation of this plan would cost far less
than e$pandin! the e$istin! freeway and would also reduce the number of cars clo!!in! the roads in both counties.
Which of the following, if true, could proponents of the plan above most appropriately cite as a piece of
evidence for the soundness of their plan?
An effecti"e commuter train tunnel between the counties would re<uire ma7or in"estment in mass transit
within both counties.
)he ma7ority of all "ehicles on the nation5s freeways are tra"elin! from one state to another.
+0' of the cars on 4reeway P are dri"en by people who li"e in 3ounty H and wor% in 3ounty L.
(any new freeways are bein! built in areas that are presently ser"ed by commuter trains.
A lar!e proportion of the "ehicles on 4reeway P are commercial truc%s carryin! transcontinental shipments.
15. In Kanuary of last year, 4astfood Min! started usin! a new lowfat oil to coo% its 4ast 4ries, instead of the less
healthful corn oil that it had been usin!. Aow 4astfood Min! is plannin! to switch bac%, sayin! that the chan!e
has hurt sales of 4ast 4ries. &owe"er, this claim is incorrect, since accordin! to 4astfood Min!5s own sales
fi!ures, 4astfood Min! sold ,0 percent more 4ast 4ries last year than in the pre"ious year. Which of the
following, if true, most strongly supports the argument against ,astfood 2ings claim?
)otal sales of all foods at 4astfood Min!5s locations increased by less than ,0 percent last year.
4astfood Min! en7oys hi!her profit mar!ins on its Soft 6rin%s than it does on 4ast 4ries.
4astfood Min!5s customers prefer the taste of 4ast 4ries coo%ed in corn oil to 4ast 4ries coo%ed in lowfat oil.
)he number of customers that "isited 4astfood Min! locations was more than 20 percent hi!her last year than
the year before.
)he year before last, 4astfood Min! e$perienced a 20 percent increase in 4ast 4ries sales o"er the pre"ious year.
16. )he Fest Indian manatee, a distant relati"e of the elephant, returned to the sea some 50 million years a!o.
)hese thousandpound herbi"ores inhabit the warm coastal waters where Americans li%e to play. 6espite
conser"ation efforts, criminal penalties for harmin! these creatures, and an o"erabundance of SAO# )&#
(AAA)##N bumper stic%ers, none of these animals can be considered safe. Which of the following, if true,
most strongly supports the argument above?
=ast year, se"eral manatees were mysteriously %illed by an unidentified to$in.
All manatees swim at depths than ma%e them "ulnerable to the blades of motorboat en!ines.
(ost tourists are unaware of the on!oin! efforts to sa"e the manatee.
)he population of manatees in the wild has dwindled to fewer than 2,500 animals.
Althou!h do;ens of manatee deaths are documented each year, many more deaths !o unreported.
,.. Bettin! on sports, e"en for small sta%es amon! friends, is a form of !amblin!. )herefore, no police officer
should e"er bet on sports. Which of the following, if true, most strengthens the conclusion that no police
officer should ever bet on sports?
>amblin! is ille!al in many states and countries.
Some people who bet on sports are con"icted criminals.
:eople who bet on sports sometimes can ill afford to lose the amounts they wa!er.
Ao police officer should e"er !amble.
(any philosophers consider !amblin! to be immoral.
,-. #thanol is a deri"ati"e of corn and other !rains. Fhen burned as fuel, it emits si!nificantly lower le"els of carbon
mono$ide, a ma7or atmospheric pollutant, than does !asoline. 4or that reason, en"ironmentalists claim that ethanol
is a better source of ener!y than !asoline. Which of the following, if true, most strongly supports the environmentalists
claim?
Fhen burned as fuel, ethanol does not release any pollutants at hi!her le"els than does !asoline.
#thanol is comparable in price to !asoline.
A"ailable supplies of corn are sufficient to supply se"eral years5 worth of ethanol.
(ost !asoline companies already possess the technolo!y to produce ethanol.
#thanol can be used as heatin! fuel.
30
19. (edical education in the United States has focused almost e$clusi"ely on curati"e medicine, while pre"enti"e
care has been !i"en scant attention. )his is mis!uided. (edical schools should in"est as much time in
teachin! their students how to pre"ent illness as in teachin! them how to cure it. Which of the following, if
true, most strengthens the argument above?
(any conta!ious diseases can be pre"ented with "accines.
In ,@--, for e"ery three cents the United States spent on pre"ention, it spent @. cents on curati"e treatment.
)he number of students enrolled in medical school is the hi!hest it has e"er been.
(ore people die each year from disease than from accidental causes.
As the population !rows, the number of doctors in certain specialties has not been %eepin! pace.
20. Some animals, such as dolphins, do!s, and African !rey parrots, seem to e$hibit co!niti"e functions typically
associated with hi!herorder primates such as chimpan;ees, !orillas, and humans. Some parrots, for e$ample,
ha"e "ocabularies of hundreds of words that they can strin! to!ether in a comprehensible synta$. )his clearly
shows that humans and primates are not the only animals capable of usin! lan!ua!e to communicate. Ene parrot,
named Ale$, has been %nown to as% to be petted or %issed and will e$hibit a!!ression if the !esture offered is not
the specific one re<uested. Which of the following, if true, would most strengthen the conclusion above?
6olphins can be trained to assist di"ers in ocean rescues.
>orillas in capti"ity often learn hand si!nals for food and water.
6o!s are capable of sensin! their owners5 moods and often e$hibit concern if they sense sadness.
3himpan;ees can memori;e lon! se<uences of %ey punches on machines that dispense food.
Ale$ does not e$hibit a!!ression when offered a !esture that he specifically re<uested.
2,. )o pre"ent o"ercrowdin!, last month the town ;onin! board limited the number of new buildin!s that can be
constructed in the town in any !i"en year. )he board claims that doin! so will preser"e open spaces and
lessen the strain on municipal resources such as schools and !arba!e disposal. 3ritics of the chan!es ar!ue
that the plan will harm the community or, at the "ery least, will fail in its purpose. Which o, the ,ollo+i!
)ost su""orts the clai)s o, the critics o, the "la-
Q Ether towns ha"e had mi$ed success with similar ;onin! plans.
Q Ao new schools ha"e been built in the town in ten years.
Q :roperty ta$es in the town are hi!her than in nei!hborin! towns.
Q Under the new plan, de"elopers may still erect apartment buildin!s.
Q )he nearest !arba!e dump is se"eral miles away from the town.
22. At any !i"en time, appro$imately fifteen percent of all homes in 4lorida are on the mar%et. In )e$as, howe"er, only
se"en percent of all homes are on the mar%et at any !i"en time. )herefore, one will ha"e a wider selection of
homes to choose from if one loo%s for a home in 4lorida rather than in )e$as. Which of the following, if true,
would most seriously strengthen the argument above?
&omes in 4lorida tend to be less e$pensi"e than those in )e$as.
(ort!a!es are easier to obtain for homes in 4lorida than for homes in )e$as.
)he construction industry in )e$as has reported si!nificant !rowth o"er the past year.
)he cost of constructin! new homes in )e$as is hi!her than in 4lorida.
)he total number of homes in 4lorida is three times !reater than the total number in )e$as.
Critical Reasoi! To"ic 7$ E3aluate
,. A certain pharmaceutical firm claims that its dietary supplement, 6ietol, is hi!hly effecti"e in helpin! obese people
lose wei!ht and impro"e physical endurance. )he company supports this claim by pro"idin! the results of a recent
e$periment that in"ol"ed o"er -00 sub7ects sufferin! from obesity. 6urin! the @wee% e$periment, each of the
sub7ects was re<uired to ta%e a re!ular dose of 6ietol e"ery day immediately after their daily 2hour wor%outs
super"ised by a professional fitness instructor. As a result of the e$periment, the sub7ects lost an a"era!e of ,2
pounds of wei!ht per person, and o"er @5' of all participants demonstrated hi!her physical endurance. The
answer to which of the following 3uestions would be most helpful in evaluating the conclusion of the
pharmaceutical
company?
Fhat was the daily dose of 6ietol that the sub7ects were re<uired to ta%e?
Fhat was the ma$imum wei!ht lost by any participant durin! the @wee% pro!ram?
Fhat would be the a"era!e wei!ht loss and the impro"ement in endurance in a !roup of sub7ects with similar
characteristics in"ol"ed in the same physical fitness pro!ram but not ta%in! 6ietol?
Fhat was the a"era!e a!e amon! the sub7ects participatin! in the e$periment?
6id the ma7ority of sub7ects e$perience a si!nificant impro"ement in physical stren!th?
31
2. )he recordin! industry is fi!htin! a losin! battle* it simply does not ha"e the resources to prosecute all of the
indi"iduals who ille!ally download music from the Internet. Because the number of indi"iduals who will be
char!ed with a crime is so limited, the actions of the recordin! industry will ha"e a minimal impact on the
number of people who ille!ally download music. The answer to which of the following 3uestions would best
help evaluate the accuracy of the conclusion above?
Fill recordin! industry lawyers dedicate the ma7ority of their time to prosecutin! those who ille!ally download
music?
Is a small minority of indi"iduals responsible for the ma7ority of ille!al son! downloads?
6o many indi"iduals who ille!ally download son!s share their music files with other Internet users?
Fill new Internet security technolo!y permit the recordin! industry to more <uic%ly and easily identify
indi"iduals who ille!ally download music?
Fill the threat of prosecution alter the beha"ior of those who ille!ally download music?
0. Fhile political discourse and the media in the United States ha"e focused on the rise of 7ob outsourcin!, few ha"e
mentioned the sharp fall of talent 8insourcin!,9 or the drop in enrollment of forei!nborn !raduate students since
200,, and its dire results. )he decrease in such insourcin! will hurt America5s competiti"eness in basic research
and applied technolo!y, with serious conse<uences for years to come. )he deinternationali;ation of !raduate
pro!rams across the country will also ne!ati"ely affect the !lobal outloo% and e$perience of the American students
remainin! in those pro!ramsI they will not ha"e the opportunity to learn about forei!n cultures directly from
members of those cultures. Fhat distin!uishes the decline of talent insourcin! from the rise of 7ob outsourcin! is
that the former can be easily rectified by a policy chan!e of the United States !o"ernment. The answer to which of
the following 3uestions would be most useful in evaluating the authors claim regarding the impact of decreased
insourcing in America?
Fhat is the cost to re"erse the trend of insourcin! in America?
&ow does insourcin! replace domestic 7obs lost from outsourcin!?
Since 200,, what has been the decrease in the number of forei!nborn students in America?
Fhat opportunities do American !raduate students ha"e to interact re!ularly with forei!ners who are not students?
Fhat effect would a !o"ernment policy ha"e on the number of forei!n !raduate students?
1. :rofits for one of 3ompany L5s fla!ship products ha"e been declinin! slowly for se"eral years. )he 34E in"esti!ated and
determined that inflation has raised the cost of producin! the product but consumers who were sur"eyed
reported that they weren5t willin! to pay more than the current price. As a result, the 34E recommended that the
company stop producin! this product because the 3#E only wants products whose profit mar!ins are increasin!.
The answer to which of the following 3uestions would be most useful in evaluating whether the ',/s
decision to divest the company of its flagship product is warranted?
6oes the company ha"e new and profitable products a"ailable with which to replace the fla!ship product?
Fill the rest of 3ompany L5s mana!ement team a!ree with the 34E5s recommendation?
Are there additional features which could be added to the product and for which consumers mi!ht be willin! to
pay a hi!her price?
Is there a way to alter the manufacturin! or distribution processes in order to reduce the cost to produce the
fla!ship product?
Fhat percenta!e of 3ompany L5s re"enues is represented by sales of the fla!ship product in <uestion?
5. )he 3i"il Ser"ice Act of ,--0, also %nown as the :endleton Act, which created a professional corps of administrators, was
passed after a disappointed officesee%er assassinated :resident Kames A. >arfield. 4or a hundred years, this system
has anchored American !o"ernment ser"ice to competence rather than corruption. )he best way to preser"e this state in
the new millennium is to maintain the :endleton Act as it is. Which of the following would be
most useful to evaluate the arguments conclusion?
)he methods that the Swiss and British !o"ernments ha"e used to pre"ent corruption in !o"ernment ser"ice
for the past one hundred years
)he current le"el of 7ob satisfaction amon! !o"ernment officesee%ers and officeholders
)he le"els of competence and corruption in American !o"ernment ser"ice between ,@50 and the present
)he number of :residents assassinated since the passa!e of the :endleton Act
)he percenta!e of officeholders fired or con"icted on char!es stemmin! from corruption durin! the first
hundred years of the :endleton Act
32
6. Poolo!ists warn of an imminent sur!e in the number of bird species that will become e$tinct within this
century. Ae"ertheless, these ;oolo!ists are wron!. Ene need only consider the information !athered on
nati"e Aorth American raptors, such as bald ea!les and pere!rine falcons. Se"eral of these species came
close to "anishin! between ,@00 and ,@.0, but since ,@.0, the local populations of these raptors ha"e
rebounded. The answer to which of the following 3uestions provides information that would be most helpful in
evaluating the argument above?
&ow many species of nonnati"e raptors ha"e been introduced into Aorth America since ,@.0?
Fhat special efforts, if any, ha"e been made to rescue nati"e Aorth American raptors since ,@.0?
&ow many years5 e$perience do the ;oolo!ists ha"e in e"aluatin! patterns of e$tinction amon! animals?
)o what de!ree ha"e nati"e Aorth American raptors mi!rated to other parts of the world?
&ow many acres of woodland are set aside each year as bird refu!es?
.. (ost cable tele"ision companies currently re<uire customers to subscribe to pac%a!es of channels, but consumer
!roups ha"e recently proposed le!islation that would force the companies to offer a la carte pricin!.
Subscribers would pay less, ar!ue the consumer !roups, because they could purchase only the desired
channels. &owe"er, the cable industry ar!ues that under the current pac%a!e pricin!, popular channels
subsidi;e lesspopular ones, pro"idin! more options for "iewers. 4or this reason, the industry claims that it is
always cheaper for the consumer to purchase many bundled channels than to buy them indi"idually. Which of
the following would be most important for the government to determine before deciding whether to re3uire
cable television companies to offer a la carte pricing in order to reduce consumer costs?
Fhether the total number of channels offered to consumers would decrease, alon! with pro!rammin!
di"ersity, as a result of the a la carte pricin! structure
Fhether ad"ertisin! re"enue for the cable tele"ision companies would decrease as a result of the a la carte
pricin! structure
Fhether the "ast ma7ority of consumers would !reatly reduce the number of channels purchased if !i"en the
option of purchasin! them indi"idually
Fhether cable and satellite companies currently ha"e the ability to buy channels indi"idually from
pro!rammers and content pro"iders
Fhether a la carte subscribers would be re<uired to ha"e new tele"ision settop bo$es
8. )he director of pro!rammin! at AAA, the Aational Aews Aetwor%, proclaimed that, despite char!es to the contrary,
the Aetwor% does not ha"e a conser"ati"e bias. )he director ac%nowled!ed that, when a liberal news personality
was recently added to the Aetwor%, an additional conser"ati"e news pro!ram was also established. &owe"er, the
director ar!ued that far from demonstratin! any bias, these actions reflect a commitment to presentin! a balanced
perspecti"e in interpretin! current e"ents. Which of the following pieces of information would be most useful in
evaluating the validity of the programming directors argument?
)he Aetwor% has established an additional liberal news pro!ram whene"er an additional conser"ati"e news
personality has been added.
)he Aetwor% airs a pro!ram focused lar!ely on the opinions of its "iewers on contemporary issues.
)he newly added liberal news personality came from a ri"al networ% with an ac%nowled!ed liberal bias.
)he newly added liberal news personality has indicated that he has not felt any editorial pressure in his new
en"ironment.
)he "iewer ratin!s of the Aetwor% ha"e been lower this past year than in pre"ious years.
9. 3hild de"elopment specialists ha"e obser"ed that adolescents who recei"e lar!e wee%ly allowances tend to spend
money on items considered fri"olous by their parents whereas adolescents who recei"e small wee%ly allowances
do not. )hus, in order to ensure that their children do not spend money on fri"olous items, parents should not !i"e
their children lar!e wee%ly allowances. Which of the following pieces of information would be most useful in
evaluating the validity of the conclusion above?
)he a"era!e amount of money recei"ed by adolescents who recei"e lar!e wee%ly allowances
Any differences amon! parents in the standard used to 7ud!e an item as fri"olous
)he educational bac%!round of the child de"elopment specialists who made this obser"ation
)he difference between the a"era!e annual income of families in which the parents !i"e their children lar!e
wee%ly allowances and that of families in which the parents !i"e their children small wee%ly allowances
)he percenta!e of adolescents who recei"e no wee%ly allowance
33
10. )he downturn in the economy last year has prompted many companies to ma%e widely publici;ed layoffs,
resultin! in thousands of lost 7obs. #conomists predicted that these layoffs would cause people !enerally to
cut bac% on their discretionary spendin! e"en if their 7obs were secure, in anticipation of comin! hard times.
&owe"er, this prediction has not come to pass, since there has been no increase in the amount of money set
aside by the !eneral public in sa"in!s accounts. The answer to which of the following 3uestions would be
most useful in evaluating the significance of the savings patterns described above?
Fhat business sectors were most affected by the layoffs?
&ow much of their sa"in!s, on a"era!e, do laidoff employees deplete before findin! new employment?
Fhat has been the percent increase in the cost of necessities such as food, housin!, and utilities durin! the
period since the layoffs?
Fhat percenta!e of people laid off ha"e sa"in!s accounts?
Fhat has been the a"era!e salary durin! the period since the layoffs?
,,. Because of a rare type of fun!us that %illed off many cacao trees in Bra;il, there was an unusually mea!er
har"est of cocoa beans this year. )he wholesale price of cocoa solids and cocoa butter has increased
si!nificantly and does not loo% li%ely to fall in the foreseeable future. As a result, the retail price of chocolate is
certain to increase within si$ months. The answer to which of the following questions would provide
information relevant to evaluating the claims made in the argument above?
&as the price of cocoa remained steady durin! other period of poor har"est?
Are consumers willin! to spend more for chocolate?
&a"e the prices of other in!redients in chocolate decreased recently?
Fhat percenta!e of cacao trees in Bra;il were affected by the fun!us?
3an the fun!us be eliminated within the ne$t si$ months?
,2. Scientists ha"e determined that an effecti"e way to lower cholesterol is to eat three ser"in!s of whole !rains e"ery
day. Studies ha"e shown that the cholesterol le"els of people who did so were si!nificantly lower after si$ months
than were those of people who did not, e"en thou!h the cholesterol le"els of the two !roups were the same before
the studies be!an. 3learly, eatin! whole !rains can ha"e an appreciable effect on cholesterol le"els. The answer
to which of the following questions, if true, would be most useful in evaluating the claim about whole
grains above?
Is it realistic to e$pect people to eat three ser"in!s of whole !rains per day?
Fere the two !roups of people in the study in"ol"ed in the same e$ercise pro!ram?
3an the same drop in cholesterol be achie"ed throu!h medication?
6id the study continue to trac% the sub7ects beyond si$ months?
Are most consumers aware of the different between whole !rains and processed !rains?
Critical Reasoi! To"ic 8$ Para*o9
,. >ree% tra!edy, one of the endurin! pillars of our belief system, dramati;ed the concept that the misfortune a person
suffers is not an accident, but rather a lo!ical outcome of flaws in that person5s natureI the misfortune is thus that
person5s 8fault.9 Aonetheless, today the public broadly supports ban%ruptcy protection, family welfare
and other 8social safety net9 pro!rams that shield the destitute in the face of their hardships, at ta$payer e$pense.
Which of the following, if true, would best resolve the parado$ in the statements above?
)he ancient >ree%s had few, if any, such social safety net pro!rams in their society.
)he ma7ority of the public is more familiar with the wor%s of Sha%espeare than those of >ree% tra!edy.
Some people insist that society, not the indi"idual, is to blame for most accidents.
(any people in financial difficulties feel too ashamed to declare ban%ruptcy or to ta%e ad"anta!e of other
social safety net pro!rams.
)he reli!ions practiced by most people today stron!ly encoura!e people to contribute to charities that assist
innocent people in7ured in natural disasters, such as hurricanes.
34
2. A certain pharmaceutical firm recently de"eloped a new medicine, 6endadrine, that pro"ides hi!hly effecti"e
treatment of se"ere stomach disorders that were pre"iously thou!ht to be incurable. &owe"er, to de"elop the
new medicine, the company spent nearly $5 billion in research and de"elopment costs. >i"en the si;e of the
mar%et for 6endadrine and the amount of the initial in"estment in its de"elopment, the company would need
to sell 6endadrine at a price that is at least 5 times !reater than its "ariable costs 7ust to brea% e"en. Het the
company5s mana!ement claims that 6endadrine will soon become the ma7or dri"er of the firm5s profits. Which
of the following statements best reconciles the managements claim with the evidence on the e$penditures
associated with the development of 4endadrine?
)he pharmaceutical firm has been !ranted a patent to become the sole producer and distributor of
6endadrine, and dru!s under patent protection typically sell at prices that are at least ,0 times !reater than
their "ariable costs.
6e"elopment of some pharmaceutical products in"ol"es substantial initial e$penditures on research, testin!,
and appro"al.
In issues related to personal health, corporate profits should not become the primary consideration.
Se"eral other pharmaceutical companies are wor%in! on new medicines that may become effecti"e
substitutes for 6endadrine.
Fhile 6endadrine can be hi!hly effecti"e in treatin! stomach disorders, it may also result in serious side
effects such as di;;iness and hallucinations.
3. Small community hospitals in poor urban areas almost always operate at a loss due to an unfortunate cycle of
factors. &i!h re"enue specialists, such as sur!eons, floc% to hospitals that are more presti!ious and can afford to
pay hi!her salaries. Aware of this, local residents patroni;e the more affluent nearby hospitals when they need
specialty care or for!o care entirely, while only utili;in! the local hospital for lowmar!in routine care. 4urther, a
si!nificant ma7ority of the community uses !o"ernment health plans, which reimburse poorly for routine care, or
lac%s insurance entirely and cannot pay. )he local hospital then loses money and cannot afford to hire specialists to
conduct the hi!hermar!in specialty care. Which of the following, if it could be accomplished, would best help
small community hospitals to break the pattern described above?
Ae!otiate hi!her reimbursement rates for specialty care with both !o"ernment health plans and pri"ate insurers.
Ad"ertise the hospital5s specialty care ser"ices in the local community as well as nearby communities to
attract more business.
:artner with a nearby affluent hospital to contract its specialists on a parttime, asneeded basis, which is
more affordable than hirin! these specialists full time.
=aunch a community outreach campai!n to educate the public about the low reimbursement rates of
!o"ernment health plans compared with the hi!h rates paid by pri"ate insurers.
Ae!otiate with pri"ate insurers for hi!her reimbursement rates for routine care.
4. )he climbin! season of 200+ was the deadliest on record for those attemptin! to con<uer (ount #"erest, the
world5s tallest mountain, as more people perished attemptin! to reach the summit in 200+ than in any other year.
Interestin!ly, most mountaineerin! e$perts attribute the hi!h number of fatalities, almost all of which occurred in the
unfor!i"in! 8death;one9 abo"e 2+,000 feet, directly to the e$ceptionally !ood weather pre"alent durin! the 200+
climbin! season. Which of the following, if true, best helps to e$plain the conclusion of the mountaineering
e$perts?
All of the forecasts were for e$tremely bad weatherI the !ood weather was a si!nificant surprise to all of the
climbers.
)he !ood weather prompted si!nificantly more people than e"er to try to reach the summit and enter the
8death;one,9 many of whom would ha"e turned bac% at a lower altitude in poorer weather.
)he !ood weather caused the 8death;one9 to ha"e warmer temperatures and less intense winds than in
recent years.
(odern e<uipment is particularly effecti"e in protectin! climbers from the elements in bad weather.
(any accomplished climbers don5t attempt (t. #"erest durin! !ood weather because they feel it is not a challen!e.
35
5. )he e$chan!e rate between the currency of 3ountry L and that of 3ountry H has historically fa"ored the
currency of 3ountry H. Because of this, citi;ens of 3ountry H often ta%e their "acations in 3ountry L, where
the e$chan!e rate ma%es hotels and restaurants more affordable. Het, citi;ens of 3ountry H rarely purchase
clothin! or electronics in 3ountry L, despite the fact that those items are more e$pensi"e in their home
country, e"en when sales ta$es are ta%en into account. Which of the following, if true, would best e$plain the
buying habits of the citi%ens of 'ountry -?
3iti;ens of 3ountry H prefer the fashions a"ailable in their own country.
Stores in 3ountry L recei"e the latest fashions and technolo!y se"eral months after they are a"ailable in
3ountry H.
)he citi;ens of 3ountry L resent the buyin! power of the currency of 3ountry H.
)he !o"ernment of 3ountry H imposes tariffs on imported !oods.
)he currencies of 3ountry L and 3ountry H are both wea% compared to the currency of 3ountry P.
6. Fhen Bob5s Bistro opened in the town of (ontrose last year, the proprietors of Andrew5s #atery, the only
other restaurant in town, feared that their business would suffer. Surprisin!ly thou!h, in the past year the
a"era!e number of meals per day ser"ed at Andrew5s #atery has actually increased si!nificantly. Which of
the following, if true, provides the best e$planation for this occurrence?
)he meals at Andrew5s #atery are substantially lower in price than those offered at Bob5s Bistro.
Bob5s Bistro is closed on Sundays, and so for Sunday brunch residents of (ontrose would either !o to
Andrew5s #atery or "enture to a nei!hborin! town.
)he profit per meal is hi!her, on a"era!e, at Bob5s Bistro than it is at Andrew5s #atery.
Bob5s Bistro attracts a lar!e number of patrons that had ne"er dined in (ontrose before, and on many days
Bob5s Bistro attracts more customers than it can seat.
Andrew5s #atery ser"es considerably more meals on wee%ends than it does on wee%days, which is not the
case at Bob5s Bistro.
.. En a"era!e, residents of 3ity L de"ote a !reater percenta!e of their yearly incomes to housin! costs than do
residents of 3ity H, thou!h the costs of insurance and fuel !enerally are e$orbitant in both commuterhea"y
cities. Het in Fealth (a!a;ine5s annual list of the country5s least affordable cities, 3ity H is deemed less
affordable than 3ity L. Which of the following, if true, best e$plains the contrast described above?
A !reater percenta!e of residents of 3ity H send their children to pri"ate schools than is the case in 3ity L.
Unli%e 3ity H, 3ity L has an efficient and ine$pensi"e public transportation system.
)he a"era!e price of a new house is hi!her in 3ity L than in 3ity H.
A number of hi!hpriced restaurants and bouti<ues ha"e recently opened in 3ity H.
Se"eral lar!e businesses ha"e recently relocated from 3ity H to 3ity L.
8. 3ertain oil companies ha"e been called poor corporate citi;ens because they ha"e opposed !o"ernment action to
limit !lobal warmin! by underminin! scientific research that characteri;es the issue as se"ere. &owe"er, these
same oil companies ha"e also in"ested millions of dollars in scientific research to address the lon! term effects of
climate chan!e. Which of the following best e$plains the apparent discrepancy in the situation described above?
)he oil companies only recently be!an in"estin! in scientific research to address climate chan!e issues.
)he research dollars in"ested by the oil companies are specifically earmar%ed for de"elopin! practical
technolo!ies that mi!ht be used to combat !lobal warmin!.
)he !o"ernment action opposed by the oil companies would ne!ati"ely impact their profits.
)he scientific research that characteri;es !lobal warmin! as a se"ere problem has not been definiti"ely pro"en.
)he oil companies don5t belie"e that any scientific research related to climate chan!e will ultimately ser"e
their interests.
@. 3ountry L contains many ri"ers that flow down from its hi!h mountains. )hese ri"ers ha"e been dammed to
harness the hydroelectric power that can be deri"ed from this resource. (ore than enou!h power is !enerated from
these dams to meet the country5s ener!y needs. Het, citi;ens of 3ountry L often e$perience power shorta!es
or e"en outa!es. Which of the following, if true, best e$plains the situation described above?
)he flow of the ri"ers is hea"ier in the sprin! than at other times of year.
3iti;ens of 3ountry L rely hea"ily on electronic appliances in their homes.
3ountry L has not in"ested in alternate sources of ener!y.
(ost of the electricity !enerated in 3ountry L is sold to other countries.
Some of the most powerful ri"ers in 3ountry L ha"e yet to be dammed.
36
10. As a result of consumers5 increased awareness of the health ris%s associated with hea"y consumption of red meat, the
meat of the ostrich has become increasin!ly popular as a lowfat, lowcholesterol alternati"e to beef. Accordin!ly, the
number of ostrich farms in the United States has nearly <uadrupled since ,@-0, floodin! mar%ets with ostrich products.
&owe"er, the priceperpound of ostrich meat, ad7usted for inflation, has not decreased at all o"er the past se"eral years.
Which of the following, if true, would best e$plain the steady price1per1pound of ostrich meat?
)he demand for ostrich products has outpaced the supply.
Estriches are especially difficult to breed in capti"ity.
Ether types of meat, such as por% and poultry, ha"e decreased in price o"er the past se"eral years.
:ur"eyors of beef ha"e not attempted to counter beef5s poor public ima!e.
)he number of supermar%ets that carry ostrich meat has increased steadily since ,@-0.
,,. =ife in an urban center, with daily e$periences of subways, crowded sidewal%s and dense li"in!, pro"ides countless
opportunities for social contact and connection. #$urban life, by contrast, e$ists in the openness of freeways and
oneacre lots, e$hibitin! little that would su!!est social commonality. Het studies show that people who li"e in
e$urbia consistently rate their feelin! of connectedness to their community as stron! whereas urban dwellers
more fre<uently e$press feelin!s of isolation. Which of the following, if true, contributes most to an
e$planation of the difference in the reported feelings of connectedness between urban and e$urban dwellers?
)he cost of li"in! in e$urbia is si!nificantly lower than that in urban areas.
2ates of attendance at houses of worship are much hi!her in e$urban areas than in urban ones.
#$urban school districts often spend more per capita on their students than do those in urban areas.
)he rate of "iolent crime is much lower in e$urban areas than in urban ones.
(any people who li"e in e$urbia mo"ed there from urban areas.
12. Fhen Americans filed their ta$ returns this year, the United States )reasury 6epartment collected $0,5., billion in
ta$ re"enue, a ,0.1' increase o"er last year. Het the number of ta$ returns filed by employees did not increase
si!nificantly o"er the past year. (ach of the following, if true, could e$plain the simultaneous increase in ta$
revenue collection and the unchanged number of employee ta$ returns filed ()'(&T*
6urin! this year, the number of women who reentered the wor%force as employees, after ta%in! time off to
raise children, was !reater than the number of people who retired from the wor%force.
(any retired wor%ers owed more in ta$es this year due to an increase in di"idends and capital !ains on their
stoc% in"estments.
=ast year, ta$ re"enues were unusually low because the stoc% mar%et e$perienced a mar%ed decline, and
many ta$payers reduced their ta$able income with capital losses.
3ompared with pre"ious years, a !reater percenta!e of the total ta$ re"enue this year was collected from
small businesses and corporations.
)he employees who filed ta$ returns were paid hi!her wa!es this year than they were in the past, and thus
owed more to the !o"ernment in ta$es per person.
,0. Amtown5s public schools are supported primarily by town property ta$es. )he town plans to eliminate the property
ta$ and support schools with a new fourpercent sales ta$ on all retail items sold in town. 4our percent of current
retail sales is substantially less than the amount currently collected throu!h property ta$es, but town planners
belie"e that adoptin! this plan will not ha"e a ne!ati"e impact on the funds a"ailable for the town5s public
schools because JJJJJJ. Which of the following most logically completes the argument?
a new shoppin! center that will draw shoppers from nei!hborin! towns will open this year in Amtown
many Amtown residents are plannin! to lea"e town due to hi!h property ta$es
a number of parents who currently send their children to Amtown5s public schools do not pay property ta$es.
the sales ta$ in nei!hborin! towns is much hi!her than four percent
Amtown was recently featured in a ma!a;ine as one of the top ,0 wee%end !etaways from a nearby city.
37
,1. In recent years, the &olds"ille )ransportation Authority B&)AD has noted consistent delays on its &olds"ille
2i"er Oalley bus, which runs from downtown &olds"ille to the suburb of 2i"er Oalley. In order to decrease the
commutin! time from &olds"ille to 2i"er Oalley, the &)A recently eliminated the Kames Street stop on the
&olds"ille2i"er Oalley line. &owe"er, data show that the a"era!e commutin! time from &olds"ille to 2i"er
Oalley has actually increased since the elimination of the stop. Which of the following provides the best
e$planation for the increase in commuting time from 0oldsville to 5iver 6alley?
Almost all of the commuters who pre"iously used the Kames Street stop now use the ad7acent >reen Street
stop, causin! o"ercrowdin! and e$cessi"e boardin! and deboardin! delays at the >reen Street stop.
A small percenta!e of the commuters who pre"iously used the Kames Street stop now use alternate modes of
transportation to commute from &olds"ille to 2i"er Oalley.
@0' of &olds"ille2i"er Oalley commuters were in fa"or of eliminatin! the Kames Street stop.
)he &olds"ille2i"er Oalley bus route runs alon! 2i"er Street, which is always con!ested with hea"y
automobile traffic.
)he Kohnstown bus line, another line operated by the &)A, has also e$perienced an increase in a"era!e
commutin! time since eliminatin! a stop on its route.
,5. 3ity 3ontroller* +0' of our residents "oted to appro"e the de"eloper5s re<uest to build a national chain hotel
on the site of the old consi!nment store. )he hotel will increase our re"enue base and, therefore, pro"ide
more money for schools and community ser"ices.
(ayor* But our recent sur"ey showed that the most important reason people want to li"e here is our small
town feel resultin! from the local ownership of the "ast ma7ority of businesses.
What is the best e$planation for the apparent contradiction in opinions cited by the controller and the mayor?
(ost people belie"e ha"in! a smalltown feel is more important than ha"in! <uality schools.
A locallyowned business mi!ht be able to !enerate as much re"enue as a well%nown hotel chain.
)he recent sur"ey did not as% about preferences for a chain hotel "ersus a locallyowned bed and brea%fast.
An increase in the town5s re"enue base may not result in additional money for the schools.
)he recent sur"ey cited by the mayor polled people who are considerin! mo"in! to the town.
16. In ,@+0, 3on!ress appro"ed the 3ommunity (ental &ealth 3enters Act, which outlined plans to release the
mentally ill from institutions, incorporate these indi"iduals into their communities, and pro"ide outpatient treatment.
=eadin! associations of mental health professionals o"erwhelmin!ly applauded these !oals and appro"ed of these
plans because, the e$perts said, the treatment rather than the institutional en"ironment was the crucial element for
the welfare of these patients. Fithin twenty years, state authorities succeeded in dischar!in! @5' of these patients
from institutional care. In ,@-0, howe"er, e$ecuti"es from these same professional associations said that the pli!ht
of the mentally ill was worse than e"er. Which if the following, if true, best resolves the parado$ in the above
passage?
(ore people were dia!nosed with psychiatric disorders in ,@-0 than in ,@+0.
(any mental health professionals belie"e that if their peers had administered the pro7ect rather than the state
authorities, the results would ha"e been better.
)he state bud!et allocation for ser"ices to the mentally ill has not increased faster than the rate of inflation.
3on!ress a!reed to fund these outpatient ser"ices, pro"ided that the money come from cuts in other
domestic pro!ramsI these cuts, howe"er, ne"er materiali;ed.
(any of the released patients had, at some time, been addicted to ille!al narcotics.
17. 3ompany L manufactures swim wear and planned to launch a new line of women5s bathin! suits in (arch, which is
typically the pea% time of year for swim wear sales. )he company conducted consumer polls, which returned
fa"orable results for both style and price, and too% out ad"ertisements in ma7or fashion ma!a;ines and tele"ision
stations. Het the launch was disappointin!* sales in (arch did not e$ceed e"en half of the company5s sales durin!
the same period in the pre"ious year. (ach of the following, if true, could e$plain the disappointing sales of the new
swim wear line ()'(&T*
Aone of the stores carryin! the new swim wear line displayed it prominently.
)he company5s manufacturin! plants e$perienced difficulty in obtainin! dyes in the ad"ertised colors and so
substituted different colors.
A ma7or competitor launched a line of similar swim wear at a lower price in 4ebruary.
A scene in which a ma7or actress was to wear one of the new swimsuits in a much anticipated mo"ie to be
released in 4ebruary was ne"er filmed.
)he prediction of a cool, rainy summer by meteorolo!ists recei"ed much attention in the national media.
38
Critical Reasoi! To"ic 7$ :ol*,ace
1. =ocal authorities are considerin! an amendment to the litter law that would raise the fine for litterin! in the
community picnic area to $,,000. Since the inception of the litter law, icre)etal icreases i the litteri!
,ie ha3e "ro3e to be cosistetl1 e,,ecti3e at ,urther re*uci! the a)out o, litter i the co))uit1
"icic area. &owe"er, raisin! the fine to $,,000 would actually ha"e the unintended effect of increasin! the
amount of litter in the picnic area. Picic area users +oul* "ercei3e this ,ie to be ureasoable a*
ue,orceable0 a* +oul* *isre!ar* the litter la+ alto!ether. In the argument, the two portions in
boldface play which of the following roles?
)he first is irrefutable e"idence that the author offers in support of a predictionI the second is that prediction.
)he first is a statement of causation that the author predicts will be repeated in the case at handI the second
raises e"idence a!ainst this prediction.
)he first is a statement of fact that the author accepts to be trueI the second is presented as a conse<uence
of this fact.
)he first is e"idence that wea%ens the main position that the author defendsI the second is that position.
)he first is a statement of causation that the author predicts will not hold in the case at handI the second
offers a line of reasonin! to support this prediction.
2. :olitical analyst* A party that temporarily positions itself in the ne!li!ible crac% between the American ri!ht and left
will do little to e$pand the public debate. Fhat America needs is a permanent third party. So)e clai) that
()erica;s success ste)s ,ro) the t+o "art1 s1ste)< )hese people say that a third party would ma%e the
passa!e of le!islation and thus !o"ernance impossible. 4urthermore, they point to the current slu!!ish pace of
!o"ernment as proof that the country cannot bear the burden of a third party. =et0 )ost Euro"ea coutries
ha3e )ulti-"art1 s1ste)s a* ,e+ co)"lai about a1 iabilit1 to !o3er there< Which of the following
best describes the functions of the two sections in boldface in the argument above?
)he first is the main point of the ar!umentI the second is a premise that supports that point.
)he first opposes the premises of the ar!umentI the second is the claim that the ar!ument supports.
)he first supports the main position held by opponents of the main pointI the second is a premise that ar!ues
a!ainst that position.
)he first is the primary claim made by opponents of the main point of the ar!umentI the second is e"idence
proposed in opposition to the first.
)he first is a claim made by opponents of the main point of the ar!umentI the second is the claim that the first
opposes.
0. =etter to the editor* :roposition C, a contro"ersial measure on this year5s ballot, would prohibit the ownership of
hand!uns within the city5s limits. Under the plan, !un owners would ha"e a @0day !race period to turn in their
weapons to authorities. :roponents of the proposition ar!ue that ,e+er ha*!us o the streets +oul* lea* to
less 3iolet cri)e0 )a2i! the cit1 sa,er ,or all o, its citi>es. Unfortunately, the ban would actually ha"e the
opposite effect. Since only law abidin! citi;ens would honor the ban, ar)e* cri)ials +oul* ot ol1 2ee" their
+ea"os but +oul* also ha3e the co,i*ece to act +ith i)"uit1 o a "o"ulatio that coul* o lo!er
*e,e* itsel,< In the letter to the editor, the two portions in boldface play which of the following roles?
)he first is an obser"ation that the author uses to support a particular positionI the second is that position.
)he first is a pattern of cause and effect that the author belie"es to be trueI the second offers e"idence to
contradict this pattern.
)he first is a position that the author ar!ues will not hold in this caseI the second is the author5s position.
)he first is a prediction that the author belie"es to be untrue. )he second is a statement of fact that
undermines the author5s position.
)he first is a direct relationship that the author belie"es will not hold in this caseI the second offers e"idence
in support of the author5s position.
39
1. Scientist* #"olutionary biolo!y has lon! held that the most attracti"e males of a species, defined as those with
the hi!hest <uality physical traits that ha"e no 6arwinian sur"i"al "alue, will draw the most female mates. The
resulti! )ale o,,s"ri! +ill iherit that attracti3eess a* the)sel3es ha3e )ore chil*re as a result,
thus ensurin! widespread dissemination of the !randparents5 !enes. 2ecently, howe"er, scientists ha"e found
that the sons of 8hi!h <uality9 male flycatchers failed to inherit the father5s matin! status. 4urther, the most
attracti"e males were so busy matin! that they ne!lected their offsprin!I as a result0 the sos o, ho)elier
bir*s0 +ho too2 better care o, their o,,s"ri!0 ha* )ore success at "ro"a!ati! the s"ecies. The two
portions in boldface play which of the following roles in the scientists argument?
)he first is the conclusion of a theory disputed by the scientistI the second is the scientist5s new contention
based upon the latest e"idence.
)he first is a premise of a lon!held biolo!ical theoryI the second is an e$ample of how this theory wor%s.
)he first is an e$planation of how a biolo!ical theory is thou!ht to wor%I the second is an e$ample of research
results that do not support this theory.
)he first is an e$ample of a theory that used to be pre"alentI the second is the new theory that is now
considered predominant by scientists.
)he first introduces a lon!held theory that the scientist is !oin! to dispro"eI the second is the scientist5s new
theory to replace the one she dispro"ed.
5. )raditionally, "ideo !ame manufacturers ha"e been most stron!ly influenced by serious "ideo !amers. Because
de"oted !amers ha"e historically purchased the ma7ority of "ideo !ames, companies react to the desires of this
mar%et se!ment. Aormally, de"oted !amers cra"e speed and actionI thus0 )ost )au,acturers cotiue to
"ro*uce !a)es +ith ,aster chi"s a* ,lashier !ra"hics. Unfortunately, faster chips and flashier !raphics are
no lon!er in the industry5s best interest. The *e3ote* !a)i! )ar2et is *ee"l1 sta!at0 a* it +o;t
soo e9"a*. )o infuse new life into the "ideo !ame mar%et, manufacturers must simplify the functionality of
their !ames. By doin! so, current non!amers will be attracted to 7oin the ran%s of "ideo !ame fans. In the
argument, the two portions in boldface play which of the following roles?
)he first is a situation that the author belie"es to be trueI the second offers e"idence to e$plain this situation.
)he first is a situation that the author ar!ues should not continueI the second pro"ides e"idence that supports
the author5s position.
)he first is a statement of fact that contradicts the author5s positionI the second is the author5s position.
)he first is a statement of fact that supports the author5s positionI the second is a consideration that wei!hs
a!ainst the author5s position.
)he first is a prediction that the author belie"es should not hold in this caseI the second is an assumption that
wei!hs a!ainst the author5s position.
6. (usical composers ha"e !enerally made their most si!nificant contributions to the musical canon before they
reached the a!e of thirty. It is commonly belie"ed that this is the case because a!i! bri!s about a loss o,
co!iti3e abilit1 a* creati3e ca"acit1. &owe"er, a study pointed out that a *is"ro"ortioatel1 lar!e u)ber o,
those co)"osers +ho )a*e their )ost si!i,icat cotributios to the cao a,ter the a!e o, thirt1 beca)e
)usicias at a ol*er a!e tha is !eerall1 the case. Since by the a!e of thirty many composers ha"e been
en!a!ed as musicians for a decade or more, these findin!s su!!est that the real reason why musicians o"er thirty
rarely ma%e si!nificant contributions to the musical canon is not that they ha"e a!ed but rather that they ha"e spent
too much time as musicians. In the argument above, the two portions in
boldface play which of the following roles?
)he first is an e$planation that is challen!ed by the ar!umentI the second is a findin! used to support that
e$planation.
)he first is an e$planation that the ar!ument opposesI the second is an ob7ection raised a!ainst the
alternati"e e$planation ad"ocated by the ar!ument.
)he first is a claim ad"anced in support of a positionI the second is a findin! that clarifies that position.
)he first is an e$planation ad"ocated by the ar!umentI the second is a findin! used to challen!e that e$planation.
)he first is an e$planation that the ar!ument challen!esI the second is a findin! on which that challen!e is based.
40
.. )he city !o"ernment should in"est surplus funds in impro"in! the city5s transportation networ%. ?ost o, the
et+or2 +as "ut i "lace at a ti)e +he the cit1 +as )uch s)aller i both area a* "o"ulatio. )he subway
system is outdated and understaffed. )he buses rarely run on schedule and their routes are incon"enient.
I, the cit1 *oes ot )a2e cha!es soo to the et+or20 it +ill see )a1 o, its "ri>e* i*ustries
relocate to )ore co3eiet cities a*0 as a result0 the cit1;s ,iacial health +ill be @eo"ar*i>e*< In the
argument above, the two portions in boldface play which of the following roles?
)he first is an e$planation of a current state of affairsI the second is a prediction based on that state of affairs.
)he first is a statement of fact in opposition to the author5s conclusionI the second is that conclusion.
)he first emphasi;es an e$istin! problemI the second offers a proposal to sol"e that problem.
)he first is information the author su!!ests has been o"erloo%ed in the situation at handI the second
describes that situation.
)he first is a 7ustification of an impendin! problemI the second describes the conse<uences of that problem.
-. 3hief #conomist* Usually, the release o, ecoo)ic *ata about hi!her-tha-e9"ecte* !ro+th i the 5ross
Ao)estic Pro*uct (5AP) results i a icrease i stoc2 "rices< &owe"er, this <uarter, the release of data
about stron! >6: !rowth is most li%ely to result in a decrease rather than an increase in stoc% prices. Robust
5AP !ro+th +ill lea* to hi!her iterest rates0 icreasi! the attracti3eess o, bo*s a* causi! a
shi,t o, ca"ital ,ro) eBuit1 to *ebt securities< In the above argument, the statements in boldface play
which of the following roles?
)he first ac%nowled!es a consideration a!ainst the main conclusion of the chief economistI the second is that
conclusion.
)he first is a pattern of cause and effect that the chief economist predicts will not hold in the case at issueI the
second offers a consideration in support of that prediction.
)he first is a !enerali;ation that the chief economist accepts as trueI the second is a conse<uence that
follows from that !enerali;ation.
)he first is e"idence that the chief economist pro"ides in support of a certain predictionI the second is that
prediction.
)he first is a pattern of cause and effect that the chief economist predicts will be repeated in the case at
issueI the second ac%nowled!es a circumstance in which that pattern would not hold.
@. :olitical Analyst* After the So"iet Union collapsed, some hoped that freedom would encoura!e 2ussians to multiply, but as
a result of dislocation and insecurity, the 2ussian population continues to dwindle at the rate of .00,000 a year. )he
!o"ernment proposes to address the problem with a wide ran!e of financial incenti"es, alon! with in"estments in
impro"ed health care, road safety and the li%e. )hese are positi"e measures, but the1 ha3e bee trie* be,ore0 to little
a3ail< A better plan to re"erse the population decline is to impro"e the country5s !o"ernance
in both the public and the pri"ate sphere. I, a !reater "art o, the "o"ulatio "artici"ate* i i)"ortat *ecisios
a* share* i the coutr1;s +ealth0 the lar!er ,a)ilies +oul* result< In addition, if corruption and !reed
amon! the elite were curbed, public health would impro"e and a"era!e life e$pectancy would increase.
The two boldfaced statements serve what function in the argument above?
)he first is the main point of the analyst5s ar!umentI the second is a premise that supports the first.
)he first is a premise that undermines an alternati"e to the analyst5s proposalI the second is a premise that
supports the analyst5s main claim.
)he first is a premise that contradicts the main point made by the analystI the second is the main point of the
ar!ument.
)he first is a premise that supports a proposalI the second is that proposal.
)he first is a conclusion that the ar!ument endorsesI the second is a premise that opposes that conclusion.
,0. United #ner!y recently in"ested in a series of lar!e windmills which are able to produce renewable ener!y with
minimal ne!ati"e effect to the en"ironment. )he company has not drilled oil wells in the same area, e"en thou!h
!reater re3eues a* "ro,its coul* be !eerate* ,ro) oil +ells. Because any drillin! would disrupt the nati"e
habitat of certain marine species in the area, some en"ironmentalists assert that, by fore!oin! this drillin!,
&ite* Eer!1 has establishe* that it "laces e3iro)etal i)"act o3er ,iacial returs. &owe"er, United
#ner!y may be actin! in a manner consistent with its financial !oals. 2ecent patterns of increasin! annual
hurricane acti"ity ha"e some e$perts <uestionin! the lon!term "iability and profitability of oil wells in the area.
The two boldfaced portions play which of the following roles?
)he first supports the conclusion of the ar!umentI the second calls that conclusion into <uestion.
)he first states the conclusion of the ar!umentI the second supports that conclusion.
)he first supports the en"ironmentalists5 conclusionI the second states that conclusion.
)he first states the en"ironmentalists5 conclusionI the second pro"ides a consideration in support of that conclusion.
)he first supports the conclusion of the ar!umentI the second also supports the conclusion of the ar!ument.
41
11. (ar%etin! Analyst* )raditionally, itro*uci! a e+ cosu)er "ro*uct cotributes to the *i3ersit1 o, a
co)"a1;s "ro*uct )i9 a* results i a o3erall icrease i re3eues a* "ro,its< &owe"er, our launch of a
new brand of soft drin%s, 3ool Bree;e, planned for the end of this fiscal year, will almost certainly reduce rather
than increase the company5s profits. Since we already offer a line of chilled refreshments that are "ery similar to
3ool Bree;e, the launch of the new drin% is li%ely to cannibali;e our current sales rather than attract new
customers. Additionally, sice the costs o, "ro*uci! Cool :ree>e are hi!her tha those o, the other chille*
re,resh)ets0 a lar!e "art o, our curret sales +ill be re"lace* +ith lo+er-)ar!i re3eues<
In the argument above, which of the following statements best describes the role played by each portion in
boldface?
)he first is a pattern of cause and effect that the mar%etin! analyst predicts will be repeated in the case at
issueI the second ac%nowled!es a circumstance in which that pattern would not hold.
)he first is a !enerali;ation that the mar%etin! analyst accepts as trueI the second is a conse<uence that
follows from that !enerali;ation.
)he first ac%nowled!es a consideration a!ainst the main conclusion of the mar%etin! analystI the second is
that conclusion.
)he first is a pattern of cause and effect that the mar%etin! analyst predicts will not hold in the case at issueI
the second offers a consideration in support of that prediction.
)he first is e"idence that the mar%etin! analyst pro"ides in support of a certain predictionI the second is that
prediction.
,2. =etter writer* Ille!al dru! use is often associated with other serious problems, such as armed robbery and other
"iolent crimes. Statistics indicate that each ti)e "olice icrease their e,orce)et o, ati-*ru! la+s i the cit10
the u)ber o, 3iolet cri)es co))itte* i the cit1 *eclies as a result. &owe"er, eliminatin! criminal
penalties for dru! use would almost certainly decrease rather than increase the incidence of armed robbery and
other "iolent crime. I, *ru!s +ere o lo!er ille!al0 the "rice +oul* *ro" "reci"itousl10 a* *ru! users +oul*
o lo!er ee* to co))it cri)es to acBuire the )oe1 ecessar1 to su""ort their
*ru! habits. In the letter writers argument, the two portions in boldface play which of the following roles?
)he first is support offered by the letter writer for a certain forecastI the second is that forecast.
)he first ac%nowled!es an obser"ation that refutes the main position that the letter writer ta%esI the second is
that position.
)he first is a direct relationship between two acti"ities that the letter writer ar!ues is an infallible predictor of
future e"entsI the second ac%nowled!es a circumstance in which that relationship would not apply.
)he first is a direct relationship between two acti"ities that the letter writer predicts will not hold in the futureI
the second offers information that, if true, would support that prediction.
)he first is a statement that the letter writer belie"es is trueI the second is presented as a lo!ical inference
drawn from the truth of that statement.
,0. 2eal #state A!ent* Ae$t year, the occu"ac1 rate ,or resi*etial retal "ro"erties i Ri3erto +ill icrease
*es"ite "ro@ectios o, *ecreasi! "o"ulatio i the cit1< >enerally, when a city5s population decreases, so does
the occupancy rate for residential rental properties. &owe"er, i the cotiui! cli)ate o, icreasi! iterest
rates o ho)e )ort!a!es0 a uusuall1 lar!e u)ber o, Ri3erto resi*ets +ho +oul* t1"icall1 bu1 ho)es
+ill o"t to ret istea*< The boldfaced statements in the argument above play which of
the following roles?
)he first is a conclusion drawn by the a!entI the second is e"idence that wei!hs a!ainst the a!ent5s conclusion.
)he first is a consideration that wei!hs a!ainst the a!ent5s predictionI the second is that prediction.
)he first describes a pattern of cause and effectI the second describes a situation for which this pattern will
not hold.
)he first is the a!ent5s predictionI the second is reasonin! offered in support of this prediction.
)he first is an undisputed statement of factI the second is a position that is supported by this fact.
42
,1. :olicy analyst* ?ost !o3er)et a!ecies beco)e less e,,ecti3e o3er ti)e< Some e$perts !o so far as to
recommend that e"ery a!ency be eliminated after ,0 years and created anew by replacin! all of its e$istin!
personnel and re"ampin! its bureaucratic structure. &owe"er, this policy would be impractical since certai
!o3er)et a!ecies "er,or) 3ital ,uctios0 such as "rotecti! atioal securit10 a* there,ore
caot a,,or* e3e te)"orar1 u"hea3al. In the policy analysts argument above, the two portions in
boldface play which of the following roles?
)he first is e"idence offered in support of an opinion that the policy analyst re7ectsI the second offers
information that contradicts that e"idence.
)he first is a premise that the policy analyst accepts but ar!ues a!ainstI the second offers e"idence that
supports the analyst5s position.
)he first is a position that the policy analyst ar!ues a!ainstI the second is the position that the analyst defends.
)he first is a !enerali;ation that the policy analyst accepts as accurate and is used as the basis for an opinion
that the analyst re7ectsI the second is a consideration used to defend the analyst5s position.
)he first is a !enerali;ation that the policy analyst accepts as accurate and is used as the basis for the
analyst5s positionI the second offers another consideration used to defend that position.
,5. :olitical 3andidate* 5o3er)et subsi*i>e* "rescri"tio *ru! "las that +oul* allo+ i*i3i*uals si!i,icat
choice i *eter)ii! their bee,its a* costs are *ece"ti3el1 a""eali! to u)erous sta2ehol*ers<
&owe"er, buyin! prescription dru! co"era!e, li%e buyin! health insurance co"era!e, is not li%e buyin! a car. The
cosu)er caot "re*ict his or her ,uture health ee*s< (oreo"er, the administrators of the choicebased dru!
plans under consideration are allowed to chan!e the dru!s they co"er and the prices they char!e at any timeI this
renders informed consumer choice meanin!less and ma%es securin! appropriate co"era!e a crap shoot.
Elder and disabled indi"iduals, the predominant consumers of !o"ernment subsidi;ed prescription dru!
plans, should be offered dru! co"era!e alternati"es that do not force them to !amble with their health. In the
argument above, the two portions in boldface play which of the following roles?
)he first is a fact that the candidate ar!ues a!ainstI the second is the ultimate claim that the candidate supports.
)he first is an obser"ation which the candidate ac%nowled!es as true but to which he is ultimately opposedI
the second is a claim that the candidate uses as e"idence to support his ultimate position.
)he first is an obser"ation that the candidate ac%nowled!es as true but unfortunateI the second is an
assertion that the candidate ma%es to support his ultimate position.
)he first is an obser"ation that the candidate ar!ues a!ainstI the second is an obser"ation that the candidate
supports.
)he first is an obser"ation made by the candidateI the second is an assertion that the candidate ultimately
opposes.
16. Studies ha"e shown that "eo"le +ho 2ee" *ail1 *iet recor*s are ,ar )ore success,ul at losi! +ei!ht tha
"eo"le +ho *o;t 2ee" trac2 o, +hat the1 eat. 2esearchers belie"e that many wei!htloss efforts fail because
people eat more calories than they intend to consume. Ene study followed a !roup of patients who reported that
they could not lose wei!ht when consumin! only ,,200 calories a day. The stu*1 ,ou* that the !rou"
cosu)e*0 o a3era!e0 47C )ore tha it clai)e* a* e9ercise* 7#C less< In contrast, when dieters record
what they eat, their actual consumption more closely matches their reported consumption. The two
boldface portions in the argument above are best described by which of the following statements?
)he first is a conclusion reached by researchersI the second is e"idence that that conclusion is correct.
)he first is an e$planation of why a certain theory is thou!ht to be trueI the second is an e$ample of research
results that support this theory.
)he first is an e$ample illustratin! the truth of a certain theoryI the second is a competin! theory.
)he first is a premise upon which the researchers base their opinionI the second illustrates that their opinion
is correct.
)he first introduces a theory that the researchers ha"e dispro"edI the second is the basis for the researchers5
ar!ument.
#7< Wei!ht loss "ro!ra)s that !uaratee results )islea* their custo)ers< Ao pro!ram can ensure that someone
who follows it will lose wei!ht. )hese pro!rams prey upon unhappy and insecure people who are often dri"en more
by emotion than by reason. ?oreo3er0 )a1 "eo"le +ho lose +ei!ht +hile o a "ro!ra)
e3etuall1 re!ai the +ei!ht +ithi a 1ear< So while the pro!rams5 claims may be true for a short period,
customers will be disappointed in the lon! run. The two portions in boldface play which of the following roles?
)he first is a !enerali;ationI the second is a conse<uence of that !enerali;ation.
)he first is the author5s positionI the second is a consideration in support of that position.
)he first is an opinion under e$aminationI the second is e"idence wei!hin! a!ainst that opinion.
)he first is an assertion that the author <uestionsI the second is e"idence in support of that assertion.
)he first is e"idence in support of the author5s main pointI the second is the author5s main point.
43
,-. 4or nearly a century, physiolo!ists erroneously belie"ed that a buil*u" o, lactic aci* i )uscle tissue +as
res"osible ,or the soreess that )a1 "eo"le e9"eriece a,ter streuous e9ercise< )he acid, they claimed,
is the waste produced by metabolic acti"ity in the muscle and reaches 8threshold9 le"els, causin! soreness, when
the muscle has depleted its o$y!en supply. Researchers ha3e recetl1 *isco3ere*0 ho+e3er0 that lactic aci* is
actuall1 the ,uel that "o+ers )uscular acti3it1< )herefore, the cause of muscle soreness remains un%nown. In
the argument above, the portions in boldface play which of the following roles?
)he first is an assertion that the author accepts as trueI the second is a consideration in support of that assertion.
)he first is an assertion that the author accepts as trueI the second describes a situation that the author
posits as contrary to that assertion.
)he first is an assertion that the author ar!ues a!ainstI the second is e"idence presented as contrary to the
author5s ar!ument.
)he first is e"idence that the author belie"es is no lon!er "alidI the second is additional e"idence that the
author uses to support his main point.
)he first is a claim that the author belie"es to be in"alidI the second is the author5s main point.
#D< The "resi*et;s o)iees to ,e*eral circuit courts ha3e bee @u*!e* coser3ati3e ,or their sta*s o hot-
butto issues< But a re"iew of their financial disclosure forms and Senate <uestionnaires re"eals that the
nominees are more notable for their close ties to corporate and economic interests, especially the ener!y and
minin! industries. Some of them were paid lobbyists for those same interests. 4urther, the o)iees +ith
i*ustr1 ties +ere o3er+hel)i!l1 a""oite* to circuit courts re!ar*e* as tra*itioal battle!rou*s o3er
liti!atio a,,ecti! these i*ustries< Independent obser"ers who follow the federal bench belie"e that the
e$tensi"e corporate in"ol"ement amon! so many of the nominees is unprecedented. In the argument above,
the two portions in boldface play which of the following roles?
)he first is a !enerali;ation that the author aims to attac%I the second is that attac%.
)he first is a pattern that the author ac%nowled!es as trueI the second is the author5s conclusion based on
that ac%nowled!ment.
)he first is a phenomenon that the author accepts as trueI the second is e"idence in support of the author5s
conclusion.
)he first is the author5s position based on the e"idence citedI the second is a pattern presented in support of
that position.
)he first is an e$ception to a rule introduced in the ar!umentI the second pro"ides the reasonin! behind the
e$ception.
20. I, iterest rates re)ai at their curret hi!h le3els0 )a1 "eo"le +ho curretl1 ret their resi*eces +ill
hesitate to "urchase ho)es< As the price of real estate continues to climb, the costs of a mort!a!e will be too
burdensome. Sellers will be forced to lower their as%in! prices. So0 hi!h iterest rates +ill e3etuall1 cause the
real estate )ar2et to stabili>e< In the argument above, the portions in boldface play which of the
following roles?
)he first is the author5s main pointI the second is a prediction that follows from that point.
)he first is a consideration that the author belie"es will result in a certain situationI the second is that situation.
)he first is a consideration that wei!hs a!ainst the author5s main pointI the second is the author5s main point.
)he first is a predictionI the second is e"idence in support of that prediction.
)he first is the author5s main pointI the second is e"idence used to ar!ue a!ainst that point.
2,. 3orporate Strate!ist* It is !enerally true that a reduction in the price of a !ood results in an increase in the demand
for this product, leadin! to hi!her sales. &owe"er, I belie"e that the )aa!e)et;s strate!1 o, sti)ulati! the
sales o, our lu9ur1 cars b1 i)"le)eti! a series o, a!!ressi3e "rice re*uctios is seriousl1 ,la+e*<
6ramatic price reductions on our lu$ury cars will erode the ima!e of e$clusi"ity and premium
<uality associated with these "ehicles. I, our cars beco)e substatiall1 chea"er0 the1 +ill o lo!er
re"reset the s1)bol o, status a* ,iacial success0 thus losi! their )ai a""eal to our custo)ers<
Which of the following statements best describes the role of each portion in boldface in the argument above?
)he first represents the main position of the corporate strate!istI the second ac%nowled!es a consideration
that wei!hs a!ainst that position.
)he first is an assumption made by the corporate strate!ist about the efficacy of the mana!ement5s strate!yI
the second is e"idence that supports the strate!ist5s reasonin!.
)he first is e"idence supportin! the main position of the corporate strate!istI the second is that position.
)he first is e"idence supportin! the position of the corporate strate!istI the second is a !enerali;ation that will
not hold in the case at issue.
)he first is the main position of the corporate strate!istI the second is e"idence in support of that position.
44
22. The ,i!ht a!aist the *ru! tra*e i Coutr1 X shoul* ,ocus ,or the ti)e bei! o ti!htei! the coutr1;s
bor*ers a* tar!eti! its )a@or s)u!!lers< Fipin! out poppy fields in rural areas means e"en !reater hardship
for an economically depressed farmin! population. Rather0 the &ite* %atios a* the !o3er)et o, Coutr1
X )ust care,ull1 rebuil* a!ricultural i,rastructure i areas +here the ecoo)1 *e"e*s o these "o""1
,iel*s< What purpose do the two boldface sentences serve in the passage?
)he first is the conclusion drawn by the spea%erI the second is the alternati"e to that conclusion.
)he first is a shortterm solution to a problemI the second is a lon!term solution to the same problem.
)he first presents a problemI the second poses an ideal solution to the problem.
)he first presents a popular solution to a problemI the second presents a solution preferred by the author.
)he first presents an ar!umentI the second presents e"idence to support the ar!ument.
23. (ayor* Some of my critics claim that the city5s current bud!et deficit has been caused by my policies, and that I am
responsible for the deficit. Althou!h I admit that the cit1 has ru a bu*!et *e,icit *uri! )1 teure, I do not
a!ree that I am at fault for this problem. )he economic policies of the prior administration caused the current
deficit, and +ere it ot ,or the ecoo)ic "olicies o, )1 a*)iistratio0 the curret *e,icit +oul* be
e3e +orse< In the mayor5s ar!ument, the two bol*,ace portions play which of the followin! roles?
)he first is a premise that has been used a!ainst the mayorI the second supports the critics of the mayor.
)he first is a statement accepted by the mayorI the second is a conse<uence of the critics5 claims.
)he first is a fact that the mayor belie"es does not contradict his conclusionI the second offers support in
consideration of that conclusion.
)he first is e"idence of unlawful acti"ity by the mayorI the second is e"idence offered by the mayor to e$plain
that acti"ity.
)he first is e"idence that undermines the mayor5s main positionI the second is a statement that follows from
that position.
24. E3iro)etal or!ai>atios +at to "reser3e the la* surrou*i! the Wil!ri Wil*eress (rea ,ro)
resi*etial *e3elo")et< )hey plan to do this by purchasin! that land from the farmers who own it. )hat plan is ill
concei"ed* if the farmers did sell their land, they would sell it to the hi!hest bidder, and de"elopers would outbid any
other bidders. En the other hand, these ,ar)ers +ill e3er actuall1 sell a1 o, the la*0 "ro3i*e* that ,ar)i!
it re)ais 3iable< But farmin! will not remain "iable if the farms are left unmoderni;ed, and most of the farmers
lac% the financial resources moderni;ation re<uires. And that is e$actly why a more sensible
preser"ation strate!y would be to assist the farmers to moderni;e their farms to the e$tent needed to maintain
"iability. In the ar!ument as a whole, the two bol*,ace proportions play which of the followin! roles?
)he first presents a !oal that the ar!ument re7ects as illconcei"edI the second is e"idence that is presented
as !rounds for that re7ection.
)he first presents a !oal that the ar!ument concludes cannot be attainedI the second is a reason offered in
support of that conclusion.
)he first presents a !oal that the ar!ument concludes can be attainedI the second is a 7ud!ment disputin!
that conclusion.
)he first presents a !oal, strate!ies for achie"in! which are bein! e"aluated in the ar!umentI the second is a
7ud!ment pro"idin! a basis for the ar!ument5s ad"ocacy of a particular strate!y.
)he first presents a !oal that the ar!ument endorsesI the second presents a situation that the ar!ument
contends must be chan!ed if that !oal is to be met in the foreseeable future.
25. Ecoo)ist$ )ropicorp, which constantly see%s profitable in"estment opportunities, has been buyin! and clearin!
sections of tropical forest for cattle ranchin!, althou!h pastures newly created there become useless for !ra;in!
after 7ust a few years. )he company has not !one into rubber tappin!, e"en thou!h !reater "ro,its ca be )a*e
,ro) rubber ta""i!, which lea"es the forest intact. )hus, some en"ironmentalists conclude that Tro"icor" has
ot acte* +holl1 out o, ecoo)ic sel,-iterest< &owe"er, these en"ironmentalists are probably wron!. )he initial
in"estment re<uired for a successful rubbertappin! operation is lar!er than that needed for a cattle ranch.
4urthermore, there is a shorta!e of wor%ers employable in rubbertappin! operations, and finally, ta$es
are hi!her on profits from rubber tappin! than on profits from cattle ranchin!. In the economist5s ar!ument,
the two bol*,ace* portions play which of the followin! roles?
)he first supports the conclusion of the economist5s ar!umentI the second calls that conclusion into <uestion.
)he first states the conclusion of the economist5s ar!umentI the second supports that conclusion.
)he first supports the en"ironmentalists5 conclusionI the second states that conclusion.
)he first states the en"ironmentalists5 conclusionI the second states the conclusion of the economist5s ar!ument.
#ach supports the conclusion of the economist5s ar!ument.
45
26. It is well documented that people ha"e positi"e responses to some words, such as 8%ind9 and 8wonderful9 and
ne!ati"e responses to others, such as 8e"il9 and 8nausea.9 2ecently, psycholo!ical e$periments ha"e
re"ealed that "eo"le also ha3e "ositi3e or e!ati3e res"oses to )a1 osese +or*s. )his shows
that people5s responses to words are conditioned not only by what the words mean, but also by how they
sound. )he bol*,ace portion plays which one of the followin! roles in the ar!ument?
A. It is a premise offered in support of the conclusion that people ha"e either a positi"e or a ne!ati"e response
to any word.
B. It is a conclusion for which the only support pro"ided is the claim that people5s responses to words are
conditioned both by what the words mean and by how they sound.
3. It is a !enerali;ation partially supported by the claim that meanin!ful words can tri!!er positi"e or ne!ati"e
responses in people.
6. It is a premise offered in support of the conclusion that people5s responses to words are en!endered not only
by what the words mean, but also by how they sound.
#. It is a conclusion supported by the claim that people5s responses under e$perimental conditions are
essentially different from their responses in ordinary situations.
27. :sycholo!ist* The obli!atio to e9"ress !ratitu*e caot be ,ul,ille* ao1)ousl1. &owe"er much
society may ha"e chan!ed o"er the centuries, human psycholo!y is still dri"en primarily by personal
interaction. )hus, the important social function of positi"ely reinforcin! those beha"iors that ha"e beneficial
conse<uences for others can be ser"ed only if the benefactor %nows the source of the !ratitude. )he
bol*,ace portion plays which one of the followin! roles in the ar!ument?
A. It is an illustration of a premise that is used to support the ar!ument5s conclusion.
B. It is used to counter a consideration that mi!ht be ta%en to undermine the ar!ument5s conclusion.
3. It is used to support indirectly a claim that the ar!ument in turn uses to support directly the conclusion.
6. It is used to identify the social benefit with which t he ar!ument is concerned.
#. It is the conclusion that the ar!ument is intended to support.
2-. Seemin!ly inconse<uential chan!es in sea temperature due to !lobal warmin! e"entually result in declines in fish
and seabird populations. A rise of 7ust two de!rees pre"ents the "ertical mi$in! of seawater from different strata.
)his restricts the a"ailability of upwellin! nutrients to phytoplan%ton. Since >oo"la2to0 +hich ,ee* u"o
"h1to"la2to0 feed the rest of the food chain, the declines are ine"itable. )he bol*,ace portion plays which
one of the followin! roles in the ar!ument?
A. It is a hypothesis supported by the fact that phytoplan%ton feed on upwellin! nutrients.
B. It is intended to pro"ide an e$ample of the ways in which the "ertical mi$in! of seawater affects feedin! habits.
3. It helps show how !lobal temperature chan!es affect lar!er sea animals directly.
6. It is offered as one reason that !lobal warmin! must be curtailed.
#. It is offered in support of the idea that !lobal warmin! poses a threat to all or!anisms.
29. Autritionist* Because humans ha"e e"ol"ed "ery little since the de"elopment of a!riculture, it is clear that hu)as
are still biolo!icall1 a*a"te* to a *iet o, +il* ,oo*s, consistin! mainly of raw fruits and "e!etables, nuts and
seeds, lean meat, and seafood. Strayin! from this diet has often resulted in chronic illness and other physical
problems. )hus, the more our diet consists of wild foods, the healthier we will be. )he bol*,ace portion plays
which one of the followin! roles in the ar!ument?
A. It is a conclusion for which the only support offered is the claim that strayin! from a diet of wild foods has
often resulted in chronic illness and other physical problems.
B. It is a premise for which no 7ustification is pro"ided, but which is used to support the ar!ument5s main conclusion.
3. It is a phenomenon for which the main conclusion of the nutritionist5s ar!ument is cited as an e$planation.
6. It is an intermediate conclusion for which one claim is offered as support, and which is used in turn to support
the ar!ument5s main conclusion.
#. It is a premise offered in support of the claim that humans ha"e e"ol"ed "ery little since the de"elopment of
a!riculture.
46
0,. :edi!reed do!s, includin! those officially classified as wor%in! do!s, must conform to standards set by
or!ani;ations that issue pedi!rees. )hose standards !enerally specify the physical appearance necessary for a do!
to be reco!ni;ed as belon!in! to a breed but stipulate nothin! about other !enetic traits, such as those that enable
breeds ori!inally de"eloped as wor%in! do!s to perform the wor% for which they were de"eloped. Since do!
breeders try to maintain only those traits specified by pedi!ree or!ani;ations, and traits that breeders do not try to
maintain ris% bein! lost, certai traits li2e her*i! abilit1 ris2 bei! lost a)o! "e*i!ree* *o!s. )herefore,
pedi!ree or!ani;ations should set standards re<uirin! wor%in! ability in pedi!reed do!s classified as wor%in! do!s.
)he bol*,ace portion plays which one of the followin! roles in the ar!ument?
A. It is a claim on which the ar!ument depends but for which no support is !i"en.
B. It is a subsidiary conclusion used in support of the main conclusion.
3. It ac%nowled!es a possible ob7ection to the proposal put forth in the ar!ument.
6. It summari;es the position that the ar!ument as a whole is directed toward discreditin!.
#. It pro"ides e"idence necessary to support a claim stated earlier in the ar!ument.
0,. :rofessor Kones has claimed that chemical compound chlorocetin contained in industrial waste and pre"iously
considered harmless is in fact "ery dan!erous. Kones has e$amined se"eral areas with hi!h concentration of
chlorocetin and found that certai 3ital biolo!ical "rocesses0 such as "hotos1thesis0 are slo+er i these
areas tha is usual ,or the is"ecte* s"ecies< )he professor says that althou!h he failed to establish an e$act
mechanism by which chlorocetin hampers photosynthesis, his findin!s are sufficient to state that chlorocetin is a
dan!erous chemical affectin! natural world. )his conclusion, howe"er, is unwarranted because all that
Pro,essor Eoes actuall1 establishe* is a )ere correlatio bet+ee the le3el o, chloroceti a* the s"ee*
o, "hotos1thesis R correlation that mi!ht not reflect any causal relationship between the factors. )he
hi!hli!hted portions of the te$t perform which of the followin! functions in the ar!ument?
A. the first is the position the author disa!rees with while the second is the author5s own position
B. the first contains the ar!ument the author criti<ues while the second identifies a lo!ical flaw in this ar!ument
3. the first is an e"idence in the ar!ument the author disputes while the second is a statement supportin! the
author5s own position
6. the first describes a phenomenon which wei!hs a!ainst the author5s opinion while the second reasons in
fa"or of this opinion
#. the first contains information that, if true, could o"erthrow the author5s ar!ument while the second discredits
this information
02. Business 3onsultant* Some corporations shun the use of e$ecuti"e titles because they fear that the use o, titles
i*icati! "ositio i the cor"oratio te*s to ihibit co))uicatio u" a* *o+ the cor"orate hierarch1<
Since an e$ecuti"e who uses a title is treated with more respect by outsiders, howe"er, use o, a title
ca ,acilitate a e9ecuti3e;s *eali!s +ith e9teral busiesses< )he ob"ious compromise is for these
e$ecuti"es to use their corporate titles e$ternally but not internally, since e"en if it is widely %nown that the
corporation5s e$ecuti"es use e$ecuti"e titles outside their or!ani;ation, this %nowled!e does not by itself
inhibit communication within the corporation. In the consultant5s reasonin!, the two portions in bol*,ace play
which of the followin! roles?
A. )he first presents an obstacle to achie"in! a certain !oalI the second presents a reason for considerin! that
!oal to be undesirable.
B. )he first is a consideration that has led to the adoption of a certain strate!yI the second presents a reason
a!ainst adoptin! that strate!y.
3. )he first describes a concern that the consultant dismisses as insi!nificantI the second is a consideration that
ser"es as the basis for that dismissal.
6. )he first is a belief for which the consultant offers supportI the second is part of that support.
#. )he first is a belief a!ainst which e"idence is offeredI the second is part of the e"idence offered a!ainst that belief.
47
00. 3ritics of certain pollutioncontrol re!ulations ha"e claimed that the )oe1 s"et o3er the last *eca*e i
or*er to re*uce e)issios o, carbo )oo9i*e a* o, 3olatile or!aic co)"ou*s has bee +aste*<
)he e"idence they offer in support of this claim mi!ht appear compellin!* despite the money spent, annual
emissions of these pollutants ha"e been increasin! steadily. )his e"idence is far from ade<uate, howe"er,
since o3er the last *eca*e a substatial u)ber o, e+ i*ustrial ,acilities that e)it these "ollutats
ha3e bee built< In the reasonin! !i"en, the two portions in bol*,ace play which of the followin! roles?
A. )he first identifies a claim that the reasonin! see%s to show is falseI the second is e"idence that has been
cited by others in support of that claim.
B. )he first identifies a claim that the reasonin! see%s to show is falseI the second is a position for which the
reasonin! see%s to pro"ide support.
3. )he first is a position that the reasonin! contends is inade<uately supported by the e"idenceI the second is a
position for which the reasonin! see%s to pro"ide support.
6. )he first is a position that the reasonin! contends is inade<uately supported by the e"idenceI the second is
e"idence used to support the reasonin!5s contention.
#. )he first is a position that the reasonin! contends is inade<uately supported by the e"idenceI the second is
e"idence that has been used to support that position.
01. Astronomer* Ebser"ations of the Shoema%er=e"i comet on its collision course with Kupiter showed that the comet
bro%e into fra!ments before enterin! Kupiter5s atmosphere in ,@@1, but they did not show how bi! those fra!ments
were. Ae"ertheless, some indication of their si;e can be inferred from spectro!raphic analyses of Kupiter5s outer
atmosphere. (,ter the ,ra!)ets; etr10 these aal1ses re3eale* u"rece*ete* traces o, sul,ur< )he
fra!ments themsel"es almost certainly contained no sulfur, but astronomers belie"e that the cloud layer below
Kupiter5s outer atmosphere does contain sulfur. Since sul,ur +oul* ha3e see"e* ito the outer at)os"here i,
co)et ,ra!)ets ha* "eetrate* this clou* la1er0 it is li%ely that some of the fra!ments were at least lar!e
enou!h to ha"e passed throu!h Kupiter5s outer atmosphere without bein! burned up. In the astronomer5s ar!ument,
the two portions in bol*,ace play which of the followin! roles?
A. )he first is a claim that the astronomer see%s to show is trueI the second ac%nowled!es a consideration that
wei!hs a!ainst the truth of that claim.
B. )he first is a claim that the astronomer see%s to show is trueI the second pro"ides e"idence in support of the
truth of that claim.
3. )he first and the second are each considerations ad"anced in support of the conclusion of the ar!ument.
6. )he first pro"ides e"idence in support of the conclusion of the ar!umentI the second is that conclusion.
#. )he first is a circumstance for which the astronomer see%s to pro"ide an e$planationI the second
ac%nowled!es a consideration that wei!hs a!ainst the e$planation pro"ided by the astronomer.
Critical Reasoi! To"ic 8$ ?iscellaeous
+. If, in a tennis tournament, a match reaches a fifthset tiebrea%, the lowerran%ed player always loses the
tiebrea% Band, therefore, the matchD. If 2afael, the secondran%ed player, wins a tournament by beatin!
2o!er, the topran%ed player, then the match must not ha"e included a fifthset tiebrea%. Which of the
following arguments most closely mimics the reasoning used in the above argument?
If a woman with a family history of twins !ets pre!nant three times, she will ha"e one set of twins. Kennifer,
who falls into this cate!ory, had two sets of twins, so she must not ha"e !otten pre!nant e$actly three times.
If a salesman sells more product than anyone else in a calendar year, then he will earn an alle$pensespaid
"acation. Koe earned an alle$pensepaid "acation, so he must ha"e sold more product than anyone else for
the year.
A newspaper can char!e a 50' premium for ads if its circulation surpasses ,00,000I if the circulation does
not pass ,00,000, therefore, the newspaper can5t char!e any %ind of premium for ads.
If a student is in the top ,0' of her class, she will earn a colle!e scholarship. Anna is not in the top ,0' of
her class, so she will not earn a scholarship.
All of the players on a football team recei"e a cash bonus if the team wins the Super Bowl. If <uarterbac% )om
Brady earned a cash bonus last year, he must ha"e been a member of the winnin! Super Bowl team.
48
2. Studies show that repeated e$posure to an aller!en can tri!!er an aller!ic person5s defense mechanisms to
the e$tent that e"entually, e"en minimal e$posure to the aller!en can pro"o%e a stron!er than normal, e"en
lifethreatenin! reaction. Which of the following is the best analogy for the process by which minimal contact
with an allergen can cause a ma+or reaction?
:eople with unhealthy diets are more susceptible to diabetes and hi!h blood pressure.
Because minor earth<ua%es can wea%en the e$istin! infrastructure in an area, a series of minor earth<ua%es
can sometimes lead to more dama!e than a sin!le ma7or earth<ua%e.
:eanut aller!y sufferers should a"oid all potential contact with peanuts.
A student who fails a test can still earn a !ood !rade if she wor%s hard for the rest of the semester.
6ri"ers who speed re!ularly are more li%ely to be cau!ht than those who speed only occasionally.
0. )he 86oppler effect9 refers to the percei"ed chan!e in pitch that occurs when the source of a sound is in motion
relati"e to the obser"er. 4or e$ample, the siren on a passin! police car will sound hi!her than its true pitch as the
car approaches, sound the same as its true pitch as the car passes, and sound lower than its true pitch as the
car tra"els away from the obser"er. If two trains pass each other going opposite directions on parallel east1
west tracks, which of the following observations provides another illustration of the effect described above?
If the eastbound train blows its horn as they pass, passen!ers on the westbound train will hear a sound that
decreases in pitch.
If the eastbound train blows its horn as they pass, passen!ers on the westbound train will hear a sound that
increases in pitch.
If the eastbound train blows its horn as they pass, passen!ers on the eastbound train will hear a sound that
decreases in pitch.
If the eastbound train blows its horn as they pass, passen!ers on the eastbound train will hear a sound that
increases in pitch.
If the eastbound train blows its horn as they pass, passen!ers on the eastbound train will hear a sound that is
steady in pitch.
4. Ei)$ )he United States (int has recently announced that the risin! cost of raw copper, nic%el, and ;inc has
pushed the cost of manufacturin! each penny and nic%el to more than , cent and 5 cents respecti"ely. In
addition, there are "ery few, if any, items that can be purchased for 5 cents, and "irtually none that can be
purchased for a penny. Since it appears that both the penny and the nic%el no lon!er pro"ide a necessary
function in today5s economy, I propose that the !o"ernment simply stop mintin! the coins at a loss and
declare the base unit of U.S. currency to be the dime.
?ar1$ 6on5t be silly. )his would mean that all prices would round up to the nearest dime and nobody would !o for
that. )here is simply no way that 3on!ress would appro"e, much less consider, such an unpopular measure. In her
response to 7im, 8ary does all of the following ()'(&T*
(a%es an assumption about how Kim5s proposal would affect prices.
Implies that 3on!ressional appro"al is necessary for Kim5s proposal to ta%e effect.
:ro"ides a reason why Kim5s proposal would be difficult to implement, re!ardless of its 7ustification.
Assumes that 3on!ress will ne"er appro"e a policy that has little public support.
Attempts to undermine all or some of the premises upon which Kim 7ustifies his proposal.
5. Ewner of Ka"aKoint* E"er the past year, the coffee store has become a daily han!out for more and more
teena!ers. (any of our adult customers do not appear comfortable with this %ind of crowd and some of them ha"e
told me that they will no lon!er stop here for a coffee drin%. Since my !oal is to ma$imi;e our re"enue, I want you to
discoura!e teena!ers from comin! here and start culti"atin! a more adult crowd. Store mana!er* Are you
sure? En a"era!e, each teena!er spends 7ust as much as the a"era!e adult does, and we ha"e far more new
customers than we ha"e lost o"er the past year. The store manager responds to the owner by
"""""
<uestionin! the "eracity of owner5s e"idence
ar!uin! that it would be difficult to implement the owner5s directi"e
offerin! new e"idence implyin! that the status <uo is not incompatible with the owner5s !oal
demonstratin! that the a"era!e teena!e customer is as profitable as the a"era!e adult customer
offerin! new e"idence refutin! that presented by the owner
49
+. >arba!e in this nei!hborhood probably will not be collected until )hursday this wee%. >arba!e is usually
collected here on Fednesdays, and the !arba!e collectors in this city are e$tremely reliable. &owe"er,
(onday was a public holiday, and after a public holiday that falls on a (onday, !arba!e throu!hout the city is
supposed to be collected one day later than usual. )he ar!ument proceeds by
treatin! se"eral pieces of irrele"ant e"idence as thou!h they pro"ide support for the conclusion
indirectly establishin! that one thin! is li%ely to occur by directly rulin! out all of the alternati"e possibilities
pro"idin! information that allows application of a !eneral rule to a specific case
!enerali;in! about all actions of a certain %ind on the basis of a description of one such action
treatin! somethin! that is probable as thou!h it were ine"itable
.. Kane* :rofessor &arper5s ideas for modifyin! the desi!n of !uitars are of no "alue because there is no !eneral
a!reement amon! musicians as to what a !uitar should sound li%e and, conse<uently, no widely accepted
basis for e"aluatin! the merits of a !uitar5s sound.
(ar%* Fhat5s more, &arper5s ideas ha"e had enou!h time to be adopted if they really resulted in superior sound.
It too% only ten years for the )orres desi!n for !uitars to be almost uni"ersally adopted because of the impro"ement
it ma%es in tonal <uality. Fhich one of the followin! most accurately describes the relationship between Kane5s
ar!ument and (ar%5s ar!ument?
(ar%5s ar!ument shows how a wea%ness in Kane5s ar!ument can be o"ercome.
(ar%5s ar!ument has a premise in common with Kane5s ar!ument.
(ar% and Kane use similar techni<ues to ar!ue for different conclusions.
(ar%5s ar!ument restates Kane5s ar!ument in other terms.
(ar%5s ar!ument and Kane5s ar!ument are based on conflictin! suppositions.
-. Kor!e* Hou won5t be able to write well about the roc% music of the ,@+0s, since you were 7ust an infant then. 2oc%
music of the ,@+0s was created by and for people who were then in their teens and early twenties.
2uth* Hour reasonin! is absurd. )here are li"in! writers who write well about ancient 2oman culture, e"en thou!h
those writers are ob"iously not a part of ancient 2oman culture. Fhy should my youth alone pre"ent me from
writin! well about the music of a period as recent as the ,@+0s? 2uth responds to Kor!e5s criticism by
challen!in! his claim that she was not in her teens or early twenties durin! the ,@+0s
clarifyin! a definition of popular culture that is left implicit in Kor!e5s ar!ument
usin! the e$ample of classical culture in order to le!itimi;e contemporary culture as an ob7ect worthy of
serious consideration
offerin! an analo!y to counter an unstated assumption of Kor!e5s ar!ument
castin! doubt on her opponent5s <ualification to ma%e 7ud!ments about popular culture
@. Anne* &alley5s 3omet, now in a part of its orbit relati"ely far from the Sun, recently flared bri!htly enou!h to
be seen by telescope. Ao comet has e"er been obser"ed to flare so far from the Sun before, so such a flare
must be hi!hly unusual.
Sue* Aonsense. Usually no one bothers to try to obser"e comets when they are so far from the Sun. )his
flare was obser"ed only because an obser"atory was trac%in! &alley5s 3omet "ery carefully. Sue challen!es
Anne5s reasonin! by
pointin! out that Anne5s use of the term 8obser"ed9 is e$cessi"ely "a!ue
drawin! attention to an inconsistency between two of Anne5s claims
presentin! e"idence that directly contradicts Anne5s e"idence
offerin! an alternati"e e$planation for the e"idence Anne cites
underminin! some of Anne5s e"idence while a!reein! with her conclusion
,0. In!rid* 2oc% music has produced no son!s as durable as the son!s of the ,@10s, which continue to be recorded by
numerous performers.
Kerome* )rue, roc% son!s are usually recorded only once. If the ori!inal recordin! continues to be popular, howe"er,
that fact can indicate durability, and the best roc% son!s will pro"e to be durable. Kerome responds to In!rid5s claim
by
intentionally misinterpretin! the claim
showin! that the claim necessarily leads to a contradiction
underminin! the truth of the e"idence that In!rid presents
su!!estin! an alternati"e standard for 7ud!in! the point at issue
claimin! that In!rid5s %nowled!e of the period under discussion is incomplete
50
CR Challe!e Set (The Su"er #00)
,. )he importance of the o;one layer to terrestrial animals is that it entirely filters out some wa"elen!ths of li!ht but
lets others throu!h. &oles in the o;one layer and the dan!ers associated with these holes are well
documented. &owe"er, one dan!er that has not been !i"en sufficient attention is that these holes could lead
to se"ere eye dama!e for animals of many species. Fhich one of the followin! is most stron!ly supported by
the statements abo"e, if they are true?
A. All wa"elen!ths of sunli!ht that can cause eye dama!e are filtered out by the o;one layer where it is intact.
B. 4ew species of animals li"e on a part of the earth5s surface that is not threatened by holes in the o;one layer.
3. Some species of animals ha"e eyes that will not suffer any dama!e when e$posed to unfiltered sunli!ht.
6. A sin!le wa"elen!th of sunli!ht can cause se"ere dama!e to the eyes of most species of animals.
#. Some wa"elen!ths of sunli!ht that cause eye dama!e are more li%ely to reach the earth5s surface where
there are holes in the o;one layer than where there are not.
2. Ene of the most "e$in! problems in historio!raphy is datin! an e"ent when the usual sources offer conflictin!
chronolo!ies of the e"ent. &istorians should attempt to minimi;e the number of competin! sources, perhaps by
eliminatin! the less credible ones. Ence this is achie"ed and se"eral sources are left, as often happens, historians
may try, thou!h on occasion unsuccessfully, to determine independently of the usual sources which date is more
li%ely to be ri!ht. Fhich one of the followin! inferences is most stron!ly supported by the information abo"e?
A. Fe ha"e no plausible chronolo!y of most of the e"ents for which attempts ha"e been made by historians to
determine the ri!ht date.
B. Some of the e"ents for which there are conflictin! chronolo!ies and for which attempts ha"e been made by
historians to determine the ri!ht date cannot be dated reliably by historians.
3. Attachin! a reliable date to any e"ent re<uires determinin! which of se"eral conflictin! chronolo!ies is most
li%ely to be true.
6. 6eterminin! independently of the usual sources which of se"eral conflictin! chronolo!ies is more li%ely to be
ri!ht is an ineffecti"e way of datin! e"ents.
#. )he soundest approach to datin! an e"ent for which the usual sources !i"e conflictin! chronolo!ies is to
undermine the credibility of as many of these sources as possible.
0. In an e$periment, twoyearold boys and their fathers made pie dou!h to!ether usin! rollin! pins and other
utensils. #ach fatherson pair used a rollin! pin that was distincti"ely different from those used by the other
fatherson pairs, and each father repeated the phrase 8rollin! pin9 each time his son used it. But when the
children were as%ed to identify all of the rollin! pins amon! a !roup of %itchen utensils that included se"eral
rollin! pins, each child pic%ed only the one that he had used. Fhich one of the followin! inferences is most
supported by the information abo"e?
A. )he children did not !rasp the function of a rollin! pin.
B. Ao two children understood the name 8rollin! pin9 to apply to the same ob7ect.
3. )he children understood that all rollin! pins ha"e the same !eneral shape.
6. #ach child was able to identify correctly only the utensils that he had used.
#. )he children were not able to distin!uish the rollin! pins they used from other rollin! pins.
1. )he increasin! comple$ity of scientific in<uiry has led to a proliferation of multiauthored technical articles.
2eports of clinical trials in"ol"in! patients from se"eral hospitals are usually coauthored by physicians from
each participatin! hospital. =i%ewise, physics papers reportin! results from e$periments usin! subsystems
de"eloped at "arious laboratories !enerally ha"e authors from each laboratory. If all of the statements abo"e
are true, which one of the followin! must be true?
A. 3linical trials in"ol"in! patients from se"eral hospitals are ne"er conducted solely by physicians from 7ust one
hospital.
B. (ost reports of clinical trials in"ol"in! patients from se"eral hospitals ha"e multiple authors.
3. Fhen a technical article has multiple authors, they are usually from different institutions.
6. :hysics papers authored by researchers from multiple laboratories usually report results from e$periments
usin! subsystems de"eloped at each laboratory.
#. (ost technical articles are authored solely by the researchers who conducted the e$periments these articles
report.
51
5. Some en"ironmentalists <uestion the prudence of e$ploitin! features of the en"ironment, ar!uin! that there
are no economic benefits to be !ained from forests, mountains, or wetlands that no lon!er e$ist. (any
en"ironmentalists claim that because nature has intrinsic "alue it would be wron! to destroy such features of
the en"ironment, e"en if the economic costs of doin! so were outwei!hed by the economic costs of not doin!
so. Fhich one of the followin! can be lo!ically inferred from the passa!e?
A. It is economically imprudent to e$ploit features of the en"ironment.
B. Some en"ironmentalists appeal to a noneconomic 7ustification in <uestionin! the defensibility of e$ploitin!
features of the en"ironment.
3. (ost en"ironmentalists appeal to economic reasons in <uestionin! the defensibility of e$ploitin! features of
the en"ironment.
6. (any en"ironmentalists pro"ide only a noneconomic 7ustification in <uestionin! the defensibility of e$ploitin!
features of the en"ironment.
#. #"en if there is no economic reason for protectin! the en"ironment, there is a sound noneconomic
7ustification for doin! so.
+. Some ar!ue that laws are instituted at least in part to help establish a particular moral fabric in society. But the
primary function of law is surely to help order society so that its institutions, or!ani;ations, and citi;enry can wor%
to!ether harmoniously, re!ardless of any further moral aims of the law. Indeed, the hi!hest courts ha"e on occasion
treated moral beliefs based on conscience or reli!ious faith as !rounds for ma%in! e$ceptions in the application of
laws. )he statements abo"e, if true, most stron!ly support which one of the followin!?
A. )he manner in which laws are applied sometimes ta%es into account the beliefs of the people !o"erned by
those laws.
B. )he law has as one of its functions the orderin! of society but is de"oid of moral aims.
3. Actions based on reli!ious belief or on moral con"iction tend to recei"e the protection of the hi!hest courts.
6. )he way a society is ordered by law should not reflect any moral con"ictions about the way society ou!ht to
be ordered.
#. )he best way to promote cooperation amon! a society5s institutions, or!ani;ations, and citi;enry is to institute
order in that society by means of law.
.. Aewtonian physics dominated science for o"er two centuries. It found consistently successful application,
becomin! one of the most hi!hly substantiated and accepted theories in the history of science. Ae"ertheless,
#instein5s theories came to show the fundamental limits of Aewtonian physics and to surpass the Aewtonian
"iew in the early ,@00s, !i"in! rise once a!ain to a physics that has so far en7oyed wide success. Fhich one
of the followin! lo!ically follows from the statements abo"e?
A. )he history of physics is characteri;ed by a pattern of one successful theory subse<uently surpassed by another.
B. =on!standin! success or substantiation of a theory of physics is no !uarantee that the theory will continue to
be dominant indefinitely.
3. #"ery theory of physics, no matter how successful, is e"entually surpassed by one that is more successful.
6. Ence a theory of physics is accepted, it will remain dominant for centuries.
#. If a lon!accepted theory of physics is surpassed, it must be surpassed by a theory that is e<ually successful.
-. )he solidity of brid!e piers built on pilin!s depends lar!ely on how deep the pilin!s are dri"en. :rior to ,.00,
pilin!s were dri"en to 8refusal,9 that is, to the point at which they refused to !o any deeper. In a ,5-- in<uiry
into the solidity of piers for Oenice5s 2ialto Brid!e, it was determined that the brid!e5s builder, Antonio 6a
:onte, had met the contemporary standard for refusal* he had caused the pilin!s to be dri"en until additional
penetration into the !round was no !reater than two inches after twentyfour hammer blows. Fhich one of the
followin! can properly be inferred from the passa!e?
A. )he 2ialto Brid!e was built on unsafe pilin!s.
B. )he standard of refusal was not sufficient to ensure the safety of a brid!e.
3. 6a :onte5s standard of refusal was less strict than that of other brid!e builders of his day.
6. After ,5--, no brid!es were built on pilin!s that were dri"en to the point of refusal.
#. It is possible that the pilin!s of the 2ialto Brid!e could ha"e been dri"en deeper e"en after the standard of
refusal had been met.
52
@. #"ery moral theory de"eloped in the Festern tradition purports to tell us what a !ood life is. &owe"er, most
people would 7ud!e someone who perfectly embodied the ideals of any one of these theories not to be li"in!
a !ood lifeG the %ind of life they would want for themsel"es and their children. )he statements abo"e, if true,
most stron!ly support which one of the followin!?
A. (ost people desire a life for themsel"es and their children that is better than a merely !ood life.
B. A person who fits the ideals of one moral theory in the Festern tradition would not necessarily fit the ideals of
another.
3. (ost people ha"e a conception of a !ood life that does not match that of any moral theory in the Festern tradition.
6. A !ood life as described by moral theories in the Festern tradition cannot be reali;ed.
#. It is impossible to de"elop a theory that accurately describes what a !ood life is.
,0. (ystery stories often feature a brilliant detecti"e and the detecti"e5s dull companion. 3lues are presented in the
story, and the companion wron!ly infers an inaccurate solution to the mystery usin! the same clues that the
detecti"e uses to deduce the correct solution. )hus, the author5s strate!y of includin! the dull companion
!i"es readers a chance to sol"e the mystery while also di"ertin! them from the correct solution. Fhich one of
the followin! is most stron!ly supported by the information abo"e?
A. (ost mystery stories feature a brilliant detecti"e who sol"es the mystery presented in the story.
B. (ystery readers often sol"e the mystery in a story simply by spottin! the mista%es in the reasonin! of the
detecti"e5s dull companion in that story.
3. Some mystery stories !i"e readers enou!h clues to infer the correct solution to the mystery.
6. )he actions of the brilliant detecti"e in a mystery story rarely di"ert readers from the actions of the detecti"e5s
dull companion.
#. )he detecti"e5s dull companion in a mystery story !enerally unco"ers the misleadin! clues that di"ert readers
from the mystery5s correct solution.
,,. 3S;anne5s art inspired the ne$t !eneration of artists, twentiethcentury modernist creators of abstract art.
Fhile most e$perts ran% 3S;anne as an early modernist, a small few re7ect this idea. 4ranToise 3achin, for
e$ample, bluntly states that such an ascription is 8o"erplayed,9 and says that 3S;anne5s wor% is 8too often
obser"ed from a modern point of "iew.9 Fhich one of the followin! statements is most stron!ly supported by
the information abo"e?
A. 3S;anne5s wor% is hi!hly contro"ersial.
B. 3S;anne was an early creator of abstract art.
3. 3S;anne5s wor% helped to de"elop modernism.
6. (odern art owes less to 3S;anne than many e$perts belie"e.
#. 3S;anne5s wor% tends to be misinterpreted as modernist.
,2. =i!ht is re!istered in the retina when photons hit molecules of the pi!ment rhodopsin and chan!e the
molecules5 shape. #"en when they ha"e not been struc% by photons of li!ht, rhodopsin molecules sometimes
chan!e shape because of normal molecular motion, thereby introducin! error into the "isual system. )he
amount of this molecular motion is directly proportional to the temperature of the retina. Fhich one of the
followin! conclusions is most stron!ly supported by the information abo"e?
A. )he temperature of an animal5s retina depends on the amount of li!ht the retina is absorbin!.
B. )he "isual systems of animals whose body temperature matches that of their surroundin!s are more error
prone in hot surroundin!s than in cold ones.
3. As the temperature of the retina rises, rhodopsin molecules react more slowly to bein! struc% by photons.
6. 2hodopsin molecules are more sensiti"e to photons in animals whose retinas ha"e lar!e surface areas than
in animals whose retinas ha"e small surface areas.
#. (olecules of rhodopsin are the only pi!ment molecules that occur naturally in the retina.
,0. 4la"onoids are a common component of almost all plants, but a specific "ariety of fla"onoid in apples has been
found to be an antio$idant. Antio$idants are %nown to be a factor in the pre"ention of heart disease. Fhich one of
the followin! can be properly inferred from the passa!e?
A. A diet composed lar!ely of fruits and "e!etables will help to pre"ent heart disease.
B. 4la"onoids are essential to pre"entin! heart disease.
3. #atin! at least one apple each day will pre"ent heart disease.
6. At least one type of fla"onoid helps to pre"ent heart disease.
#. A diet deficient in antio$idants is a common cause of heart disease.
53
14. Shar%s ha"e a hi!her ratio of cartila!e mass to body mass than any other or!anism. )hey also ha"e a !reater
resistance to cancer than any other or!anism. Shar% cartila!e contains a substance that inhibits tumor !rowth by
stoppin! the de"elopment of a new blood networ%. In the past 20 years, none of the responses amon! terminal
cancer patients to "arious therapeutic measures has been more positi"e than the response amon! those who
consumed shar% cartila!e. If the claims made above are true, then each of the following could be true
E!E"T#
BAD Ao or!anism resists cancer better than shar%s do, but some resist cancer as well as shar%s.
BBD )he or!anism most susceptible to cancer has a hi!her percenta!e of cartila!e than some or!anisms that are
less susceptible to cancer.
B3D )he substance in shar% cartila!e that inhibits tumor !rowth is found in most or!anisms.
B6D In the past 20 years many terminal cancer patients ha"e impro"ed dramatically followin! many sorts of therapy
B#D Some or!anisms ha"e immune systems more efficient than a shar%5s immune system.
,5. >ood students learn more than what their parents and teachers compel them to learn. )his re<uires that
these students deri"e pleasure from the satisfaction of their curiosity, and one cannot e$perience such
pleasure unless one is capable of concentratin! on a topic so intently that one loses trac% of one5s own
identity. If the statements abo"e are true, each of the followin! could also be true #L3#:)*
BAD Some people who are capable of becomin! so absorbed in a topic that they lose trac% of their own identities
are ne"ertheless incapable of deri"in! pleasure from the satisfaction of their curiosity.
BBD (ost !ood students do not deri"e pleasure from the satisfaction of their curiosity.
B3D (any people who deri"e pleasure simply from the satisfaction of their curiosity are not !ood students.
B6D Some people who are not !ood students deri"e pleasure from losin! trac% of their own identities.
B#D (ost people who are capable of becomin! so absorbed in a topic that they lose trac% of their own identities
are not !ood students.
,+. 4or a tenmonth period, the total monthly sales of new cars within the country of 3alistan remained constant. 6urin! this
period the monthly sales of new cars manufactured by (ar"el Automobile 3ompany doubled, and its share of the new car
mar%et within 3alistan increased correspondin!ly. At the end of this period, emission standards were imposed on new
cars sold within 3alistan. 6urin! the three months followin! this imposition, (ar"el Automobile 3ompany5s share of the
3alistan mar%et declined substantially e"en thou!h its monthly sales within 3alistan remained constant at the le"el
reached in the last month of the tenmonth period. If the statements abo"e are
true, which one of the followin! 3AAAE) be true?
BAD )he total monthly sales within 3alistan of new cars by companies other than (ar"el Automobile 3ompany
decreased o"er the three months followin! the imposition of the emission standards.
BBD E"er the three months before the imposition of the emission standards, the combined mar%et share of
companies other than (ar"el Automobile 3ompany sellin! new cars in 3alistan decreased.
B3D If the emission standards had not been imposed, (ar"el Automobile 3ompany would ha"e lost an e"en lar!er
share of the number of new cars sold in 3alistan than, in fact, it did.
B6D A decrease in the total monthly sales of new cars within 3alistan will occur if the emission standards remain in
effect.
B#D Since the imposition of the emission standards, (ar"el Automobile 3ompany5s a"era!e profit on each new
car sold within 3alistan has increased.
,.. )wo thin!s are true of all immoral actions. 4irst, if they are performed in public, they offend public sensibilities.
Second, they are accompanied by feelin!s of !uilt. If all the statements abo"e are true, then which one of the
followin! must be false?
BAD Some immoral actions that are not performed in public are not accompanied by feelin!s of !uilt.
BBD Immoral actions are wron! solely by "irtue of bein! accompanied by feelin!s of !uilt.
B3D Some actions that offend public sensibilities if they are performed in public are not accompanied by feelin!s
of !uilt.
B6D Some actions that are accompanied by feelin!s of !uilt are not immoral, e"en if they fre<uently offend public
sensibilities.
B#D #"ery action performed in public that is accompanied by feelin!s of !uilt is immoral.
54
,-. Kournalist* A free mar%etplace of ideas ensures that all ideas !et a fair hearin!. #"en ideas tainted with pre7udice
and malice can prompt beneficial outcomes. In most countries, howe"er, the !o"ernment is responsible for o"er
half the information released to the public throu!h all media. 4or this reason, the power of !o"ernments o"er
information needs to be curtailed. #"eryone !rants that !o"ernments should not suppress free e$pression, yet
!o"ernments continue to construct near monopolies on the publication and dissemination of enormous amounts of
information. Fhich one of the followin! most accurately e$presses the conclusion of the 7ournalist5s ar!ument?
A. )he freedom of the mar%etplace of ideas is in 7eopardy.
B. :reser"in! a free mar%etplace of ideas is important.
3. )he control that !o"ernments ha"e o"er information needs to be reduced.
6. Ideas that ha"e malicious content or stem from <uestionable sources can be "aluable.
#. >o"ernments ha"e near monopolies on the dissemination of many %inds of information.
,@. I a!ree that &o!an5s actions resulted in !rie"ous in7ury to Finters. And I do not deny that &o!an fully reali;ed
the nature of his actions and the effects that they would ha"e. Indeed, I would not disa!ree if you pointed out
that intentionally causin! such effects is reprehensible, other thin!s bein! e<ual. But in as%in! you to concur
with me that &o!an5s actions not be wholly condemned I emphasi;e a!ain that &o!an mista%enly belie"ed
Finters to be the robber who had been terrori;in! westside apartment buildin!s for the past se"eral months.
Fhich one of the followin! most accurately e$presses the conclusion of the ar!ument?
A. &o!an should not be considered responsible for the in7uries sustained by Finters.
B. )he robber who had been terrori;in! westside apartment buildin!s should be considered to be as
responsible for Finters5s in7uries as &o!an.
3. )he actions of &o!an that seriously in7ured Finters are not completely blameworthy.
6. &o!an thou!ht that Finters was the person who had been terrori;in! westside apartment buildin!s for the
last few months.
#. )he actions of &o!an that seriously in7ured Finters were reprehensible, other thin!s bein! e<ual.
20. =ast month E34, Inc., announced what it described as a uni<ue new product* an ad7ustable computer
wor%station. )hree days later #r!o)ech un"eiled an almost identical product. )he two companies claim that
the similarities are coincidental and occurred because the desi!ners independently reached the same
solution to the same problem. )he similarities are too fundamental to be mere coincidence, howe"er. )he two
products not only loo% ali%e, but they also wor% ali%e. Both are oddly shaped with identically placed control
panels with the same types of controls. Both allow the same types of ad7ustments and the same types of
optional enhancements. )he main point of the ar!ument is that
A. the two products ha"e many characteristics in common
B. #r!o)ech must ha"e copied the desi!n of its new product from E345s desi!n
3. the similarities between the two products are not coincidental
6. product desi!ners sometimes reach the same solution to a !i"en problem without consultin! each other
#. new products that at first appear to be uni<ue are sometimes simply "ariations of other products
2,. :rediction, the hallmar% of natural sciences, appears to ha"e been possible by reducin! phenomena to
mathematical e$pressions. Some social scientists also want the power to predict accurately and assume they
ou!ht to perform the same reduction. But this would be a mista%eI it would ne!lect data that are not easily
mathemati;ed and thereby would only distort the social phenomena. Fhich one of the followin! most
accurately e$presses the main conclusion of the ar!ument?
A. )he social sciences do not ha"e as much predicti"e power as the natural sciences.
B. (athematics plays a more important role in the natural sciences than it does in the social sciences.
3. )here is a need in the social sciences to impro"e the ability to predict.
6. :henomena in the social sciences should not be reduced to mathematical formulas.
#. :rediction is responsible for the success of the natural sciences.
22. :sycholo!ist* Althou!h studies of youn! children ha"e re"ealed important facts about the influence of the
en"ironment on lan!ua!e ac<uisition, it is clear that one cannot attribute such ac<uisition solely to
en"ironmental influences* innate mechanisms also play a role. So, the most reasonable <uestion that ou!ht to
be studied is whether JJJJJJJ. Fhich one of the followin! most lo!ically completes the passa!e?
A. lan!ua!e ac<uisition can e"er be fully e$plained
B. innate mechanisms are a contributin! factor in lan!ua!e learnin!
3. lan!ua!e ac<uisition is solely the product of innate mechanisms
6. parents and peers are the most important influence on a child5s learnin! of a lan!ua!e
#. innate mechanisms play a more important role in lan!ua!e ac<uisition than a child5s immediate en"ironment
55
20. :eople with serious financial problems are so worried about money that they cannot be happy. )heir misery
ma%es e"eryone close to themGfamily, friends, and collea!uesGunhappy as well. Enly if their financial
problems are sol"ed can they and those around them be happy. Fhich one of the followin! statements can
be properly inferred from the passa!e?
A. Enly serious problems ma%e people unhappy.
B. :eople who sol"e their serious financial problems will be happy.
3. :eople who do not ha"e serious financial problems will be happy.
6. If people are unhappy, they ha"e serious financial problems.
#. If people are happy, they do not ha"e serious financial problems.
21. Some types of or!anisms ori!inated throu!h endosymbiosis, the en!ulfin! of one or!anism by another so that a
part of the former becomes a functionin! part of the latter. An unusual nucleomorph, a structure that contains 6AA
and resembles a cell nucleus, has been disco"ered within a plant %nown as a chlorarachniophyte. )wo "ersions of
a particular !ene ha"e been found in the 6AA of this nucleomorph, and one would e$pect to find only a sin!le
"ersion of this !ene if the nucleomorph were not the remains of an en!ulfed or!anism5s nucleus. Fhich one of the
followin! is most stron!ly supported by the information abo"e?
A. Enly or!anisms of types that ori!inated throu!h endosymbiosis contain nucleomorphs.
B. A nucleomorph within the chlorarachniophyte holds all of the !enetic material of some other or!anism.
3. Aucleomorphs ori!inated when an or!anism endosymbiotically en!ulfed a chlorarachniophyte.
6. )wo or!anisms will not under!o endosymbiosis unless at least one of them contains a nucleomorph.
#. 3hlorarachniophytes emer!ed as the result of two or!anisms ha"in! under!one endosymbiosis.
25. If you ha"e no %eyboardin! s%ills at all, you will not be able to use a computer. And if you are not able to use a
computer, you will not be able to write your essays usin! a word processin! pro!ram. If the statements abo"e
are true, which one of the followin! must be true?
A. If you ha"e some %eyboardin! s%ills, you will be able to write your essays usin! a word processin! pro!ram.
B. If you are not able to write your essays usin! a word processin! pro!ram, you ha"e no %eyboardin! s%ills.
3. If you are able to write your essays usin! a word processin! pro!ram, you ha"e at least some %eyboardin! s%ills.
6. If you are able to use a computer, you will probably be able to write your essays usin! a word processin! pro!ram.
#. If you are not able to write your essays usin! a word processin! pro!ram, you are not able to use a computer.
2+. )he a$is of #arth5s daily rotation is tilted with respect to the plane of its orbit at an an!le of rou!hly 20
de!rees. )hat an!le can be %ept fairly stable only by the !ra"itational influence of #arth5s lar!e, nearby (oon.
Fithout such a stable and moderate a$is tilt, a planet5s climate is too e$treme and unstable to support life.
(ars, for e$ample, has only "ery small moons, tilts at wildly fluctuatin! an!les, and cannot support life. If the
statements abo"e are true, which one of the followin! must also be true on the basis of them?
A. If (ars had a sufficiently lar!e nearby moon, (ars would be able to support life.
B. If #arth5s (oon were to lea"e #arth5s orbit #arth5s climate would be unable to support life.
3. Any planet with a stable, moderate a$is tilt can support life.
6. >ra"itational influences other than moons ha"e little or no effect on the ma!nitude of the tilt an!le of either
#arth5s or (ars5s a$is.
#. Ao planet that has more than one moon can support life.
2.. #ditorialist* 6ri"ers with a lar!e number of demerit points who additionally ha"e been con"icted of a serious
dri"in!related offense should either be sentenced to 7ail or be forced to recei"e dri"er reeducation, since to
do otherwise would be to allow a crime to !o unpunished. Enly if such dri"ers are li%ely to be made more
responsible dri"ers should dri"er reeducation be recommended for them. Unfortunately, it is always almost
impossible to ma%e dri"ers with a lar!e number of demerit points more responsible dri"ers. If the editorialist5s
statements are true, they pro"ide the most support for which one of the followin!?
A. 6ri"ers with a lar!e number of demerit points who ha"e been con"icted of a serious dri"in!related offense
should be sent to 7ail.
B. 6ri"er reeducation offers the best chance of ma%in! dri"ers with a lar!e number of demerit points
responsible dri"ers.
3. 6ri"er reeducation is not a harsh enou!h punishment for anyone con"icted of a serious dri"in!related
offense who has also accumulated a lar!e number of demerit points.
6. 6ri"er reeducation should not be recommended for those who ha"e committed no serious dri"in!related offenses.
#. 6ri"ers with a lar!e number of demerit points but no con"iction for a serious dri"in!related offense should
recei"e dri"er reeducation rather than 7ail.
56
2-. (uscular stren!th is a limited resource, and athletic techni<ues help to use this resource efficiently. Since top
athletes do not differ !reatly from each other in muscular stren!th, it follows that a re<uirement for an athlete
to become a champion is a superior mastery of athletic techni<ues. Fhich one of the followin! most
accurately e$presses the conclusion of the ar!ument?
A. Enly champion athletes ha"e a superior mastery of athletic techni<ues.
B. Superior muscular stren!th is a re<uirement for an athlete to become a champion.
3. Ao athlete can become a champion without a superior mastery of athletic techni<ues.
6. )he differences in muscular stren!th between top athletes are not !reat.
#. Athletic techni<ues help athletes use limited resources efficiently.
2@. If there are any inspired musical performances in the concert, the audience will be treated to a !ood show. But
there will not be a !ood show unless there are sophisticated listeners in the audience, and to be a sophisticated
listener one must understand one5s musical roots. If all of the statements abo"e are true, which one of the
followin! must also be true?
A. If there are no sophisticated listeners in the audience, then there will be no inspired musical performances in
the concert.
B. Ao people who understand their musical roots will be in the audience if the audience will not be treated to a
!ood show.
3. If there will be people in the audience who understand their musical roots, then at least one musical
performance in the concert will be inspired.
6. )he audience will be treated to a !ood show unless there are people in the audience who do not understand
their musical roots.
#. If there are sophisticated listeners in the audience, then there will be inspired musical performances in the
concert.
00. A politician can neither be reelected nor a"oid censure by his or her collea!ues if that politician is %nown to be
in"ol"ed in any serious scandals. Se"eral prominent politicians ha"e 7ust now been shown to be in"ol"ed in a
conspiracy that turned into a serious scandal. )hese politicians will therefore not be reelected. If the
statements abo"e are all true, which one of the followin! statements must also be true?
A. )he prominent politicians cannot escape censure by their collea!ues.
B. If there had been no scandal, the prominent politicians would be reelected.
3. Ao politician is censured unless he or she is %nown to be in"ol"ed in a serious scandal.
6. )he prominent politicians initially benefited from the conspiracy that caused the scandal.
#. Some politicians who are in"ol"ed in scandalous conspiracies a"oid detection and censure.
0,. =eachate is a solution, fre<uently hi!hly contaminated, that de"elops when water permeates a landfill site. If
and only if the landfill5s capacity to hold li<uids is e$ceeded does the leachate escape into the en"ironment,
!enerally in unpredictable <uantities. A method must be found for disposin! of leachate. (ost landfill leachate
is sent directly to sewa!e treatment plants, but not all sewa!e plants are capable of handlin! the hi!hly
contaminated water. Fhich one of the followin! can be inferred from the passa!e?
A. )he ability to predict the "olume of escapin! landfill leachate would help sol"e the disposal problem.
B. If any water permeates a landfill, leachate will escape into the en"ironment.
3. Ao sewa!e treatment plants are capable of handlin! leachate.
6. Some landfill leachate is sent to sewa!e treatment plants that are incapable of handlin! it.
#. If leachate does not escape from a landfill into the en"ironment, then the landfill5s capacity to hold li<uids has
not been e$ceeded.
02. 4rom ,@.0 to ,@-@ total ener!y use in this country increased less than ,0 percent. &owe"er, the use of
electrical ener!y in this country durin! this same period !rew by more than 50 percent, as did the !ross
national productG the total "alue of all !oods and ser"ices produced in the nation. If the statements abo"e
are true, then which one of the followin! must also be true?
A. (ost of the ener!y used in this country in ,@-@ was electrical ener!y.
B. 4rom ,@.0 to ,@-@ there was a decline in the use of ener!y other than electrical ener!y in this country.
3. 4rom ,@.0 to ,@-@ there was an increase in the proportion of ener!y use in this country that consisted of
electrical ener!y use.
6. In ,@-@ electrical ener!y constituted a lar!er proportion of the ener!y used to produce the !ross national
product than did any other form of ener!y.
#. In ,@.0 the electrical ener!y that was produced constituted a smaller proportion of the !ross national product
than did all other forms of ener!y combined.
57
00. )he number of Aorth American children who are obeseGthat is, who ha"e more body fat than do -5 percent
of Aorth American children their a!eGis steadily increasin!, accordin! to four ma7or studies conducted o"er
the past ,5 years. If the findin! reported abo"e is correct, it can be properly concluded that
A. when four ma7or studies all produce similar results, those studies must be accurate
B. Aorth American children ha"e been pro!ressi"ely less physically acti"e o"er the past ,5 years
3. the number of Aorth American children who are not obese increased o"er the past ,5 years
6. o"er the past ,5 years, the number of Aorth American children who are underwei!ht has declined
#. the incidence of obesity in Aorth American children tends to increase as the children !row older
01. 6itrama is a federation made up of three autonomous re!ions* Mor"a, (itro, and >uadar. Under the federal
re"enuesharin! plan, each re!ion recei"es a share of federal re"enues e<ual to the share of the total population of
6itrama residin! in that re!ion, as shown by a yearly population sur"ey. =ast year, the percenta!e of federal
re"enues Mor"a recei"ed for its share decreased somewhat e"en thou!h the population sur"ey on which the
re"enuesharin! was based showed that Mor"a5s population had increased. If the statements abo"e are true,
which one of the followin! must also ha"e been shown by the population sur"ey on which last year5s re"enue
sharin! in 6itrama was based?
A. Ef the three re!ions, Mor"a had the smallest number of residents.
B. )he population of Mor"a !rew by a smaller percenta!e than it did in pre"ious years.
3. )he populations of (itro and >uadar each increased by a percenta!e that e$ceeded the percenta!e by which
the population of Mor"a increased.
6. Ef the three re!ions, Mor"a5s numerical increase in population was the smallest.
#. Mor"a5s population !rew by a smaller percenta!e than did the population of at least one of the other two
autonomous re!ions.
05. Students from outside the pro"ince of (ar%land, who in any !i"en academic year pay twice as much tuition
each as do students from (ar%land, had traditionally accounted for at least twothirds of the enrollment at
3entral (ar%land 3olle!e. E"er the past ,0 years academic standards at the colle!e ha"e risen, and the
proportion of students who are not (ar%landers has dropped to around 10 percent. Fhich one of the followin!
can be properly inferred from the statements abo"e?
A. If it had not been for the hi!h tuition paid by students from outside (ar%land, the colle!e could not ha"e
impro"ed its academic standards o"er the past ,0 years.
B. If academic standards had not risen o"er the past ,0 years, students who are not (ar%landers would still
account for at least twothirds of the colle!e5s enrollment.
3. E"er the past ,0 years, the number of students from (ar%land increased and the number of students from
outside (ar%land decreased.
6. E"er the past ,0 years, academic standards at 3entral (ar%land 3olle!e ha"e risen by more than academic
standards at any other colle!e in (ar%land.
#. If the colle!e5s per capita re"enue from tuition has remained the same, tuition fees ha"e increased o"er the
past ,0 years.
0+. (ost antidepressant dru!s cause wei!ht !ain. Fhile dietin! can help reduce the amount of wei!ht !ained
while ta%in! such antidepressants, some wei!ht !ain is unli%ely to be pre"entable. )he information abo"e
most stron!ly supports which one of the followin!?
A. A physician should not prescribe any antidepressant dru! for a patient if that patient is o"erwei!ht.
B. :eople who are tryin! to lose wei!ht should not as% their doctors for an antidepressant dru!.
3. At least some patients ta%in! antidepressant dru!s !ain wei!ht as a result of ta%in! them.
6. )he wei!ht !ain e$perienced by patients ta%in! antidepressant dru!s should be attributed to lac% of dietin!.
#. All patients ta%in! antidepressant dru!s should diet to maintain their wei!ht.
0.. (aria won this year5s local sailboat race by beatin! Sue, the winner in each of the four pre"ious years. Fe can conclude
from this that (aria trained hard. )he conclusion follows lo!ically if which one of the followin! is assumed?
A. Sue did not train as hard as (aria trained.
B. If (aria trained hard, she would win the sailboat race.
3. (aria could beat a fourtime winner only if she trained hard.
6. If Sue trained hard, she would win the sailboat race.
#. Sue is usually a faster sailboat racer than (aria.
58
0-. If somethin! would ha"e been 7ustifiably re!retted if it had occurred, then it is somethin! that one should not
ha"e desired in the first place. It follows that many for!one pleasures should not ha"e been desired in the first
place. )he conclusion abo"e follows lo!ically if which one of the followin! is assumed?
A. Ene should ne"er re!ret one5s pleasures.
B. 4or!one pleasures that were not desired would not ha"e been 7ustifiably re!retted.
3. #"erythin! that one desires and then re!rets not ha"in! is a for!one pleasure.
6. (any for!one pleasures would ha"e been 7ustifiably re!retted.
#. Aothin! that one should not ha"e desired in the first place fails to be a pleasure.
0@. :sychiatrist* )a%e any "isceral emotion you care to consider. )here are always situations in which it is healthy
to try to e$press that emotion. So, there are always situations in which it is healthy to try to e$press one5s
an!er. )he conclusion of the ar!ument follows lo!ically if which one of the followin! is assumed?
A. An!er is always e$pressible.
B. An!er is a "isceral emotion.
3. Some %inds of emotions are unhealthy to e$press.
6. All emotions that are healthy to e$press are "isceral.
#. An emotion is "isceral only if it is healthy to e$press.
10. (arian Anderson, the famous contralto, did not ta%e success for !ranted. Fe %now this because Anderson
had to stru!!le early in life, and anyone who has to stru!!le early in life is able to %eep a !ood perspecti"e on
the world. )he conclusion of the ar!ument follows lo!ically if which one of the followin! is assumed?
A. Anyone who succeeds ta%es success for !ranted.
B. Anyone who is able to %eep a !ood perspecti"e on the world does not ta%e success for !ranted.
3. Anyone who is able to %eep a !ood perspecti"e on the world has to stru!!le early in life.
6. Anyone who does not ta%e success for !ranted has to stru!!le early in life.
#. Anyone who does not ta%e success for !ranted is able to %eep a !ood perspecti"e on the world.
1,. 3olumnist* Almost anyone can be an e$pert, for there are no official !uidelines determinin! what an e$pert must
%now. Anybody who mana!es to con"ince some people of his or her <ualifications in an areaGwhate"er those may
beGis an e$pert. )he columnist5s conclusion follows lo!ically if which one of the followin! is assumed?
A. Almost anyone can con"ince some people of his or her <ualifications in some area.
B. Some e$perts con"ince e"eryone of their <ualifications in almost e"ery area.
3. 3on"incin! certain people that one is <ualified in an area re<uires that one actually be <ualified in that area.
6. #"ery e$pert has con"inced some people of his or her <ualifications in some area.
#. Some people mana!e to con"ince almost e"eryone of their <ualifications in one or more areas.
12. Oa!ue laws set "a!ue limits on people5s freedom, which ma%es it impossible for them to %now for certain
whether their actions are le!al. )hus, under "a!ue laws people cannot feel secure. )he conclusion follows
lo!ically if which one of the followin! is assumed?
A. :eople can feel secure only if they %now for certain whether their actions are le!al.
B. If people do not %now for certain whether their actions are le!al, then they mi!ht not feel secure.
3. If people %now for certain whether their actions are le!al, they can feel secure.
6. :eople can feel secure if they are !o"erned by laws that are not "a!ue.
#. Enly people who feel secure can %now for certain whether their actions are le!al.
10. Art historian* >reat wor%s of art ha"e often elicited outra!e when first presentedI in #urope, Stra"ins%y5s 2ite
of Sprin! prompted a riot, and (anet5s 6S7euner sur l5herbe elicited outra!e and derision. So, since it is clear
that art is often shoc%in!, we should not hesitate to use public funds to support wor%s of art that many people
find shoc%in!. Fhich one of the followin! is an assumption that the art historian5s ar!ument re<uires in order
for its conclusion to be properly drawn?
A. (ost art is shoc%in!.
B. Stra"ins%y and (anet recei"ed public fundin! for their art.
3. Art used to be more shoc%in! than it currently is.
6. :ublic funds should support art.
#. Anythin! that shoc%s is art.
59
11. In Festern economies, more ener!y is used to operate buildin!s than to operate transportation. (uch of the
decline in ener!y consumption since the oil crisis of ,@.0 is due to more efficient use of ener!y in homes and
offices. Aew buildin! technolo!ies, which ma%e li!htin!, heatin!, and "entilation systems more efficient, ha"e cut
billions of dollars from ener!y bills in the Fest. Since ener!y sa"in!s from these efficiencies sa"e se"eral billion
dollars per year today, we can conclude that 50 to ,00 years from now they will sa"e more than $200 billion per
year Bcalculated in current dollarsD. En which one of the followin! assumptions does the ar!ument rely?
A. )echnolo!y used to ma%e buildin!s ener!y efficient will not become prohibiti"ely e$pensi"e o"er the ne$t century.
B. Another oil crisis will occur in the ne$t 50 to ,00 years.
3. Buildin!s will !radually become a less important consumer of ener!y than transportation.
6. #ner!y bills in the Fest will be $200 billion lower in the ne$t 50 to ,00 years.
#. #ner!yefficient technolo!ies based on new scientific principles will be introduced in the ne$t 50 to ,00 years.
15. #missions from automobiles that burn !asoline and automobiles that burn diesel fuel are threatenin! the
<uality of life on our planet, contaminatin! both urban air and !lobal atmosphere. )herefore, the only effecti"e
way to reduce such emissions is to replace the con"entional diesel fuel and !asoline used in automobiles with
cleanerburnin! fuels, such as methanol, that create fewer emissions. Fhich one of the followin! is an
assumption on which the ar!ument depends?
A. 2educin! the use of automobiles would not be a more effecti"e means to reduce automobile emissions than
the use of methanol.
B. )here is no fuel other than methanol that is cleanerburnin! than both diesel fuel and !asoline.
3. If !i"en a choice of automobile fuels, automobile owners would not select !asoline o"er methanol.
6. Automobile emissions constitute the most serious threat to the !lobal en"ironment.
#. At any !i"en time there is a direct correlation between the le"el of urban air pollution and the le"el of
contamination present in the !lobal atmosphere.
1+. 6octors in Britain ha"e lon! suspected that patients who wear tinted eye!lasses are abnormally prone to
depression and hypochondria. :sycholo!ical tests !i"en there to hospital patients admitted for physical complaints
li%e heart pain and di!esti"e distress confirmed such a relationship. :erhaps people whose relationship to the world
is psycholo!ically painful choose such !lasses to reduce "isual stimulation, which is percei"ed as irritatin!. At any
rate, it can be concluded that when such !lasses are worn, it is because the wearer has a tendency to be
depressed or hypochondriacal. )he ar!ument assumes which one of the followin!?
A. 6epression is not caused in some cases by an or!anic condition of the body.
B. Fearers do not thin% of the tinted !lasses as a means of distancin! themsel"es from other people.
3. 6epression can ha"e many causes, includin! actual conditions about which it is reasonable for anyone to be
depressed.
6. 4or hypochondriacs wearin! tinted !lasses, the !lasses ser"e as a "isual si!nal to others that the wearer5s
health is delicate.
#. )he tintin! does not dim li!ht to the eye enou!h to depress the wearer5s mood substantially.
1.. 3olumnist* A democratic society cannot e$ist unless its citi;ens ha"e established stron! bonds of mutual trust.
Such bonds are formed and stren!thened only by participation in ci"ic or!ani;ations, political parties, and
other !roups outside the family. It is ob"ious then that widespread reliance on mo"ies and electronic media
for entertainment has an inherently corrosi"e effect on democracy. Fhich one of the followin! is an
assumption on which the columnist5s ar!ument depends?
A. Anyone who relies on mo"ies and electronic media for entertainment is unable to form a stron! bond of
mutual trust with a citi;en.
B. 3i"ic or!ani;ations cannot usefully ad"ance their !oals by usin! electronic media.
3. Aewspapers and other forms of print media stren!then, rather than wea%en, democratic institutions.
6. 2elyin! on mo"ies and electronic media for entertainment !enerally ma%es people less li%ely to participate in
!roups outside their families.
#. :eople who rely on mo"ies and electronic media for entertainment are !enerally closer to their families than
are those who do not.
60
1-. Barnes* )he two newest employees at this company ha"e salaries that are too hi!h for the simple tas%s
normally assi!ned to new employees and duties that are too comple$ for ine$perienced wor%ers. &ence, the
salaries and the comple$ity of the duties of these two newest employees should be reduced. Fhich one of
the followin! is an assumption on which Barnes5s ar!ument depends?
A. )he duties of the two newest employees are not less comple$ than any others in the company.
B. It is because of the comple$ duties assi!ned that the two newest employees are bein! paid more than is
usually paid to newly hired employees.
3. )he two newest employees are not e$perienced at their occupations.
6. Barnes was not hired at a hi!herthana"era!e startin! salary.
#. )he salaries of the two newest employees are no hi!her than the salaries that other companies pay for
wor%ers with a similar le"el of e$perience.
1@. )he current pattern of human consumption of resources, in which we rely on nonrenewable resources, for
e$ample metal ore, must e"entually chan!e. Since there is only so much metal ore a"ailable, ultimately we
must either do without or turn to renewable resources to ta%e its place. Fhich one of the followin! is an
assumption re<uired by the ar!ument?
A. )here are renewable resource replacements for all of the nonrenewable resources currently bein! consumed.
B. Fe cannot indefinitely replace e$hausted nonrenewable resources with other nonrenewable resources.
3. A renewable resource cannot be e$hausted by human consumption.
6. 3onsumption of nonrenewable resources will not continue to increase in the near future.
#. Ultimately we cannot do without nonrenewable resources.
50. In humans, in!ested protein is bro%en down into amino acids, all of which must compete to enter the brain.
Subse<uent in!estion of su!ars leads to the production of insulin, a hormone that brea%s down the su!ars
and also rids the bloodstream of residual amino acids, e$cept for tryptophan. )ryptophan then slips into the
brain uncontested and is transformed into the chemical serotonin, increasin! the brain5s serotonin le"el. )hus,
su!ars can play a ma7or role in mood ele"ation, helpin! one to feel rela$ed and an$ietyfree. Fhich one of the
followin! is an assumption on which the ar!ument depends?
A. #le"ation of mood and freedom from an$iety re<uire increasin! the le"el of serotonin in the brain.
B. 4ailure to consume foods rich in su!ars results in an$iety and a lowerin! of mood.
3. Serotonin can be produced naturally only if tryptophan is present in the bloodstream.
6. Increasin! the le"el of serotonin in the brain promotes rela$ation and freedom from an$iety.
#. )he consumption of proteinrich foods results in an$iety and a lowerin! of mood.
5,. :ublicity campai!ns for endan!ered species are unli%ely to ha"e much impact on the most important en"ironmental
problems, for while the ease of attributin! feelin!s to lar!e mammals facilitates e"o%in! sympathy for them, it is
more difficult to elicit sympathy for other %inds of or!anisms, such as the soil microor!anisms on which lar!e
ecosystems and a!riculture depend. Fhich one of the followin! is an assumption on which the ar!ument depends?
A. )he most important en"ironmental problems in"ol"e endan!ered species other than lar!e mammals.
B. (icroor!anisms cannot e$perience pain or ha"e other feelin!s.
3. :ublicity campai!ns for the en"ironment are the most effecti"e when they elicit sympathy for some or!anism.
6. :eople i!nore en"ironmental problems unless they belie"e the problems will affect creatures with which they
sympathi;e.
#. An or!anism can be en"ironmentally si!nificant only if it affects lar!e ecosystems or a!riculture.
52. &istorian* =eibni;, the se"enteenthcentury philosopher, published his "ersion of calculus before Aewton did.
But then Aewton re"ealed his pri"ate noteboo%s, which showed he had been usin! these ideas for at least a
decade before =eibni;5s publication. Aewton also claimed that he had disclosed these ideas to =eibni; in a
letter shortly before =eibni;5s publication. Het close e$amination of the letter shows that Aewton5s few cryptic
remar%s did not re"eal anythin! important about calculus. )hus, =eibni; and Aewton each independently
disco"ered calculus. Fhich one of the followin! is an assumption re<uired by the historian5s ar!ument?
A. =eibni; did not tell anyone about calculus prior to publishin! his "ersion of it.
B. Ao third person independently disco"ered calculus prior to Aewton and =eibni;.
3. Aewton belie"ed that =eibni; was able to learn somethin! important about calculus from his letter to him.
6. Aeither Aewton nor =eibni; %new that the other had de"eloped a "ersion of calculus prior to =eibni;5s publication.
#. Aeither Aewton nor =eibni; learned crucial details about calculus from some third source.
61
50. In ,@-0, 3ountry A had a per capita !ross domestic product B>6:D that was $5,000 hi!her than that of the
#uropean #conomic 3ommunity. By ,@@0, the difference, when ad7usted for inflation, had increased to
$+,000. Since a risin! per capita >6: indicates a risin! a"era!e standard of li"in!, the a"era!e standard of
li"in! in 3ountry A must ha"e risen between ,@-0 and ,@@0. Fhich one of the followin! is an assumption on
which the ar!ument depends?
A. Between ,@-0 and ,@@0, 3ountry A and the #uropean #conomic 3ommunity e$perienced the same
percenta!e increase in population.
B. Between ,@-0 and ,@@0, the a"era!e standard of li"in! in the #uropean #conomic 3ommunity fell.
3. Some member countries of the #uropean #conomic 3ommunity had, durin! the ,@-0s, a hi!her a"era!e
standard of li"in! than 3ountry A.
6. )he per capita >6: of the #uropean #conomic 3ommunity was not lower by more than $,,000 in ,@@0 than
it had been in ,@-0.
#. In ,@@0, no member country of the #uropean #conomic 3ommunity had a per capita >6: hi!her than that of
3ountry A.
51. 3arl is clearly an incompetent detecti"e. &e has sol"ed a smaller percenta!e of the cases assi!ned to him in
the last 0 yearsGonly , out of 25Gthan any other detecti"e on the police force. Fhich one of the followin!, if
true, most seriously wea%ens the ar!ument abo"e?
A. Because the police chief re!ards 3arl as the most capable detecti"e, she assi!ns him only the most difficult
cases, ones that others ha"e failed to sol"e.
B. Before he became a detecti"e, 3arl was a nei!hborhood police officer and was hi!hly respected by the
residents of the nei!hborhood he patrolled.
3. 6etecti"es on the police force on which 3arl ser"es are pro"ided with e$tensi"e resources, includin! the use
of a lar!e computer database, to help them sol"e crimes.
6. 3arl was pre"iously a detecti"e in a police department in another city, and in the 1 years he spent there, he
sol"ed only , out of 00 crimes.
#. (any of the officers in the police department in which 3arl ser"es were hired or promoted within the last 5 years.
55. Be"era!e company representati"e* )he plastic rin!s that hold si$pac%s of be"era!e cans to!ether pose a threat to
wild animals, which often become entan!led in the discarded rin!s and suffocate as a result. 4ollowin! our lead, all
be"era!e companies will soon use only those rin!s consistin! of a new plastic that disinte!rates after only three
days5 e$posure to sunli!ht. Ence we all complete the switcho"er from the old to the new plastic rin!s,
therefore, the threat of suffocation that plastic rin!s pose to wild animals will be eliminated. Fhich one of the
followin!, if true, most seriously wea%ens the representati"e5s ar!ument?
A. )he switcho"er to the new plastic rin!s will ta%e at least two more years to complete.
B. After the be"era!e companies ha"e switched o"er to the new plastic rin!s, a substantial number of the old
plastic rin!s will persist in most a<uatic and woodland en"ironments.
3. )he new plastic rin!s are sli!htly less e$pensi"e than the old rin!s.
6. )he new plastic rin!s rarely disinte!rate durin! shippin! of be"era!e si$pac%s because most truc%s that
transport canned be"era!es protect their car!o from sunli!ht.
#. )he new plastic rin!s disinte!rate into substances that are harmful to a<uatic animals when in!ested in
substantial <uantities by them.
5+. )here is relati"ely little room for !rowth in the o"erall carpet mar%et, which is tied to the si;e of the population.
(ost who purchase carpet do so only once or twice, first in their twenties or thirties, and then perhaps a!ain
in their fifties or si$ties. )hus as the population a!es, companies producin! carpet will be able to !ain mar%et
share in the carpet mar%et only throu!h purchasin! competitors, and not throu!h more a!!ressi"e mar%etin!.
Fhich one of the followin!, if true, casts the most doubt on the conclusion abo"e?
A. (ost of the ma7or carpet producers mar%et other floor co"erin!s as well.
B. (ost established carpet producers mar%et se"eral different brand names and "arieties, and there is no
remainin! niche in the mar%et for new brands to fill.
3. )wo of the three mer!ers in the industry5s last ten years led to a decline in profits and re"enues for the newly
mer!ed companies.
6. :rice reductions, achie"ed by costcuttin! in production, by some of the dominant firms in the carpet mar%et
are causin! other producers to lea"e the mar%et alto!ether.
#. )he carpet mar%et is unli%e most mar%ets in that consumers are becomin! increasin!ly resistant to new
patterns and styles.
62
5.. Spea%er* 3ontemporary business firms need to reco!ni;e that a"oidin! social responsibility leads to the
!radual erosion of power. )his is 6a"is and Blomstrom5s Iron =aw of 2esponsibility* 8In the lon! run, those
who do not use power in a manner which society considers responsible will tend to lose it.9 )he law5s
application to human institutions certainly stands confirmed by history. )hou!h the 8lon! run9 may re<uire
decades or e"en centuries in some instances, society ultimately acts to reduce power when society thin%s it is
not bein! used responsibly. )herefore, a business that wishes to retain its power as lon! as it can, must act
responsibly. Fhich one of the followin! statements, if true, most wea%ens the spea%er5s ar!ument?
A. >o"ernment institutions are as sub7ect to the Iron =aw of 2esponsibility as business institutions.
B. :ublic relations pro!rams can cause society to consider an institution socially responsible e"en when it is not.
3. )he power of some institutions erodes more slowly than the power of others, whether they are socially
responsible or not.
6. Since no institution is eternal, e"ery business will e"entually fail.
#. Some businesses that ha"e used power in socially responsible ways ha"e lost it.
5-. :olitician* All nations that place a hi!h ta$ on income produce thereby a ne!ati"e incenti"e for technolo!ical
inno"ation, and all nations in which technolo!ical inno"ation is hampered ine"itably fall behind in the
international arms race. )hose nations that, throu!h historical accident or the foolishness of their political
leadership, wind up in a strate!ically disad"anta!eous position are destined to lose their "oice in world affairs.
So if a nation wants to maintain its "alue system and way of life, it must not allow its hi!hest ta$ brac%et to
e$ceed 00 percent of income. #ach of the followin!, if true, wea%ens the politician5s ar!ument #L3#:)*
A. )he top le"el of ta$ation must reach 15 percent before ta$ation be!ins to deter in"entors and industrialists
from introducin! new technolo!ies and industries.
B. (a%in! a !reat deal of money is an insi!nificant factor in dri"in! technolo!ical inno"ation.
3. 4allin! behind in the international arms race does not necessarily lead to a strate!ically less ad"anta!eous position.
6. )hose nations that lose influence in the world community do not necessarily suffer from a threat to their "alue
system or way of life.
#. Allowin! one5s country to lose its technolo!ical ed!e, especially as concerns weaponry, would be foolish
rather than merely a historical accident.
5@. &uman bein!s ha"e co!niti"e faculties that are superior to those of other animals, and once humans become
aware of these, they cannot be made happy by anythin! that does not in"ol"e !ratification of these faculties. Fhich
one of the followin! statements, if true, most calls into <uestion the "iew abo"e?
A. 3ertain animalsGdolphins and chimpan;ees, for e$ampleGappear to be capable of rational communication.
B. (any people familiar both with intellectual stimulation and with physical pleasures en7oy the latter more.
3. Someone who ne"er e$perienced classical music as a child will usually prefer popular music as an adult.
6. (any people who are serious athletes consider themsel"es to be happy.
#. (any people who are serious athletes lo"e !ourmet food.
+0. =o!!erhead turtles li"e and breed in distinct !roups, of which some are in the :acific Ecean and some are in the
Atlantic. Aew e"idence su!!ests that 7u"enile :acific lo!!erheads that feed near the Ba7a peninsula hatch in
Kapanese waters ,0,000 %ilometers away. Ainetyfi"e percent of the 6AA samples ta%en from the Ba7a turtles
match those ta%en from turtles at the Kapanese nestin! sites. Fhich one of the followin!, if true, most
seriously wea%ens the reasonin! abo"e?
A. Aestin! sites of lo!!erhead turtles ha"e been found off the :acific coast of Aorth America se"eral thousand
%ilometers north of the Ba7a peninsula.
B. )he distance between nestin! sites and feedin! sites of Atlantic lo!!erhead turtles is less than 5,000 %ilometers.
3. =o!!erhead hatchlin!s in Kapanese waters ha"e been declinin! in number for the last decade while the
number of nestin! sites near the Ba7a peninsula has remained constant.
6. Ainetyfi"e percent of the 6AA samples ta%en from the Ba7a turtles match those ta%en from Atlantic
lo!!erhead turtles.
#. 3ommercial a<uariums ha"e been successfully breedin! Atlantic lo!!erheads with :acific lo!!erheads for the
last fi"e years.
63
+,. :eople who ha"e speciali;ed %nowled!e about a scientific or technical issue are systematically e$cluded from
7uries for trials where the issue is rele"ant. )hus, trial by 7ury is not a fair means of settlin! disputes in"ol"in!
such issues. Fhich one of the followin!, if true, most seriously wea%ens the ar!ument?
A. )he more complicated the issue bein! liti!ated, the less li%ely it is that a 7uror without speciali;ed %nowled!e
of the field in"ol"ed will be able to comprehend the testimony bein! !i"en.
B. )he more a 7uror %nows about a particular scientific or technical issue in"ol"ed in a trial, the more li%ely it is
that the 7uror will be pre7udiced in fa"or of one of the liti!atin! parties before the trial be!ins.
3. Appointin! an impartial arbitrator is not a fair means of settlin! disputes in"ol"in! scientific or technical issues,
because arbitrators tend to fa"or settlements in which both parties compromise on the issues.
6. #$perts who !i"e testimony on scientific or technical issues tend to hed!e their conclusions by discussin! the
possibility of error.
#. #$pert witnesses in speciali;ed fields often command fees that are so hi!h that many people in"ol"ed in
liti!ation cannot afford their ser"ices.
+2. )he fi"e senses ha"e traditionally been "iewed as distinct yet complementary. #ach sense is thou!ht to ha"e
its own ran!e of stimuli that are incapable of stimulatin! the other senses. &owe"er, recent research has
disco"ered that some people taste a banana and claim that they are tastin! blue, or see a color and say that
it has a specific smell. )his shows that such people, called synesthesiacs, ha"e senses that do not respect
the usual boundaries between the fi"e reco!ni;ed senses. Fhich one of the followin! statements, if true,
most seriously wea%ens the ar!ument?
A. Synesthesiacs demonstrate a !eneral, systematic impairment in their ability to use and understand words.
B. 2ecent e"idence stron!ly su!!ests that there are other senses besides si!ht, touch, smell, hearin!, and taste.
3. )he particular ways in which sensory e$periences o"erlap in synesthesiacs follow a definite pattern.
6. )he synesthetic phenomenon has been described in the le!ends of "arious cultures.
#. Synesthesiacs can be temporarily rid of their synesthetic e$periences by the use of dru!s.
+0. Archaeolo!ist* A s%eleton of a Aorth American mastodon that became e$tinct at the pea% of the Ice A!e was
recently disco"ered. It contains a humanmade pro7ectile dissimilar to any found in that part of #urasia closest
to Aorth America. )hus, since #urasians did not settle in Aorth America until shortly before the pea% of the Ice
A!e, the first #urasian settlers in Aorth America probably came from a more distant part of #urasia. Fhich
one of the followin!, if true, most seriously wea%ens the archaeolo!ist5s ar!ument?
A. )he pro7ectile found in the mastodon does not resemble any that were used in #urasia before or durin! the
Ice A!e.
B. )he people who occupied the #urasian area closest to Aorth America remained nomadic throu!hout the Ice A!e.
3. )he s%eleton of a bear from the same place and time as the mastodon s%eleton contains a similar pro7ectile.
6. Ether Aorth American artifacts from the pea% of the Ice A!e are similar to ones from the same time found in
more distant parts of #urasia.
#. 3limatic conditions in Aorth America 7ust before the Ice A!e were more conduci"e to human habitation than
were those in the part of #urasia closest to Aorth America at that time.
+1. =obsters and other crustaceans eaten by humans are more li%ely to contract !ill diseases when sewa!e
contaminates their water. Under a recent proposal, millions of !allons of local sewa!e each day would be
rerouted many %ilometers offshore. Althou!h this would substantially reduce the amount of sewa!e in the
harbor where lobsters are cau!ht, the proposal is pointless, because hardly any lobsters li"e lon! enou!h to
be harmed by those diseases. Fhich one of the followin!, if true, most seriously wea%ens the ar!ument?
A. 3ontaminants in the harbor other than sewa!e are e<ually harmful to lobsters.
B. =obsters, li%e other crustaceans, li"e lon!er in the open ocean than in industrial harbors.
3. =obsters breed as readily in sewa!e contaminated water as in unpolluted water.
6. >ill diseases cannot be detected by e$aminin! the surface of the lobster.
#. &umans often become ill as a result of eatin! lobsters with !ill diseases.
64
+5. :eople with hi!h blood pressure are !enerally more ner"ous and an$ious than people who do not ha"e hi!h
blood pressure. )his fact show that this particular combination of personality traitsGthe socalled
hypertensi"e personalityG is li%ely to cause a person with these traits to de"elop hi!h blood pressure. )he
reasonin! in the ar!ument is most "ulnerable to criticism on the !rounds that the ar!ument
A. fails to define the term 8hypertensi"e personality9
B. presupposes that people ha"e permanent personality traits
3. simply restates the claim that there is a 8hypertensi"e personality9 without pro"idin! e"idence to support that
claim.
6. ta%es a correlation between personality traits and hi!h blood pressure as proof that the traits cause hi!h
blood pressure.
#. focuses on ner"ousness and an$iety only, i!norin! other personality traits that people with hi!h blood
pressure mi!ht ha"e
++. &i!h school students who feel that they are not succeedin! in hi!h school often drop out before !raduatin!
and !o to wor%. =ast year, howe"er, the city5s hi!h school dropout rate was si!nificantly lower than the
pre"ious year5s rate. )his is encoura!in! e"idence that the pro!ram instituted two years a!o to impro"e the
morale of hi!h school students has be!un to ta%e effect to reduce dropouts. Fhich one of the followin!, if true
about the last year, most seriously wea%ens the ar!ument?
A. )here was a recession that caused a hi!h le"el of unemployment in the city.
B. )he morale of students who dropped out of hi!h school had been low e"en before they reached hi!h school.
3. As in the precedin! year, more hi!h school students remained in school than dropped out.
6. &i!h schools in the city established placement offices to assist their !raduates in obtainin! employment.
#. )he antidropout pro!ram was primarily aimed at impro"in! students5 morale in those hi!h schools with the
hi!hest dropout rates.
+.. )he number of airplanes e<uipped with a new anticollision de"ice has increased steadily durin! the past two
years. 6urin! the same period, it has become increasin!ly common for %ey information about an airplane5s
altitude and speed to disappear suddenly from air traffic controllers5 screens. )he new anticollision de"ice,
which operates at the same fre<uency as air traffic radar, is therefore responsible for the sudden
disappearance of %ey information. Fhich one of the followin!, if true, most seriously wea%ens the ar!ument?
A. )he new anticollision de"ice has already pre"ented a considerable number of midair collisions.
B. It was not until the new anticollision de"ice was introduced that %ey information first be!an disappearin!
suddenly from controllers5 screens.
3. )he new anticollision de"ice is scheduled to be mo"ed to a different fre<uency within the ne$t two to three
months.
6. Mey information be!an disappearin! from controllers5 screens three months before the new anticollision
de"ice was first tested.
#. )he sudden disappearance of %ey information from controllers5 screens has occurred only at relati"ely lar!e
airports.
+-. Oiolent crime in this town is a becomin! a serious problem. 3ompared to last year, local law enforcement
a!encies ha"e responded to ,. percent more calls in"ol"in! "iolent crimes, showin! that the a"era!e citi;en
of this town is more li%ely than e"er to become a "ictim of a "iolent crime. Fhich one of the followin!, if true,
most seriously wea%ens the ar!ument?
A. )he town5s o"erall crime rate appears to ha"e risen sli!htly this year compared to the same period last year.
B. In !eneral, persons under the a!e of +5 are less li%ely to be "ictims of "iolent crimes than persons o"er the
a!e of +5.
3. As a result of the town5s community outreach pro!rams, more people than e"er are willin! to report "iolent
crimes to the proper authorities.
6. In response to worries about "iolent crime, the town has recently opened a community center pro"idin!
super"ised acti"ities for teena!ers.
#. 3ommunity officials ha"e shown that a relati"ely small number of repeat offenders commit the ma7ority of
"iolent crimes in the town.
65
+@. (edical researcher* As e$pected, records co"erin! the last four years of ten ma7or hospitals indicate that
babies born prematurely were more li%ely to ha"e low birth wei!hts and to suffer from health problems than
were babies not born prematurely. )hese records also indicate that mothers who had recei"ed ade<uate
prenatal care were less li%ely to ha"e low birth wei!ht babies than were mothers who had recei"ed
inade<uate prenatal care. Ade<uate prenatal care, therefore, si!nificantly decreases the ris% of low birth
wei!ht babies. Fhich one of the followin!, if true, most wea%ens the medical researcher5s ar!ument?
A. )he hospital records indicate that many babies that are born with normal birth wei!hts are born to mothers
who had inade<uate prenatal care.
B. (others !i"in! birth prematurely are routinely classified by hospitals as ha"in! recei"ed inade<uate prenatal
care when the record of that care is not a"ailable.
3. )he hospital records indicate that low birth wei!ht babies were routinely classified as ha"in! been born prematurely.
6. Some babies not born prematurely, whose mothers recei"ed ade<uate prenatal care, ha"e low birth wei!hts.
#. Fomen who recei"e ade<uate prenatal care are less li%ely to !i"e birth prematurely than are women who do
not recei"e ade<uate prenatal care.
.0. 2esearcher* :eople with certain personality disorders ha"e more theta brain wa"es than those without such
disorders. But my data show that the amount of one5s theta brain wa"es increases while watchin! )O. So
watchin! too much )O increases one5s ris% of de"elopin! personality disorders. A <uestionable aspect of the
reasonin! abo"e is that it
A. uses the phrase 8personality disorders9 ambi!uously
B. fails to define the phrase 8theta brain wa"es9
3. ta%es a correlation to imply a causal connection
6. draws a conclusion from an unrepresentati"e sample of data
#. infers that watchin! )O is a conse<uence of a personality disorder
.,. Unli%e newspapers in the old days, today5s newspapers and tele"ised news pro!rams are full of stories about
murders and assaults in our city. Ene can only conclude from this chan!e that "iolent crime is now out of
control, and, to be safe from personal attac%, one should not lea"e one5s home e$cept for absolute
necessities. Fhich one of the followin!, if true, would cast the most serious doubt on the conclusion?
A. Aewspapers and tele"ised news pro!rams ha"e more comprehensi"e co"era!e of "iolent crime than
newspapers did in the old days.
B. Aational data show that "iolent crime is out of control e"erywhere, not 7ust in the author5s city.
3. :olice records show that people e$perience more "iolent crimes in their own nei!hborhoods than they do
outside their nei!hborhoods.
6. (urder comprised a lar!er proportion of "iolent crimes in the old days than it does today.
#. Aews ma!a;ines play a more important role today in informin! the public about crime than they did in the old days.
.2. Faste mana!ement companies, which collect waste for disposal in landfills and incineration plants, report
that disposable plastics ma%e up an e"erincreasin! percenta!e of the waste they handle. It is clear that
attempts to decrease the amount of plastic that people throw away in the !arba!e are failin!. Fhich one of
the followin!, if true, most seriously wea%ens the ar!ument?
A. Because plastics create harmful pollutants when burned, an increasin! percenta!e of the plastics handled by
waste mana!ement companies are bein! disposed of in landfills.
B. Althou!h many plastics are recyclable, most of the plastics disposed of by waste mana!ement companies are not.
3. :eople are more li%ely to sa"e and reuse plastic containers than containers made of hea"ier materials li%e
!lass or metal.
6. An increasin! proportion of the paper, !lass, and metal cans that waste mana!ement companies used to
handle is now bein! recycled.
#. Fhile the percenta!e of products usin! plastic pac%a!in! is increasin!, the total amount of plastic bein!
manufactured has remained unchan!ed.
66
.0. 2umored declines in automobileindustry re"enues are e$a!!erated. It is true that automobile manufacturers5
share of the industry5s re"enues fell from +5 percent two years a!o to 50 percent today, but o"er the same
period suppliers of automobile parts had their share increase from ,5 percent to 20 percent and ser"ice
companies Bfor e$ample, distributors, dealers, and repairersD had their share increase from 20 percent to 00
percent. Fhich one of the followin! best indicates why the statistics !i"en abo"e pro"ide by themsel"es no
e"idence for the conclusion they are intended to support?
A. )he possibility is left open that the statistics for manufacturers5 share of re"enues come from a different
source than the other statistics.
B. Ao matter what chan!es the automobile industry5s o"erall re"enues under!o, the total of all shares of these
re"enues must be ,00 percent.
3. Ao e$planation is !i"en for why the re"enue shares of different sectors of the industry chan!ed.
6. (anufacturers and parts companies depend for their re"enue on dealers5 success in sellin! cars.
#. 2e"enues are an important factor but are not the only factor in determinin! profits.
.1. :olitician* )hose economists who claim that consumer price increases ha"e a"era!ed less than 0 percent
o"er the last year are mista%en. )hey clearly ha"e not shopped anywhere recently. >asoline is up ,0 percent
o"er the last yearI my auto insurance, ,2 percentI newspapers, ,5 percentI propane, ,0'I bread, 50 percent.
)he reasonin! in the politician5s ar!ument is most "ulnerable to criticism on the !rounds that the ar!ument
A. impu!ns the character of the economists rather than addressin! their ar!uments
B. fails to show that the economists mentioned are not e$perts in the area of consumer prices
3. mista%enly infers that somethin! is not true from the claim that it has not been shown to be so
6. uses e"idence drawn from a small sample that may well be unrepresentati"e
#. attempts to persuade by ma%in! an emotional appeal
.5. #ditorial* )he premier5s economic ad"isor assures her that with the elimination of wasteful spendin! the !oal of
reducin! ta$es while not si!nificantly decreasin! !o"ernment ser"ices can be met. But the premier should not
listen to this ad"isor, who in his youth was con"icted of embe;;lement. Surely his economic ad"ice is as
untrustworthy as he is himself, and so the premier should discard any hope of reducin! ta$es without a
si!nificant decrease in !o"ernment ser"ices. Fhich one of the followin! is a <uestionable ar!umentati"e
strate!y employed in the editorial5s ar!ument?
A. re7ectin! a proposal on the !rounds that a particular implementation of the proposal is li%ely to fail
B. tryin! to win support for a proposal by playin! on people5s fears of what could happen otherwise
3. critici;in! the source of a claim rather than e$aminin! the claim itself
6. ta%in! a lac% of e"idence for a claim as e"idence underminin! the claim
#. presupposin! what it sets out to establish
.+. 3otrell is, at best, able to write ma!a;ine articles of a"era!e <uality. )he most compellin! pieces of e"idence
for this are those few of the numerous articles submitted by 3otrell that are superior, since 3otrell, who is
incapable of writin! an article that is better than a"era!e, must ob"iously ha"e pla!iari;ed superior ones. )he
ar!ument is most "ulnerable to criticism on which one of the followin! !rounds?
A. It simply i!nores the e$istence of potential countere"idence.
B. It !enerali;es from atypical occurrences.
3. It presupposes what it see%s to establish.
6. It relies on the 7ud!ment of e$perts in a matter to which their e$pertise is irrele"ant.
#. It infers limits on ability from a few isolated lapses in performance.
... Acti"ist* 4ood producers irradiate food in order to prolon! its shelf life. 4i"e animal studies were recently conducted
to in"esti!ate whether this process alters food in a way that could be dan!erous to people who eat it. )he studies
concluded that irradiated food is safe for humans to eat. &owe"er, because these studies were subse<uently found
by a panel of independent scientists to be seriously flawed in their methodolo!y, it follows that irradiated food is not
safe for human consumption. )he reasonin! in the acti"ist5s ar!ument is flawed because that ar!ument
A. treats a failure to pro"e a claim as constitutin! proof of the denial of that claim
B. treats methodolo!ical flaws in past studies as proof that it is currently not possible to de"ise methodolo!ically
ade<uate alternati"es
3. fails to consider the possibility that e"en a study whose methodolo!y has no serious flaws nonetheless mi!ht
pro"ide only wea% support for its conclusion
6. fails to consider the possibility that what is safe for animals mi!ht not always be safe for human bein!s
#. fails to establish that the independent scientists %now more about food irradiation than do the people who
produced the fi"e studies
67
.-. :hilosopher* Scientists tal% about the pursuit of truth, but, li%e most people, they are selfinterested.
Accordin!ly, the professional acti"ities of most scientists are directed toward personal career enhancement,
and only incidentally toward the pursuit of truth. &ence, the acti"ities of the scientific community are lar!ely
directed toward enhancin! the status of that community as a whole, and only incidentally toward the pursuit of
truth. )he reasonin! in the philosopher5s ar!ument is flawed because the ar!ument
A. improperly infers that each and e"ery scientist has a certain characteristic from the premise that most
scientists ha"e that characteristic
B. improperly draws an inference about the scientific community as a whole from a premise about indi"idual scientists
3. presumes, without !i"in! 7ustification, that the aim of personal career enhancement ne"er ad"ances the
pursuit of truth
6. illicitly ta%es ad"anta!e of an ambi!uity in the meanin! of 8selfinterested9
#. improperly draws an inference about a cause from premises about its effects
.@. Se"eral le!islators claim that the public finds many current mo"ies so "iolent as to be morally offensi"e. &owe"er,
these le!islators ha"e misrepresented public opinion. In a sur"ey conducted by a mo"ie industry !uild, only ,.
percent of respondents thou!ht that mo"ies are o"erly "iolent, and only 0 percent found any recent mo"ie morally
offensi"e. )hese low percenta!es are tellin!, because the respondents see far more current mo"ies than does the
a"era!e mo"ie!oer. )he reasonin! in the ar!ument is flawed in that the ar!ument
A. attempts to undermine the le!islators5 credibility instead of addressin! their ar!ument
B. bases its conclusion on sub7ecti"e 7ud!ments rather than on an ob7ecti"e criterion of moral offensi"eness
3. fails to consider the possibility that "iolent mo"ies increase the pre"alence of antisocial beha"ior
6. !enerali;es from a sample that is unli%ely to be representati"e of public sentiment
#. presumes, without pro"idin! 7ustification, that the people sur"eyed based their responses on a random
samplin! of mo"ies
-0. En some hot days the smo! in &ill"iew reaches unsafe le"els, and on some hot days the wind blows into
&ill"iew from the east. )herefore, on some days when the wind blows into &ill"iew from the east, the smo! in
&ill"iew reaches unsafe le"els. )he reasonin! in the ar!ument is flawed in that the ar!ument
A. mista%es a condition that sometimes accompanies unsafe le"els of smo! for a condition that necessarily
accompanies unsafe le"els of smo!
B. fails to reco!ni;e that one set mi!ht ha"e some members in common with each of two others e"en thou!h
those two other sets ha"e no members in common with each other
3. uses the %ey term 8unsafe9 in one sense in a premise and in another sense in the conclusion
6. contains a premise that is implausible unless the conclusion is presumed to be true
#. infers a particular causal relation from a correlation that could be e$plained in a "ariety of other ways
-,. Astronomer* I ha"e asserted that our solar system does not contain enou!h meteoroids and other cosmic
debris to ha"e caused the e$tensi"e craterin! on the far side of the moon. (y opponents ha"e repeatedly
failed to demonstrate the falsity of this thesis. )heir e"idence is simply inconclusi"eI thus they should admit
that my thesis is correct. )he reasonin! in the astronomer5s ar!ument is flawed because this ar!ument
A. critici;es the astronomer5s opponents rather than their ar!uments
B. infers the truth of the astronomer5s thesis from the mere claim that it has not been pro"en false
3. i!nores the possibility that alternati"e e$planations may e$ist for the craterin!
6. presumes that the astronomer5s thesis should not be sub7ect to rational discussion and criticism
#. fails to precisely define the %ey word 8meteoroids9
-2. Some people belie"e that !ood health is due to luc%. &owe"er, studies from many countries indicate a stron!
correlation between !ood health and hi!h educational le"els. )hus research supports the "iew that !ood
health is lar!ely the result of ma%in! informed lifestyle choices. )he reasonin! in the ar!ument is most
"ulnerable to criticism on the !rounds that the ar!ument
A. presumes, without pro"idin! 7ustification that only hi!hly educated people ma%e informed lifestyle choices
B. o"erloo%s the possibility that people who ma%e informed lifestyle choices may nonetheless suffer from
inherited diseases
3. presumes, without pro"idin! 7ustification, that informed lifestyle choices are a"ailable to e"eryone
6. o"erloo%s the possibility that the same thin! may causally contribute both to education and to !ood health
#. does not ac%nowled!e that some people who fail to ma%e informed lifestyle choices are in !ood health
68
-0. Ad"ertisement* At most 7ewelry stores, the person assessin! the diamond is the person sellin! it so you can see
why an assessor mi!ht say that a diamond is of hi!her <uality than it really is. But because all diamonds sold at
>em Forld are certified in writin!, you5re assured of a fair price when purchasin! a diamond from >em Forld. )he
reasonin! in the ad"ertisement would be most stren!thened if which one of the followin! were true?
A. (any 7ewelry stores other than >em Forld also pro"ide written certification of the <uality of their diamonds.
B. )he certifications of diamonds at >em Forld are written by people with years of e$perience in appraisin! !ems.
3. )he diamonds sold at >em Forld are !enerally of hi!her <uality than those sold at other 7ewelry stores.
6. )he diamond mar%et is so "olatile that prices of the most e$pensi"e diamonds can chan!e by hundreds of
dollars from one day to the ne$t.
#. )he written certifications of diamonds at >em Forld are pro"ided by an independent company of !em specialists.
-1. Statistician* A financial ma!a;ine claimed that its sur"ey of its subscribers showed that Aorth Americans are more
concerned about their personal finances than about politics. Ene <uestion was* 8Fhich do you thin% about more*
politics or the 7oy of earnin! money?9 )his <uestion is clearly biased. Also, the readers of the ma!a;ine are a
selfselectin! sample. )hus, there is reason to be s%eptical about the conclusion drawn in the ma!a;ine5s
sur"ey. #ach of the followin!, if true, would stren!then the statistician5s ar!ument #L3#:)*
A. )he credibility of the ma!a;ine has been called into <uestion on a number of occasions.
B. )he conclusions drawn in most ma!a;ine sur"eys ha"e e"entually been dispro"ed.
3. Ether sur"eys su!!est that Aorth Americans are 7ust as concerned about politics as they are about finances.
6. )here is reason to be s%eptical about the results of sur"eys that are biased and unrepresentati"e.
#. Ether sur"eys su!!est that Aorth Americans are concerned not only with politics and finances, but also with
social issues.
-5. (odern na"i!ation systems, which are found in most of today5s commercial aircraft, are made with lowpower
circuitry, which is more susceptible to interference than the "acuumtube circuitry found in older planes. 6urin!
landin!, na"i!ation systems recei"e radio si!nals from the airport to !uide the plane to the runway. 2ecently, one
plane with lowpower circuitry "eered off course durin! landin!, its dials dimmin!, when a passen!er turned
on a laptop computer. 3learly, modern aircraft na"i!ation systems are bein! put at ris% by the electronic
de"ices that passen!ers carry on board, such as cassette players and laptop computers. Fhich one of the
followin!, if true, =#AS) stren!thens the ar!ument abo"e?
A. After the laptop computer was turned off, the plane re!ained course and its na"i!ation instruments and dials
returned to normal.
B. Fhen in use all electronic de"ices emit electroma!netic radiation, which is %nown to interfere with circuitry.
3. Ao problems with na"i!ational e<uipment or instrument dials ha"e been reported on fli!hts with no
passen!erowned electronic de"ices on board.
6. Si!nificant electroma!netic radiation from portable electronic de"ices can tra"el up to ei!ht meters, and some
passen!er seats on modern aircraft are located within four meters of the na"i!ation systems.
#. :lanes were first e<uipped with lowpower circuitry at about the same time portable electronic de"ices
became popular.
-+. Amphibian populations are declinin! in numbers worldwide. Aot coincidentally, the earth5s o;one layer has been
continuously depleted throu!hout the last 50 years. Atmospheric o;one bloc%s UOB, a type of ultra"iolet radiation
that is continuously produced by the sun, and which can dama!e !enes. Because amphibians lac% hair, hide, or
feathers to shield them, they are particularly "ulnerable to UOB radiation. In addition, their !elatinous e!!s lac% the
protection of leathery or hard shells. )hus, the primary cause of the declinin! amphibian population is the depletion
of the o;one layer. #ach of the followin!, if true, would stren!then the ar!ument #L3#:)*
A. Ef the "arious types of radiation bloc%ed by atmospheric o;one, UOB is the only type that can dama!e !enes.
B. Amphibian populations are declinin! far more rapidly than are the populations of nonamphibian species
whose tissues and e!!s ha"e more natural protection from UOB.
3. Atmospheric o;one has been si!nificantly depleted abo"e all the areas of the world in which amphibian
populations are declinin!.
6. )he natural habitat of amphibians has not become smaller o"er the past century.
#. Amphibian populations ha"e declined continuously for the last 50 years.
69
-.. Accordin! to the theory of continental drift, in prehistoric times, many of today5s separate continents were part
of a sin!le hu!e landmass. As the plates on which this landmass rested be!an to mo"e, the mass bro%e
apart, and ocean water filled the newly created chasms. It is hypothesi;ed, for e$ample, that South America
was once 7oined on its east coast with what is now the west coast of Africa. Fhich one of the followin!
disco"eries, if it were made, would most support the abo"e hypothesis about South America and Africa?
A. A lar!e band of ancient roc% of a rare type alon! the east coast of South America is of the same type as a
band on the west coast of Africa.
B. (any people today li"in! in Bra;il are !enetically <uite similar to many western Africans.
3. )he climates of western Africa and of the east coast of South America resemble each other.
6. Some of the oldest tribes of people li"in! in eastern South America spea% lan!ua!es lin!uistically similar to
"arious lan!ua!es spo%en by certain western African peoples.
#. Se"eral species of plants found in western Africa closely resemble plants !rowin! in South America.
--. (edical doctor* Sleep depri"ation is the cause of many social ills, ran!in! from irritability to potentially
dan!erous instances of impaired decision ma%in!. (ost people today suffer from sleep depri"ation to some
de!ree. )herefore we should restructure the wor%day to allow people fle$ibility in schedulin! their wor% hours.
Fhich one of the followin!, if true, would most stren!then the medical doctor5s ar!ument?
A. )he primary cause of sleep depri"ation is o"erwor%.
B. #mployees would !et more sleep if they had !reater latitude in schedulin! their wor% hours.
3. Indi"iduals "ary widely in the amount of sleep they re<uire.
6. (ore people would suffer from sleep depri"ation today than did in the past if the a"era!e number of hours
wor%ed per wee% had not decreased.
#. )he e$tent of one5s sleep depri"ation is proportional to the len!th of one5s wor%day.
-@. )o$icolo!ist* A sur"ey of oilrefinery wor%ers who wor% with (B)#, an in!redient currently used in some smo!
reducin! !asolines, found an alarmin! incidence of complaints about headaches, fati!ue, and shortness of breath.
Since !asoline containin! (B)# will soon be widely used, we can e$pect an increased incidence of headaches,
fati!ue, and shortness of breath. #ach of the followin!, if true, stren!thens the to$icolo!ist5s ar!ument #L3#:)*
A. (ost oilrefinery wor%ers who do not wor% with (B)# do not ha"e serious health problems in"ol"in!
headaches, fati!ue, and shortness of breath.
B. &eadaches, fati!ue, and shortness of breath are amon! the symptoms of se"eral medical conditions that are
potentially serious threats to public health.
3. Since the time when !asoline containin! (B)# was first introduced in a few metropolitan areas, those areas
reported an increase in the number of complaints about headaches, fati!ue, and shortness of breath.
6. 2e!ions in which only !asoline containin! (B)# is used ha"e a much !reater incidence of headaches,
fati!ue, and shortness of breath than do similar re!ions in which only (B)# free !asoline is used.
#. )he oilrefinery wor%ers sur"eyed were carefully selected to be representati"e of the broader population in
their medical histories prior to e$posure to (B)#, as well as in other rele"ant respects.
@0. >alanin is a protein found in the brain. In an e$periment, rats that consistently chose to eat fatty foods when offered
a choice between lean and fatty foods were found to ha"e si!nificantly hi!her concentrations of !alanin in
their brains than did rats that consistently chose lean o"er fatty foods. )hese facts stron!ly support the conclusion
that !alanin causes rats to cra"e fatty foods. Fhich one of the followin!, if true, most supports the ar!ument?
A. )he cra"in! for fatty foods does not in"ariably result in a rat5s choosin! those foods o"er lean foods.
B. )he brains of the rats that consistently chose to eat fatty foods did not contain si!nificantly more fat than did
the brains of rats that consistently chose lean foods.
3. )he chemical components of !alanin are present in both fatty foods and lean foods.
6. )he rats that preferred fatty foods had the hi!her concentrations of !alanin in their brains before they were
offered fatty foods.
#. 2ats that metaboli;e fat less efficiently than do other rats de"elop hi!h concentrations of !alanin in their brains.
70
@,. 4or ne$t year, the 3hefs5 Union has re<uested a ,0 percent salary increase for each of its members, whereas
the &otel (ana!ers5 Union has re<uested only an - percent salary increase for each of its members. )hese
facts demonstrate that the a"era!e dollar amount of the raises that the 3hefs5 Union has re<uested for ne$t
year is !reater than that of the raises re<uested by the &otel (ana!ers5 Union. Fhich one of the followin!, if
true, most stren!thens the ar!ument?
A. )he 3hefs5 Union has many more members than does the &otel (ana!ers5 Union.
B. )he 3hefs5 Union is a more powerful union than is the &otel (ana!ers5 Union and is therefore more li%ely to
obtain the salary increases it re<uests.
3. )he current salaries of the members of the 3hefs5 Union are, on a"era!e, hi!her than the current salaries of
the members of the &otel (ana!ers5 Union.
6. )he a"era!e dollar amount of the raises that the members of the 3hefs5 Union recei"ed last year was e<ual to
the a"era!e dollar amount of the raises that the members of the &otel (ana!ers5 Union recei"ed.
#. )he members of the 3hefs5 Union recei"ed salary increases of ,0 percent in each of the last two years, while the
members of the &otel (ana!ers5 Union recei"ed salary increases of only - percent in each of the last two years.
@2. Ad"ertisement* (ost power hed!e trimmers on the mar%et do an ade<uate 7ob of trimmin! hed!es, but many
power hed!e trimmers are dan!erous to operate and can cause serious in7ury when used by untrained
operators. Bolter Industries5 hed!e trimmer has been tested by Aational =aboratories, the most trusted name
in safety testin!. So you %now, if you buy a Bolter5s, you are buyin! a power hed!e trimmer whose safety is
assured. )he answer to which one of the followin! <uestions would be most useful in e"aluatin! the truth of
the conclusion drawn in the ad"ertisement?
A. &as Aational =aboratories performed safety tests on other machines made by Bolter Industries?
B. &ow important to the a"era!e buyer of a power hed!e trimmer is safety of operation?
3. Fhat were the results of Aational =aboratories5 tests of Bolter Industries5 hed!e trimmer?
6. Are there safer ways of trimmin! a hed!e than usin! a power hed!e trimmer?
#. 6oes any other power hed!e trimmer on the mar%et do a better 7ob of trimmin! hed!es than does Bolter
Industries5 hed!e trimmer?
@0. 3olumnist* >eor!e Erwell5s boo% ,@-1 has e$ercised much influence on a !reat number of this newspaper5s
readers. Ene thousand readers were sur"eyed and as%ed to name the one boo% that had the most influence on
their li"es. )he boo% chosen most often was the BibleI ,@-1 was second. )he answer to which one of the followin!
<uestions would most help in e"aluatin! the columnist5s ar!ument?
A. &ow many boo%s had each person sur"eyed read?
B. &ow many people chose boo%s other than ,@-1?
3. &ow many people read the columnist5s newspaper?
6. &ow many boo%s by >eor!e Erwell other than ,@-1 were chosen?
#. &ow many of those sur"eyed had actually read the boo%s they chose?
@1. Anders* )he physical structure of the brain plays an important role in thin%in!. So researchers de"elopin! 8thin%in!
machines9Gcomputers that can ma%e decisions based on both common sense and factual %nowled!eGshould
closely model those machines on the structure of the brain.
Han!* Important does not mean essential. After all, no flyin! machine closely modeled on birds has wor%edI
wor%able aircraft are structurally "ery different from birds. So thin%in! machines closely modeled on the brain are
also li%ely to fail. In de"elopin! a wor%able thin%in! machine, researchers would therefore increase their chances
of success if they focus on the brain5s function and simply i!nore its physical structure. In e"aluatin! Han!5s
ar!ument it would be most helpful to %now whether
A. studies of the physical structure of birds pro"ided information crucial to the de"elopment of wor%able aircraft
B. researchers currently wor%in! on thin%in! machines ta%e all thin%in! to in"ol"e both common sense and
factual %nowled!e
3. as much time has been spent tryin! to de"elop a wor%able thin%in! machine as had been spent in de"elopin!
the first wor%able aircraft
6. researchers who speciali;e in the structure of the brain are amon! those who are tryin! to de"elop thin%in!
machines
#. some flyin! machines that were not closely modeled on birds failed to wor%
71
@5. :ro"inces and states with strin!ent car safety re<uirements, includin! re<uired use of seat belts and annual
safety inspections, ha"e on a"era!e hi!her rates of accidents per %ilometer dri"en than do pro"inces and
states with less strin!ent re<uirements. Ae"ertheless, most hi!hway safety e$perts a!ree that more strin!ent
re<uirements do reduce accident rates. Fhich one of the followin!, if true, most helps to reconcile the safety
e$perts5 belief with the apparently contrary e"idence described abo"e?
A. Annual safety inspections ensure that car tires are replaced before they !row old.
B. 6ri"ers often become o"erconfident after their cars ha"e passed a thorou!h safety inspection.
3. )he roads in pro"inces and states with strin!ent car safety pro!rams are far more con!ested and therefore
dan!erous than in other pro"inces and states.
6. :sycholo!ical studies show that dri"ers who re!ularly wear seat belts often come to thin% of themsel"es as
serious dri"ers, which for a few people discoura!es rec%less dri"in!.
#. :ro"inces and states with strin!ent car safety re<uirements ha"e, on a"era!e, many more %ilometers of roads
then do other pro"inces and states.
@+. 3alories consumed in e$cess of those with which the body needs to be pro"ided to maintain its wei!ht are
normally stored as fat and the body !ains wei!ht. Alcoholic be"era!es are laden with calories. &owe"er,
those people who re!ularly drin% two or three alcoholic be"era!es a day and thereby e$ceed the caloric
inta%e necessary to maintain their wei!ht do not in !eneral !ain wei!ht. Fhich one of the followin!, if true,
most helps to resol"e the apparent discrepancy?
A. Some people who re!ularly drin% two or three alcoholic be"era!es a day a"oid e$ceedin! the caloric inta%e
necessary to maintain their wei!ht by decreasin! caloric inta%e from other sources.
B. #$cess calories consumed by people who re!ularly drin% two or three alcoholic be"era!es a day tend to be
dissipated as heat.
3. Some people who do not drin% alcoholic be"era!es but who eat hi!hcalorie foods do not !ain wei!ht.
6. (any people who re!ularly drin% more than three alcoholic be"era!es a day do not !ain wei!ht.
#. Some people who ta%e in fewer calories than are normally necessary to maintain their wei!ht do not lose wei!ht.
@.. Industry e$perts e$pect impro"ements in 7ob safety trainin! to lead to safer wor% en"ironments. A recent sur"ey
indicated, howe"er, that for manufacturers who impro"ed 7ob safety trainin! durin! the ,@-0s, the number of on
the7ob accidents tended to increase in the months immediately followin! the chan!es in the trainin! pro!rams.
Fhich one of the followin!, if true, most helps to resol"e the apparent discrepancy in the passa!e abo"e?
A. A similar sur"ey found that the number of on the7ob accidents remained constant after 7ob safety trainin! in
the transportation sector was impro"ed.
B. (anufacturers tend to impro"e their 7ob safety trainin! only when they are increasin! the si;e of their wor%force.
3. (anufacturers tend to impro"e 7ob safety trainin! only after they ha"e noticed that the number of onthe7ob
accidents has increased.
6. It is li%ely that the increase in the number of onthe7ob accidents e$perienced by many companies was not
merely a random fluctuation.
#. Si!nificant safety measures, such as protecti"e e<uipment and !o"ernment safety inspections, were in place
well before the impro"ements in 7ob safety trainin!.
@-. 3i!arette companies claim that manufacturin! both low and hi!hnicotine ci!arettes allows smo%ers to
choose how much nicotine they want. &owe"er, a recent study has shown that the le"els of nicotine found in
the blood of smo%ers who smo%e one pac% of ci!arettes per day are identical at the end of a day5s worth of
smo%in!, whate"er the le"el of nicotine in the ci!arettes they smo%e. Fhich one of the followin!, if true, most
helps to e$plain the findin! of the nicotine study?
A. Blood cannot absorb more nicotine per day than that found in the smo%e from a pac%a!e of the lowest
nicotine ci!arettes a"ailable.
B. Smo%ers of the lowestnicotine ci!arettes a"ailable !enerally smo%e more ci!arettes per day than smo%ers of
hi!hnicotine ci!arettes.
3. (ost nicotine is absorbed into the blood of a smo%er e"en if it is deli"ered in smaller <uantities.
6. )he le"el of tar in ci!arettes is hi!her in lownicotine ci!arettes than it is in some hi!hnicotine ci!arettes.
#. Fhen ta%in! in nicotine by smo%in! ci!arettes is discontinued, the le"el of nicotine in the blood decreases steadily.
72
@@. 2aisins are made by dryin! !rapes in the sun. Althou!h some of the su!ar in the !rapes is carameli;ed in the
process, nothin! is added. (oreo"er, the only thin! remo"ed from the !rapes is the water that e"aporates
durin! the dryin!, and water contains no calories or nutrients. )he fact that raisins contain more iron per
calorie than !rapes do is thus pu;;lin!. Fhich one of the followin!, if true, most helps to e$plain why raisins
contain more iron per calorie than do !rapes?
A. Since !rapes are bi!!er than raisins, it ta%es se"eral bunches of !rapes to pro"ide the same amount of iron
as a handful of raisins does.
B. 3arameli;ed su!ar cannot be di!ested, so its calories do not count toward the calorie content of raisins.
3. )he body can absorb iron and other nutrients more <uic%ly from !rapes than from raisins because of the
relati"ely hi!h water content of !rapes.
6. 2aisins, but not !rapes, are a"ailable year round, so many people !et a !reater share of their yearly iron
inta%e from raisins than from !rapes.
#. 2aisins are often eaten in combination with other ironcontainin! foods, while !rapes are usually eaten by themsel"es.
,00.Oer"et mon%eys use different alarm calls to warn each other of nearby predators, dependin! on whether the
dan!er comes from land or from the air. Fhich one of the followin!, if true, contributes most to an e$planation
of the beha"ior of "er"et mon%eys described abo"e?
A. By "aryin! the pitch of its alarm call, a "er"et mon%ey can indicate the number of predators approachin!.
B. 6ifferent landbased predators are responsible for different numbers of "er"et mon%ey deaths.
3. Ao predators that pose a dan!er to "er"et mon%eys can attac% both from land and from the air.
6. Oer"et mon%eys a"oid landbased predators by climbin! trees but a"oid predation from the air by di"in! into
folia!e.
#. 3ertain landbased predators feed only on "er"et mon%eys, whereas e"ery predator that attac%s "er"et
mon%eys from the air feeds on many different animals.
73
ERROR: undefined
OFFENDING COMMAND: DeleteMe
STACK:

You might also like